Questions
Questions
1. (Refer to Figure 4.) If an aircraft with a gross weight of 2,000 pounds was
subjected to a 60° constant-altitude bank, the total load would be
A. 3,000 pounds.
B. 4,000 pounds.
C. 12,000 pounds.
The load factor in a 60° bank is 2 Gs. Load factor = G load x aircraft weight.
Therefore, 2,000 x 2 = 4,000 pounds.
2. (Refer to Figure 5.) The horizontal dashed line from point C to point E represents
the
C to E is the maximum positive load limit. In this case it is 3.8 Gs, which is
appropriate for normal category airplanes.
3. To hold an airplane in level flight at airspeeds from very slow to very fast, a pilot
must coordinate thrust and
A. angle of incidence.
B. gross weight.
C. angle of attack.
https://online.prepware.com/quiz_review_graded?quiz_id=6393576 6/3/2024, 5 53 pm
Page 1 of 218
:
Straight-and-level flight may be sustained at a wide range of speeds. The pilot
coordinates angle of attack and thrust in all speed regimes if the aircraft is to be
held in level flight.
4. To maintain a standard rate turn as the airspeed increases, the bank angle of the
aircraft will need to
A. remain constant.
B. increase.
C. decrease.
As airspeed is increased in a constant bank angle the rate of turn will decrease.
For example, in a 30 degree bank at 100 knots your rate of turn will equal 6.5
degrees per second. If you maintain the 30 degree bank angle and increase speed
to 150 knots your rate of turn will decrease to 4.4 degrees per second. Therefore,
to maintain a standard rate turn of either 3 or 1.5 degrees per second (high-speed
aircraft), as airspeed is increased bank angle will need to be increased.
B. wing root, with the stall progression toward the wing tip.
center trailing edge, with the stall progression outward toward the wing root and
C.
tip.
A rectangular wing has a tendency to stall first at the wing root with the stall
pattern progressing outward to the tip. This type of stall pattern decreases
undesirable rolling tendencies and increases lateral control when approaching a
stall.
To compensate for added lift which would result if the airspeed were increased
during a turn, the angle of attack must be decreased, or the angle of bank
increased, if a constant altitude is to be maintained.
https://online.prepware.com/quiz_review_graded?quiz_id=6393576 6/3/2024, 5 53 pm
Page 2 of 218
:
Answer (A) is incorrect because either the angle of attack can be decreased or the
angle of bank increased to maintain altitude as airspeed is increased. Answer (B) is
incorrect because to maintain constant altitude in a turn as the airspeed increases,
the angle of bank must decrease.
Answer (A) is incorrect because if you are generating more lift than drag, an
increase in the angle of attack will cause the airplane to climb. Answer (C) is
incorrect because the angle of attack must be increased in order to maintain
altitude, if airspeed is being decreased.
8. To produce the same lift while in ground effect as when out of ground effect, the
airplane requires
If the airplane is brought into ground effect with a constant angle of attack, it will
experience an increase in lift coefficient and a reduction in the thrust required. The
reduction of the wing-tip vortices due to ground effect alters the spanwise lift
distribution and reduces the induced flow. The reduction in induced flow causes a
significant reduction in induced drag, but has no direct effect on parasite drag.
At lower airspeeds the angle of attack must be less to generate sufficient lift to
A.
maintain altitude.
https://online.prepware.com/quiz_review_graded?quiz_id=6393576 6/3/2024, 5 53 pm
Page 3 of 218
:
altitude.
To maintain the lift and weight forces in balance, and to keep the airplane straight-
and-level in a state of equilibrium, as velocity increases, angle of attack must be
decreased. Conversely, as the airplane slows, the decreasing velocity requires the
angle of attack be increased enough to create sufficient lift to maintain flight.
Therefore, for every angle of attack, there is a corresponding indicated airspeed
required to maintain altitude in steady, unaccelerated flight - all other factors being
constant.
Answer (A) is incorrect because to provide sufficient lift, the angle of attack must
be increased as the airspeed is reduced. Answer (C) is incorrect because the angle
of attack must be increased to compensate for the decreased lift.
10. Which statement is true, regarding the opposing forces acting on an airplane in
steady-state level flight?
B. Thrust is greater than drag and weight and lift are equal.
During straight-and-level flight at a constant airspeed, thrust and drag are equal
and lift and weight are equal.
11. In theory, if the angle of attack and other factors remain constant and the airspeed
is doubled, the lift produced at the higher speed will be
Lift is proportional to the square of the airplane's velocity. For example, an airplane
traveling at 200 knots has four times the lift as the same airplane traveling at 100
knots, if the angle of attack and other factors remain constant.
Answer (A) is incorrect because as airspeed doubles, lift will be four times greater
than at the lower speed. Answer (B) is incorrect because as airspeed doubles, lift
will be four times greater than at the lower speed.
12. (Refer to Figure 3.) The L/D ratio at a 2° angle of attack is approximately the same
as the L/D ratio for a
https://online.prepware.com/quiz_review_graded?quiz_id=6393576 6/3/2024, 5 53 pm
Page 4 of 218
:
B. 10.5° angle of attack.
The glide ratio (L/D) at a 2° angle of attack is about 7.6:1, which is the same glide
ratio (L/D) as at a 16.5° angle of attack.
13. For a given angle of bank, in any airplane, the load factor imposed in a
coordinated constant-altitude turn
Answer (B) is incorrect because rate of turn does not affect the load factor. Answer
(C) is incorrect because stall speed will increase with an increase in bank.
14. When transitioning from straight-and-level flight to a constant airspeed climb, the
angle of attack and lift
A. are increased and remain at a higher lift-to-weight ratio to maintain the climb.
B. remain the same and maintain a steady state lift-to-weight ratio during the climb.
C. are momentarily increased and lift returns to a steady state during the climb.
When transitioning from level flight to a climb, the forces acting on the airplane go
through certain changes. The first change, an increase in lift, occurs when back
pressure is applied to the elevator control. This initial change is a result of the
increase in the angle of attack which occurs when the airplane's pitch attitude is
raised. This results in a climbing attitude. When the inclined flight path and the
climb speed are established, the angle of attack and the corresponding lift
stabilize at approximately the original value.
15. If an airplane category is listed as utility, it would mean that this airplane could be
operated in which of the following maneuvers?
https://online.prepware.com/quiz_review_graded?quiz_id=6393576 6/3/2024, 5 53 pm
Page 5 of 218
:
C. Any maneuver except acrobatics or spins.
Utility category airplanes can do all normal category maneuvers plus limited
acrobatics, including spins (if approved).
Answer (A) is incorrect because the utility category includes spins. Answer (C) is
incorrect because the normal category prohibits acrobatics and spins.
16. To maintain altitude during a turn, the angle of attack must be increased to
compensate for the decrease in the
Lift during a bank is divided into two components, one vertical and the other
horizontal. The vertical component of lift must be equal to the weight to maintain
altitude. Since the vertical component of lift decreases as the bank angle
increases, the angle of attack must be progressively increased to produce
sufficient vertical lift to support the airplane's weight. The increased back elevator
pressure provides the increased angle of attack.
Answer (A) is incorrect because the phrase 'the resultant component of drag' has
no meaning. Answer (C) is incorrect because as the horizontal component of lift
decreases, the vertical component of lift increases, therefore the angle of attack
must be decreased.
17. The ratio between the total airload imposed on the wing and the gross weight of
an aircraft in flight is known as
Load factor is the ratio between the total airload imposed on the wings of an
airplane and the gross weight of the airplane. An increased load factor increases
stalling speed and a decreased load factor decreases stalling speed.
https://online.prepware.com/quiz_review_graded?quiz_id=6393576 6/3/2024, 5 53 pm
Page 6 of 218
:
C. An increase in angle of attack will decrease pressure below the wing, and increase
drag.
19. If the same angle of attack is maintained in ground effect as when out of ground
effect, lift will
If the airplane is brought into ground effect with a constant angle of attack, it will
experience an increase in lift coefficient and a reduction in the thrust required. The
reduction of the wing-tip vortices due to ground effect alters the spanwise lift
distribution and reduces the induced flow. The reduction in induced flow causes a
significant reduction in induced drag, but has no direct effect on parasite drag.
20. If the airspeed is increased from 89 knots to 98 knots during a coordinated level
45° banked turn, the load factor will
When a turn is made at a higher true airspeed at a given bank angle, the inertia is
greater and the horizontal lift component required for the turn is greater, causing
the turning rate to become slower. Therefore, at a given angle of bank, a higher
true airspeed will make the radius of turn larger because the airplane will be
turning at a slower rate. This compensates for added centrifugal force, allowing
the load factor to remain the same.
21. (Refer to Figure 4.) What is the stall speed of an airplane under a load factor of 2.5
G's if the unaccelerated stall speed is 60 knots?
A. 62 knots.
B. 84 knots.
C. 96 knots.
https://online.prepware.com/quiz_review_graded?quiz_id=6393576 6/3/2024, 5 53 pm
Page 7 of 218
:
From a load factor of 2.5, go horizontally to the curved line labeled "Load Factor."
From that point of intersection, go up to the curve labeled "Stall Speed Increase."
From there, go to the left and read the increase: 60%. The new accelerated stall
speed will be 160%, or 1.6 times the original value.
60 x 1.6 = 96 knots.
The angle of attack of an airfoil directly controls the distribution of pressure below
and above it. By changing the angle of attack, the pilot can control lift, airspeed,
and drag.
Answer (A) is incorrect because the angle of incidence is a fixed angle between the
chordline of the aircraft and the aircraft's longitudinal axis. Answer (B) is incorrect
because the amount of airflow above and below the wing stays constant.
23. Why is it necessary to increase back elevator pressure to maintain altitude during
a turn? To compensate for the
B. loss of the horizontal component of lift and the increase in centrifugal force.
Lift during a bank is divided into two components, one vertical and the other
horizontal. The vertical component of lift must be equal to the weight to maintain
altitude. Since the vertical component of lift decreases as the bank angle
increases, the angle of attack must be progressively increased to produce
sufficient vertical lift to support the airplane's weight. The increased back elevator
pressure provides the increased angle of attack.
Answer (B) is incorrect because this describes a skidding turn. Answer (C) is
incorrect because slight opposite aileron pressure may be required to compensate
for the overbanking tendency, not to maintain altitude.
24. Your flight takes you in the path of a large aircraft. In order to avoid the vortices
you should fly
https://online.prepware.com/quiz_review_graded?quiz_id=6393576 6/3/2024, 5 53 pm
Page 8 of 218
:
B. below the altitude of the large aircraft.
Fly above the jet's flight path whenever possible, because the vortices descend.
Avoid flight below and behind a large aircraft's path.
The greatest vortex strength is produced when the generating airplane is heavy,
C.
clean, and fast.
Vortices sink at 400 to 500 fpm. Vortex generation begins when lift is being
produced at takeoff. Greatest vortex strength is produced when the airplane is
heavy, clean, and slow. The primary hazard is loss of control due to induced roll
caused by the spinning vortices.
Answer (A) is incorrect because vortex generation begins at the rotation point
when the airplane takes off. Answer (C) is incorrect because the greatest vortex
strength is when the generating aircraft is heavy, clean, and slow.
26. Airplane wing loading during a level coordinated turn in smooth air depends upon
the
A. rate of turn.
B. angle of bank.
C. true airspeed.
For any given angle of bank, the rate of turn varies with the airspeed. If the angle
of bank is held constant and the airspeed is increased, the rate of turn will
decrease. Because of this, there is no change in centrifugal force for any given
bank angle. Therefore, the load factor remains the same. Load factor varies with
changing bank angle and increases at a rapid rate after the angle of bank reaches
50°.
Answer (A) is incorrect because rate of turn and true airspeed do not have an
impact on wing loading in a coordinated turn. Answer (C) is incorrect because rate
of turn and true airspeed do not have an impact on wing loading in a coordinated
turn.
https://online.prepware.com/quiz_review_graded?quiz_id=6393576 6/3/2024, 5 53 pm
Page 9 of 218
:
A. Altitude.
B. Airspeed.
C. Angle of attack.
The maximum range condition is obtained at maximum L/D ratio which occurs at a
particular angle of attack and lift coefficient. As gross weight decreases, the
airspeed for maximum L/D decreases.
Answer (A) is incorrect because as weight decreases, the maximum range altitude
may increase. Answer (C) is incorrect because angle of attack does not play a role
in determining maximum range factor.
28. In a rapid recovery from a dive, the effects of load factor would cause the stall
speed to
A. increase.
B. decrease.
C. not vary.
There is a direct relationship between the load factor imposed upon the wing and
its stalling characteristics. The stalling speed increases in proportion to the square
root of the load factor.
29. Which is true with respect to vortex circulation in the wake turbulence generated
by an aircraft?
The vortex strength is greatest when the generating aircraft is heavy, clean, and
B.
slow.
When vortex circulation sinks into ground effect, it tends to dissipate rapidly and
C.
offer little danger.
The strength of a vortex is governed by the weight, speed, and the shape of the
wing of the generating aircraft. Maximum vortex strength occurs when the
generating aircraft is heavy, clean, and slow.
30. While holding the angle of bank constant in a level turn, if the rate of turn is varied
the load factor would
A. remain constant regardless of air density and the resultant lift vector.
vary depending upon speed and air density provided the resultant lift vector varies
B.
proportionately.
https://online.prepware.com/quiz_review_graded?quiz_id=6393576 6/3/2024, 5 53 pm
Page 10 of 218
:
C. vary depending upon the resultant lift vector.
For any given angle of bank the rate of turn varies with the airspeed. In other
words, if the angle of bank is held constant and the airspeed is increased, the rate
of turn will decrease; or if the airspeed is decreased, the rate of turn will increase.
Because of this, there is no change in centrifugal force for any given bank.
Therefore, the load factor remains the same.
Answer (B) is incorrect because rate of turn will vary based on airspeed with a
constant angle of bank. Answer (C) is incorrect because load factor varies based
on the resultant load vector.
31. Recovery from a stall in any airplane becomes more difficult when its
The recovery from a stall in any airplane becomes progressively more difficult as
its center of gravity moves aft.
32. (Refer to Figure 3.) How much altitude will this airplane lose in 3 statute miles of
gliding at an angle of attack of 8°?
A. 440 feet.
B. 880 feet.
C. 1,320 feet.
To find the glide ratio (L/D) at an angle of attack of 8°, move upward from the angle
of attack scale to the L/D curve. Then move horizontally to the right to find the
value located on the L/D scale. This gives a 12:1 glide ratio. The question only
deals with glide ratio, so that is the only scale needed. With this glide ratio, the
airplane will descend 1 foot of altitude for every 12 feet covered horizontally.
L ÷ D = 12 ÷ 1 = horizontal distance ÷ vertical distance
12 ÷ 1 = 5,280 feet (3 SM) ÷ X
X = 1,320 feet.
33. If the airplane attitude initially tends to return to its original position after the
elevator control is pressed forward and released, the airplane displays
https://online.prepware.com/quiz_review_graded?quiz_id=6393576 6/3/2024, 5 53 pm
Page 11 of 218
:
C. neutral dynamic stability.
Static stability deals with initial tendencies. Positive static stability is the initial
tendency of the airplane to return to its original state after being disturbed.
34. (Refer to Figure 4.) What is the stall speed of an airplane under a load factor of 2
Gs if the unaccelerated stall speed is 60 knots?
A. 66 knots.
B. 74 knots.
C. 84 knots.
From a load factor of 2, go horizontally to the curved line labeled 'Load Factor.'
From that point of intersection, go up to the curve labeled 'Stall Speed Increase.'
From there, go to the left and read the increase: 40%. The new accelerated stall
speed will be 140%, or 1.4 times, the original value. 60 x 1.4 = 84 knots.
35. If the airspeed is increased from 90 knots to 135 knots during a level 60° banked
turn, the load factor will
At a given angle of bank, a higher airspeed will make the radius of the turn larger
and the airplane will be turning at a slower rate. This compensates for added
centrifugal force, allowing the load factor to remain the same.
36. In theory, if the airspeed of an airplane is doubled while in level flight, parasite drag
will become
A. twice as great.
B. half as great.
Parasite drag has more influence at high speed, and induced drag has more
influence at low speed. For example, if an airplane in a steady flight condition at
100 knots is then accelerated to 200 knots, the parasite drag becomes four times
as great.
37. By changing the angle of attack of a wing, the pilot can control the airplane's
https://online.prepware.com/quiz_review_graded?quiz_id=6393576 6/3/2024, 5 53 pm
Page 12 of 218
:
A. lift, airspeed, and drag.
By changing the angle of attack, the pilot can control lift, airspeed, and drag.
Answer (B) is incorrect because the angle of attack does not affect the CG.
Answer (C) is incorrect because the angle of attack also determines drag.
38. While executing a 60 degree level turn, your aircraft is at a load factor of 2.0. What
does this mean?
A. The total load on the aircraft's structure is two times its weight..
Load factor is the ratio between the total airload supported by the wing to the total
weight of the airplane; the total load supported by the wings divided by the total
weight of the airplane.
39. On a wing, the force of lift acts perpendicular to and the force of drag acts parallel
to the
A. chord line.
B. flightpath.
C. longitudinal axis.
Lift acts upward and perpendicular to the relative wind. Drag acts parallel to and in
the same direction as the relative wind, which is parallel to the flightpath.
Answer (A) is incorrect because there is not a fixed relationship between lift and
drag with respect to the airplane's chord line or longitudinal axis. Answer (C) is
incorrect because there is not a fixed relationship between lift and drag with
respect to the airplane's chord line or longitudinal axis.
Lift during a bank is divided into two components, one vertical and the other
https://online.prepware.com/quiz_review_graded?quiz_id=6393576 6/3/2024, 5 53 pm
Page 13 of 218
:
horizontal. The vertical component of lift must be equal to the weight to maintain
altitude. Since the vertical component of lift decreases as the bank angle
increases, the angle of attack must be progressively increased to produce
sufficient vertical lift to support the airplane's weight. The increased back elevator
pressure provides the increased angle of attack.
41. An aircraft wing is designed to produce lift resulting from a difference in the
A. negative air pressure below and a vacuum above the wing's surface.
vacuum below the wing's surface and greater air pressure above the wing's
B.
surface.
higher air pressure below the wing's surface and lower air pressure above the
C.
wing's surface.
The wing is designed to provide actions greater than its weight by shaping it to
develop a relatively positive high-pressure lifting action from the air mass below
the wing and a negative low-pressure lifting action from lowered pressure above
the wing. The increased speed of the air over the top of the airfoil produces the
drop in pressure. The pressure difference between the upper and lower surface
does not account for all the lift produced. Air also strikes the lower surface of the
wing, and the reaction of this downward-backward flow results in an upward-
forward force on the wing.
The indicated stalling speed is most affected by load factor. The airplane's stalling
speed increases in proportion to the square root of the load factor, whereas a
change in altitude (air density) has no effect on the indicated stalling speed.
43. In theory, if the airspeed of an aircraft in level flight is cut in half while in level flight,
parasite drag will become
A. one-third as much.
B. one-half as much.
C. one-fourth as much.
https://online.prepware.com/quiz_review_graded?quiz_id=6393576 6/3/2024, 5 53 pm
Page 14 of 218
:
44. When landing behind a large aircraft, which procedure should be followed for
vortex avoidance?
A. Stay above its final approach flightpath all the way to touchdown.
C. Stay well below its final approach flightpath and land at least 2,000 feet behind.
Stay at or above the large aircraft's final approach flight path. Note the touchdown
point and land beyond it.
Answer (A) is incorrect because roll oscillations refer to lateral stability. Answer (C)
is incorrect because roll oscillations refer to lateral stability.
46. As airspeed decreases in level flight below that speed for maximum lift/drag ratio,
total drag of an airplane
Parasite drag is greatest at higher airspeeds. Induced drag is the by-product of lift
and becomes a greater influence at higher angles of attack and slower airspeeds.
It increases in direct proportion to increases in the angle of attack. Any angle of
attack lower or higher than that for L/D(MAX) reduces the lift-drag ratio and
consequently increases the total drag.
Answer (A) is incorrect because total drag increases when airspeed decreases
below L/D(MAX) due to increased induced drag. Answer (C) is incorrect because
parasite drag decreases with speeds below L/D(MAX).
47. Which is true regarding the use of flaps during level turns?
https://online.prepware.com/quiz_review_graded?quiz_id=6393576 6/3/2024, 5 53 pm
Page 15 of 218
:
A. The lowering of flaps increases the stall speed.
C. Raising flaps will require added forward pressure on the yoke or stick.
Answer (A) is incorrect because flaps decrease the stall speed. Answer (C) is
incorrect because raising the flaps decreases the lift provided, therefore, back
pressure is required to maintain altitude.
48. (Refer to Figure 1.) At the airspeed represented by point A, in steady flight, the
airplane will
At point A, the total drag curve is at its lowest point. When an aircraft is flown at
the airspeed and angle of attack that results in the lowest total drag possible, then
the resulting L/D ratio is at its maximum.
Answer (B) is incorrect because the minimum L/D ratio occurs when parasite drag
is very high, a result of high airspeeds. Answer (C) is incorrect because the
maximum coefficient of lift occurs at slower airspeeds, which results in higher
induced drag and a lower L/D ratio.
49. If the airplane attitude remains in a new position after the elevator control is
pressed forward and released, the airplane displays
Neutral static stability is the initial tendency of the airplane to remain in the new
condition after its equilibrium has been disturbed. When an airplane's attitude is
momentarily displaced and it remains at the new attitude, it is displaying neutral
longitudinal static stability. Longitudinal stability makes an airplane stable about its
lateral axis (pitch).
Answer (B) is incorrect because positive longitudinal static stability is the initial
tendency of the airplane to return to its original attitude after the elevator control is
pressed forward and released. Answer (C) is incorrect because neutral longitudinal
dynamic stability is the overall tendency for the airplane to remain in the new
https://online.prepware.com/quiz_review_graded?quiz_id=6393576 6/3/2024, 5 53 pm
Page 16 of 218
:
condition over a period of time.
50. A load factor of 1.2 means the total load on an aircraft's structure is 1.2 times its
A. gross weight.
B. load limit.
C. gust factor.
In aerodynamics, load factor is the ratio of the maximum load an aircraft can
sustain to the gross weight of the aircraft. For example, a load factor of 1.2 means
the total load on an aircraft's structure is 1.2 times its gross weight.
The definition of a stall is when the airplane exceeds the critical angle of attack.
This happens because the smooth airflow over the airplane's wing is disrupted
and the lift degenerates rapidly. This can occur at any airspeed, in any attitude,
with any power setting.
52. During a takeoff made behind a departing large jet airplane, the pilot can minimize
the hazard of wingtip vortices by
being airborne prior to reaching the jet's flightpath until able to turn clear of its
A.
wake.
C. extending the takeoff roll and not rotating until well beyond the jet's rotation point.
Vortices begin to form when the jet rotates. Plan to be off the runway prior to
reaching the jet's point of rotation, then fly above or turn away from the jet's flight
path.
53. (Refer to Figure 2.) Select the correct statement regarding stall speeds.
A. Power-off stalls occur at higher airspeeds with the gear and flaps down.
B. In a 60° bank the airplane stalls at a lower airspeed with the gear up.
https://online.prepware.com/quiz_review_graded?quiz_id=6393576 6/3/2024, 5 53 pm
Page 17 of 218
:
Power-on stalls occur at lower airspeeds than power-off stalls because of
increased airflow over the wing and because some lift is produced by the vertical
component of thrust, reducing the lift needed to be produced by velocity. Power-
on or -off stalls occur at a lower airspeed in a shallower bank.
Answer (A) is incorrect because stall speed is lower with power-off stalls, with gear
and flaps down. Answer (B) is incorrect because the gear position alone will not
affect stall speed in a 60° bank.
54. (Refer to Figure 3.) If an airplane glides at an angle of attack of 10°, how much
altitude will it lose in 1 mile?
A. 240 feet.
B. 480 feet.
C. 960 feet.
To find the glide ratio (L/D) at an angle of attack of 10°, move upward from the
angle of attack scale to the L/D curve. Then move horizontally to the right to find
the value located on the L/D scale. This gives an 11:1 glide ratio. The question
only deals with glide ratio, so that is the only scale needed. With this glide ratio,
the airplane will descend 1 foot of altitude for every 11 feet covered horizontally.
L ÷ D = 11 ÷ 1 = horizontal distance ÷ vertical distance
11 ÷ 1 = 5,280 feet (1 SM) ÷ X
X = 480 feet
55. To avoid possible wake turbulence from a large jet aircraft that has just landed
prior to your takeoff, at which point on the runway should you plan to become
airborne?
B. At the point where the jet touched down, or just prior to this point.
C. Approximately 500 feet prior to the point where the jet touched down.
Vortices cease to be generated when the aircraft lands. Plan to become airborne
beyond this point.
reduced pressure resulting from a laminar flow over the upper camber of an airfoil,
C.
which acts perpendicular to the mean camber.
https://online.prepware.com/quiz_review_graded?quiz_id=6393576 6/3/2024, 5 53 pm
Page 18 of 218
:
Lift opposes the downward force of weight. It is produced by the dynamic effect of
the air acting on the wing, and acts perpendicular to the flight path (relative wind)
through the wing's center of lift.
57. (Refer to Figure 2.) Select the correct statement regarding stall speeds. The
airplane will stall
10 knots higher in a power-on 60° bank with gear and flaps up than with gear and
A.
flaps down.
25 knots lower in a power-off, flaps-up, 60° bank, than in a power-off, flaps-down,
B.
wings-level configuration.
10 knots higher in a 45° bank, power-on stall than in a wings-level stall with flaps
C.
up.
The stalling speed (in knots) for a power-on 60° bank, with gear and flaps up is 76
knots. For a power-on, 60° bank, and gear and flaps down, the stalling speed is 66
knots, which is 10 knots slower.
Answer (B) is incorrect because the airplane will stall 35 knots higher (not 25) in
this specified configuration. Answer (C) is incorrect because this configuration is
not specified in the question.
58. While maintaining a constant angle of bank and altitude in a coordinated turn, an
increase in airspeed will
For any given angle of bank the rate of turn varies with the airspeed. In other
words, if the angle of bank is held constant and the airspeed is increased, the rate
of turn will decrease; or if the airspeed is decreased, the rate of turn will increase.
Because of this, there is no change in centrifugal force for any given bank.
Therefore, the load factor remains the same.
Answer (A) is incorrect because load factor remains the same at a constant angle
of bank. Answer (C) is incorrect because the rate of turn in a constant angle of
bank will decrease with an increase in airspeed, and the load factor will remain the
same.
59. What is the best indicator to the pilot of the load factor on the airplane?
A. How firmly the pilot is pressed into the seat during a maneuver.
https://online.prepware.com/quiz_review_graded?quiz_id=6393576 6/3/2024, 5 53 pm
Page 19 of 218
:
B. Amount of pressure required to operate the controls.
Load factor can be detected by noting how firmly the pilot is pressed into the seat
during a maneuver. If an aircraft is pulled up from a dive, subjecting the pilot to 3
Gs, they would be pressed down into the seat with a force equal to three times
their own weight.
60. Which is true regarding the forces acting on an aircraft in a steady-state descent?
The sum of all
In steady-state flight, the sum of the opposing forces is equal to zero. The sum of
all upward forces equals the sum of all downward forces. The sum of all forward
forces equals the sum of all backward forces.
61. Load factor is the lift generated by the wings of an aircraft at any given time
Load factor is the ratio between the total airload supported by the wing to the total
weight of the airplane; the total load supported by the wings divided by the total
weight of the airplane.
Answer (B) is incorrect because load factor multiplied by airplane weight equals
required lift. Answer (C) is incorrect because load factor is lift divided by the total
weight of the airplane.
62. In small airplanes, normal recovery from spins may become difficult if the
The recovery from a stall in any airplane becomes progressively more difficult as
its center of gravity moves aft. This is particularly important in spin recovery, as
there is a point in rearward loading of any airplane at which a 'flat' spin will
https://online.prepware.com/quiz_review_graded?quiz_id=6393576 6/3/2024, 5 53 pm
Page 20 of 218
:
develop.
Answer (A) is incorrect because rotation is around the CG in a spin. Answer (C) is
incorrect because an airplane must first stall in order to spin.
63. (Refer to Figure 5.) The vertical line from point E to point F is represented on the
airspeed indicator by the
V(NE) (never exceed airspeed), the vertical line from point E to F, is marked on
airspeed indicators with a red radial line, the upper limit of the yellow arc.
64. The angle of attack at which a wing stalls remains constant regardless of
B. dynamic pressure, but varies with weight, bank angle, and pitch attitude.
C. weight and pitch attitude, but varies with dynamic pressure and bank angle.
When the angle of attack becomes so great that the air can no longer flow
smoothly over the top wing surface, it becomes impossible for the air to follow the
contour of the wing. This is the stalling or critical angle of attack. For any given
airplane, the stalling or critical angle of attack remains constant regardless of
weight, dynamic pressure, bank angle, or pitch attitude. These factors will affect
the speed at which the stall occurs, but not the angle.
Answer (B) is incorrect because the stall speed varies with weight and bank angle.
Answer (C) is incorrect because the stall speed varies with weight and bank angle.
65. Which is correct with respect to rate and radius of turn for an airplane flown in a
coordinated turn at a constant altitude?
A. For a specific angle of bank and airspeed, the rate and radius of turn will not vary.
To maintain a steady rate of turn, the angle of bank must be increased as the
B.
airspeed is decreased.
The faster the true airspeed, the faster the rate and larger the radius of turn
C.
regardless of the angle of bank.
At a specific angle of bank and a specific airspeed, the radius of the turn and the
rate of turn would remain constant if the altitude were maintained. Rate of turn
varies with airspeed, or bank angle. If the angle of bank is held constant and the
https://online.prepware.com/quiz_review_graded?quiz_id=6393576 6/3/2024, 5 53 pm
Page 21 of 218
:
airspeed is increased, the rate of turn will decrease and the radius of turn will
increase. To maintain a constant rate of turn as the airspeed is increased, the
angle of bank must be increased.
Answer (B) is incorrect because you must decrease the angle of bank when the
airspeed is decreased if you are to maintain a steady rate of turn. Answer (C) is
incorrect because for a given bank angle a faster airspeed gives a slower rate of
turn.
66. If an airplane is loaded to the rear of its CG range, it will tend to be unstable about
its
A. vertical axis.
B. lateral axis.
C. longitudinal axis.
Lateral stability is controlled by the CG along the longitudinal axis. An airplane will
become less laterally stable as the CG is moved further rearward along the
longitudinal axis. Longitudinal stability (pitching) is stability about the lateral axis.
Answer (A) is incorrect because the CG has little to do with the vertical axis.
Answer (C) is incorrect because lateral stability is not greatly affected by the CG
location.
67. (Refer to Figure 1.) At an airspeed represented by point B, in steady flight, the pilot
can expect to obtain the airplane's maximum
A. endurance.
B. glide range.
C. coefficient of lift.
Point B represents the airspeed that results in the greatest L/D ratio. At this point
the aircraft will have its maximum glide range.
Answer (A) is incorrect since only jet aircraft will obtain maximum endurance at
L/D(MAX). Answer (C) is incorrect because the critical angle of attack and the
maximum coefficient of lift occur at the same point, where total drag is also high
because of an increase in induced drag.
68. A propeller rotating clockwise as seen from the rear, creates a spiraling slipstream.
The spiraling slipstream, along with torque effect, tends to rotate the airplane to
the
A. right around the vertical axis, and to the left around the longitudinal axis.
https://online.prepware.com/quiz_review_graded?quiz_id=6393576 6/3/2024, 5 53 pm
Page 22 of 218
:
B. left around the vertical axis, and to the right around the longitudinal axis.
C. left around the vertical axis, and to the left around the longitudinal axis.
The slipstream strikes the vertical fin on the left causing a yaw to the left, at the
same time it causes a rolling moment to the left, the opposite of the direction the
propeller is spinning.
69. (Refer to Figure 4.) What increase in load factor would take place if the angle of
bank were increased from 60° to 80°?
A. 3 Gs.
B. 3.5 Gs.
C. 4 Gs.
Proceed vertically along the line above 60° angle of bank to where it intersects the
curve labeled 'Load Factor.' Next, proceed along this line to the left to the
corresponding load factor or 'G' unit, which is 2 Gs in this case. Now, repeat the
procedure using 80° of bank, which should yield a load factor of 6, which is a
difference of 4 Gs.
70. A sweptwing airplane with weak static directional stability and increased dihedral
causes an increase in
C. longitudinal stability.
When the dihedral effect is large in comparison with static direction stability, the
dutch roll motion has weak dampening and is increased.
Answer (A) is incorrect because Mach tuck tendency occurs when going through
the sound barrier. Answer (C) is incorrect because longitudinal stability is not
affected by directional stability or dihedral.
Maximum range condition would occur where the proportion between speed and
https://online.prepware.com/quiz_review_graded?quiz_id=6393576 6/3/2024, 5 53 pm
Page 23 of 218
:
power required is greatest. The maximum range condition (of propeller driven
airplanes) is obtained at maximum lift-drag ratio (L/D(MAX)). The best angle of
glide is one that allows the airplane to travel the greatest distance over the ground
with the least loss of altitude. This is also the airplane's maximum L/D and is
usually expressed as a ratio. This implies that the airplane should be flown at
L/D(MAX) to obtain the greatest glide distance.
Answer (A) is incorrect because ground friction is reduced when breaking ground.
Answer (C) is incorrect because a higher angle of attack is required to maintain the
same lift coefficient when leaving the ground.
74. To generate the same amount of lift as altitude is increased, an airplane must be
flown at
https://online.prepware.com/quiz_review_graded?quiz_id=6393576 6/3/2024, 5 53 pm
Page 24 of 218
:
A. the same true airspeed regardless of angle of attack.
In order to maintain its lift at a higher altitude, an airplane must fly at a greater true
airspeed for any given angle of attack.
75. If the airspeed is decreased from 98 knots to 85 knots during a coordinated level
45 degree banked turn, the load factor will
At a given angle of bank, a lower airspeed will make the radius of the turn smaller
and the airplane will be turning at a faster rate. This compensates for the reduced
centrifugal force, allowing the load factor to remain the same.
76. To increase the rate of turn and at the same time decrease the radius, a pilot
should
The horizontal component of lift will equal the centrifugal force of steady, turning
flight. To increase the rate of turn, the angle of bank may be increased and the
airspeed may be decreased.
77. The need to slow an aircraft below V(A) is brought about by the following weather
phenomenon:
https://online.prepware.com/quiz_review_graded?quiz_id=6393576 6/3/2024, 5 53 pm
Page 25 of 218
:
B. Turbulence which causes an increase in stall speed.
Answer (A) is incorrect because indicated stall speed is not affected by changes in
density altitude. Answer (C) is incorrect because the higher load factors imposed
on the aircraft by turbulence increases stall speed.
78. (Refer to Figure 5.) What does the intersection of the dashed line at point C
represent?
A. Va.
Point C represents the intersection of the positive limit load factor and the line of
maximum positive lift capability. The airspeed at this point is the minimum
airspeed at which the limit load can be developed aerodynamically. Any airspeed
greater than this provides a positive lift capability sufficient to damage the aircraft.
Conversely, any airspeed less than this does not provide positive lift capability
sufficient to cause damage from excessive flight loads. This point is commonly
referred to as maneuvering speed or V(A).
79. Longitudinal stability involves the motion of the airplane controlled by its
A. rudder.
B. elevator.
C. ailerons.
Longitudinal stability or pitching is the motion around the lateral axis. Pitch is
controlled by the elevator.
Answer (A) is incorrect because the rudder affects directional stability. Answer (C)
is incorrect because the ailerons affect lateral stability.
https://online.prepware.com/quiz_review_graded?quiz_id=6393576 6/3/2024, 5 53 pm
Page 26 of 218
:
80. One of the main functions of flaps during the approach and landing is to
Extending the flaps increases wing lift and also increases induced drag. The
increased drag enables the pilot to make steeper approaches without an increase
in airspeed.
Answer (A) is incorrect because the flaps will increase the angle of descent without
increasing the airspeed. Answer (C) is incorrect because the flaps increase lift and
induced drag.
81. An aircraft airfoil is designed to produce lift resulting from a difference in the
A. negative air pressure below and a vacuum above the airfoil's surface.
vacuum below the airfoil's surface and greater air pressure above the airfoil's
B.
surface.
higher air pressure below the airfoil's surface and lower air pressure above the
C.
airfoil's surface.
The highest velocity is at the top of the airfoil with the lowest velocity at the
bottom. Because there is a difference of velocity above and below the wing, the
result is a higher pressure at the bottom of the wing and a lower pressure on the
top of the wing. This low-pressure area produces an upward force known as the
Magnus effect, the physical phenomenon whereby an object's rotation affects its
path through a fluid, including air.
82. The uncontrolled firing of the fuel/air charge in advance of normal spark ignition is
known as
A. instantaneous combustion.
B. detonation.
C. pre-ignition.
When a cylinder head gets too hot, it can ignite the fuel/air mixture before the
spark. This condition is called preignition.
https://online.prepware.com/quiz_review_graded?quiz_id=6393576 6/3/2024, 5 53 pm
Page 27 of 218
:
with temperatures that are too high.
Answer (A) is incorrect because when power is being decreased, the manifold
pressure should be reduced before reducing the RPM. Answer (C) is incorrect
because when power is being decreased, the manifold pressure should be
reduced before reducing the RPM.
If the fuel/air mixture is too rich, excessive fuel consumption, rough engine
operation, and appreciable loss of power will occur. Because of excessive fuel, a
cooling effect takes place which causes below normal temperatures in the
combustion chambers. This cooling results in spark plug fouling. Unless the
mixture is leaned with a gain in altitude, the mixture becomes excessively rich.
85. During preflight in cold weather, crankcase breather lines should receive special
attention because they are susceptible to being clogged by
https://online.prepware.com/quiz_review_graded?quiz_id=6393576 6/3/2024, 5 53 pm
Page 28 of 218
:
C. ice from crankcase vapors that have condensed and subsequently frozen.
The crankcase breather requires special consideration when preparing for cold
weather. Frozen breather lines can create numerous problems. When crankcase
vapors cool, they may condense in the breather line and subsequently freeze it
closed. Special care is recommended during the preflight to ensure that the
breather system is free of ice.
Answer (A) is incorrect because oil that lays in the bottom of the crankcase never
gets into the breather lines. Answer (B) is incorrect because a low air temperature
is usually associated with a low moisture content.
86. Before shutdown, while at idle, the ignition key is momentarily turned OFF. The
engine continues to run with no interruption; this
is normal because the engine is usually stopped by moving the mixture to idle cut-
A.
off.
B. should not normally happen. Indicates a magneto not grounding in OFF position.
Answer (A) is incorrect because the engine should stop when the ignition key is
turned to the OFF position. Answer (C) is incorrect because this indicates there is
a faulty ground wire.
C. The propeller control regulates the engine RPM and in turn the propeller RPM.
The propeller control regulates the engine RPM and in turn, the propeller RPM.
The RPM is registered on the tachometer.
Answer (A) is incorrect because the prop governor causes the pitch angle of the
prop blades to change to help maintain a specified airspeed. Answer (B) is
incorrect because a high blade angle will increase propeller drag with less engine
power.
https://online.prepware.com/quiz_review_graded?quiz_id=6393576 6/3/2024, 5 53 pm
Page 29 of 218
:
88. Leaving the carburetor heat on while taking off
Use of carburetor heat enriches the mixture, which tends to reduce the output of
the engine and also increases the operating temperature. Therefore, the heat
should not be used when full power is required (such as during takeoff) or during
normal engine operations except to check for the presence of, or removal of
carburetor ice. A decrease in engine output will increase distance required to
reach lift off speed. Therefore, it will increase ground roll.
89. Unless adjusted, the fuel/air mixture becomes richer with an increase in altitude
because the amount of fuel
Fuel flow remains constant if no adjustments are made. The same volume of air
goes into the carburetor, but the weight and density of the air is less, causing an
excessively rich mixture, which causes spark plug fouling and decreased power.
90. What should be expected when making a downwind landing? The likelihood of
In this situation, the pilot should aim at the near end of the runway, because of the
tailwind increasing the aircraft's ground speed.
91. Your aircraft has an exhaust manifold type heating system. The exhaust manifold
should be periodically inspected to avoid
Carbon monoxide poisoning from exhaust gases leaking into the cockpit from a
https://online.prepware.com/quiz_review_graded?quiz_id=6393576 6/3/2024, 5 53 pm
Page 30 of 218
:
faulty exhaust manifold has been linked to several fatal aircraft accidents.
A. a 'rich' mixture.
Detonation is a sudden explosion or shock to a small area of the piston top, rather
than the normal smooth burn in the combustion chamber. It can be caused by low
grade fuel or a lean mixture.
93. What will occur if no leaning is made with the mixture control as the flight altitude
increases?
The volume of air entering the carburetor decreases and the amount of fuel
A.
decreases.
The density of air entering the carburetor decreases and the amount of fuel
B.
increases.
The density of air entering the carburetor decreases and the amount of fuel
C.
remains constant.
Fuel flow remains constant if no adjustments are made. The same volume of air
goes into the carburetor, but the weight and density of the air is less, causing an
excessively rich mixture, which causes spark plug fouling and decreased power.
94. For internal cooling, reciprocating aircraft engines are especially dependent on
Answer (A) is incorrect because although cowl flaps aid internal cooling, they are
not the primary cooling source. Answer (C) is incorrect because the proper
freon/compressor output ratio controls cabin cooling.
https://online.prepware.com/quiz_review_graded?quiz_id=6393576 6/3/2024, 5 53 pm
Page 31 of 218
:
95. To develop maximum power and thrust, a constant-speed propeller should be set
to a blade angle that will produce a
Smaller angle of attack makes the blades take smaller amounts of air, which in
turn allows the engine to run at higher RPM, producing more power.
A low-pitch, high RPM setting is utilized to obtain maximum power for takeoff.
Then, after the airplane is airborne, an increasing blade angle/pitch will cause
lower RPM which provides adequate thrust and better economy while maintaining
the proper airspeed.
B. add full power, while holding the brakes, and momentarily turn off the ignition.
C. run on one magneto, lean the mixture, and look for a rise in manifold pressure.
Answer (B) is incorrect because it is not necessary to add full power when
performing the check. Answer (C) is incorrect because the way to detect a broken
magneto ground wire is to turn the ignition to the OFF position; if the engine
continues to run, the problem is confirmed.
https://online.prepware.com/quiz_review_graded?quiz_id=6393576 6/3/2024, 5 53 pm
Page 32 of 218
:
A. ratio of thrust horsepower to brake horsepower.
Since the efficiency of any machine is the ratio of useful power output to actual
power input, propeller efficiency is the ratio of thrust horsepower to brake
horsepower.
Answer (B) is incorrect because effective pitch is the actual distance a propeller
advances in one revolution. Answer (C) is incorrect because the ratio of geometric
pitch to effective pitch is called slippage.
Rapid throttle operation can induce detonation, which may detune the crankshaft.
Answer (B) is incorrect because carburetor ice can cause the engine to stop
running, but it will not affect the engine crankshaft counterweights. Answer (C) is
incorrect because operating with an excessively rich mixture fouls the spark plugs,
but does not affect the crankshaft.
B. the most power can be obtained for any given throttle setting.
a given power can be obtained with the highest manifold pressure or throttle
C.
setting.
The throttle setting determines the amount of air flowing into the engine. The
mixture control is then adjusted to get the best fuel/air ratio, resulting in the best
power the engine can develop at this particular throttle setting.
Answer (A) is incorrect because the cylinder heads will be the coolest when
mixture is richest. Answer (C) is incorrect because this describes the highest
power setting.
https://online.prepware.com/quiz_review_graded?quiz_id=6393576 6/3/2024, 5 53 pm
Page 33 of 218
:
A. volume of fuel and volume of air entering the cylinder.
The mixture control is used to change the fuel to air mixture entering the
combustion chamber (cylinder). Fuel-to-air ratio is the weight of fuel to a given
weight of air.
Answer (A) is incorrect because, as altitude increases, the amount of air in a fixed
volume decreases. Answer (C) is incorrect because the carburetor is where the
fuel/air ratio is established prior to entering the cylinders.
102. The basic purpose of adjusting the fuel/air mixture control at altitude is to
decrease the amount of fuel in the mixture to compensate for increased air
B.
density.
increase the amount of fuel in the mixture to compensate for the decrease in
C.
pressure and density of the air.
Fuel flow remains constant if no adjustments are made. The same volume of air
goes into the carburetor, but the weight and density of the air is less, causing an
excessively rich mixture, which causes spark plug fouling and decreased power.
A. engine to overheat.
Fuel flow remains constant if no adjustments are made. The same volume of air
goes into the carburetor, but the weight and density of the air is less, causing an
excessively rich mixture, which causes spark plug fouling and decreased power.
Answer (A) is incorrect because a lean mixture will cause the engine to overheat.
Answer (C) is incorrect because an engine runs smoother when the mixture is
adjusted for the altitude.
B. the spark plugs receive an electrical jolt caused by a short in the wiring.
https://online.prepware.com/quiz_review_graded?quiz_id=6393576 6/3/2024, 5 53 pm
Page 34 of 218
:
C. the unburned fuel/air charge in the cylinders is subjected to instantaneous
combustion.
Detonation is a sudden explosion or instantaneous combustion of the fuel/air
mixture in the cylinders, producing extreme heat and severe structural stresses on
the engine.
105. A fixed-pitch propeller is designed for best efficiency only at a given combination
of
Answer (A) is incorrect because altitude does not affect propeller efficiency.
Answer (C) is incorrect because altitude does not affect propeller efficiency.
106. Which is true regarding preheating an aircraft during cold weather operations?
C. Hot air should be blown directly at the engine through the air intakes.
Low temperatures may cause a change in the viscosity of engine oils, batteries
may lose a high percentage of their effectiveness, and instruments may stick.
Because of this, preheating the engines as well as the cabin before starting is
desirable in low temperatures. Extreme caution should be used in the preheat
process to avoid fire.
107. The reason for variations in geometric pitch (twisting) along a propeller blade is
that it
permits a relatively constant angle of incidence along its length when in cruising
A.
flight.
B. prevents the portion of the blade near the hub from stalling during cruising flight.
C. permits a relatively constant angle of attack along its length when in cruising flight.
https://online.prepware.com/quiz_review_graded?quiz_id=6393576 6/3/2024, 5 53 pm
Page 35 of 218
:
Twisting, or variations in the geometric pitch of the blades, permits the propeller to
operate with a relatively constant angle of attack along its length when in cruising
flight.
108. If the ground wire between the magneto and the ignition switch becomes
disconnected, the engine
C. could accidentally start if the propeller is moved with fuel in the cylinder.
Answer (A) is incorrect because both magnetos remain on when the ground wire is
disconnected. Answer (B) is incorrect because the engine can still be started, and
the magnetos cannot be turned off.
Use of carburetor heat enriches the mixture which tends to reduce the output of
the engine and also increases the operating temperature.
110. The most probable reason an engine continues to run after the ignition switch has
been turned off is
https://online.prepware.com/quiz_review_graded?quiz_id=6393576 6/3/2024, 5 53 pm
Page 36 of 218
:
Answer (A) is incorrect because glowing carbon deposits is a result of preignition.
Answer (B) is incorrect because a magneto ground wire should be in contact with
the engine casing to provide grounding.
C. the fuel mixture is ignited too early by hot carbon deposits in the cylinder.
Answer (B) is incorrect because detonation may occur with an excessively lean
fuel mixture and a loss in power. Answer (C) is incorrect because this describes
preignition.
112. For takeoff, the blade angle of a controllable-pitch propeller should be set at a
Smaller angle of attack makes the blades take smaller amounts of air, which in
turn allows the engine to run at higher RPM, producing more power.
A. throttle.
B. manifold pressure.
C. mixture control.
The fuel/air ratio of the combustible mixture delivered to the engine is controlled
by the mixture control.
Answer (A) is incorrect because the throttle regulates the total volume of fuel and
air entering the combustion chamber. Answer (B) is incorrect because the manifold
pressure indicates the engine's power output.
114. Which statement is true concerning the effect of the application of carburetor
heat?
https://online.prepware.com/quiz_review_graded?quiz_id=6393576 6/3/2024, 5 53 pm
Page 37 of 218
:
A. It enriches the fuel/air mixture.
Use of carburetor heat enriches the mixture which tends to reduce the output of
the engine and also increases the operating temperature.
A. prevents the fuel/air combination from becoming too rich at higher altitudes.
C. prevents the fuel/air combination from becoming lean as the airplane climbs.
As the aircraft climbs, the fuel/air mixture becomes richer and the excessive fuel
causes the engine to lose power and to run rougher. The mixture control provides
a means for the pilot to decrease fuel to compensate for this imbalance in mixture
as altitude increases.
Answer (B) is incorrect because the throttle regulates the airflow through the
carburetor's venturi. Answer (C) is incorrect because the fuel/air ratio becomes
richer as the aircraft climbs.
A. a defective bearing.
The oil pressure indication varies inversely with the oil temperature. High
temperature and low-pressure usually indicate low oil level.
Answer (A) is incorrect because a defective bearing will increase metal particles in
the oil, but will not significantly affect the oil temperature. Answer (C) is incorrect
because a rich mixture results in lower engine operating temperatures; therefore, it
would not increase engine oil temperature.
117. Which is the correct symbol for the stalling speed or the minimum steady flight
speed in a specified configuration?
A. V(S).
B. V(S1).
https://online.prepware.com/quiz_review_graded?quiz_id=6393576 6/3/2024, 5 53 pm
Page 38 of 218
:
C. V(S0).
V(S1) is the stalling speed or the minimum steady flight speed obtained in a
specific configuration.
Answer (A) is incorrect because V(S) is the stalling speed or the minimum steady
flight speed at which the airplane is controllable. Answer (C) is incorrect because
V(S0) is the stalling speed or the minimum steady flight speed in the landing
configuration.
118. You are flying an aircraft equipped with an electronic flight display and the air data
computer fails. What instrument is affected?
A. ADS-B In capability.
B. Airspeed indicator.
C. Attitude indicator.
The pitot-static inputs are received by the air data computer (ADC). If the ADC
fails, the airspeed indicator will be affected.
119. What altimeter setting is required when operating an aircraft at 18,000 feet MSL?
B. 29.92 "Hg.
Each person operating an aircraft shall maintain the cruising altitude or flight level
of that aircraft by reference to an altimeter that is set, when operating at or above
18,000 feet MSL, to 29.92 "Hg.
120. Which is the best technique for minimizing the wing-load factor when flying in
severe turbulence?
Control airspeed with power, maintain wings level, and accept variations of
B.
altitude.
https://online.prepware.com/quiz_review_graded?quiz_id=6393576 6/3/2024, 5 53 pm
Page 39 of 218
:
variations caused by gusts.
Answer (B) is incorrect because the flap operating range is indicated by the white
arc on the airspeed indicator. Answer (C) is incorrect because V(FE) is the
maximum flap extended speed.
B. instrument error.
C. non-standard temperature.
Calibrated airspeed is the indicated airspeed corrected for position (or installation),
and instrument errors.
123. The ratio of an airplane's true airspeed to the speed of sound in the same
atmospheric conditions is
A. equivalent airspeed.
B. transonic airflow.
C. mach number.
Mach number means the ratio of true airspeed to the speed of sound.
124. What is an advantage of an electric turn coordinator if the airplane has a vacuum
system for other gyroscopic instruments?
https://online.prepware.com/quiz_review_graded?quiz_id=6393576 6/3/2024, 5 53 pm
Page 40 of 218
:
C. It will not tumble as will vacuum-driven turn indicators.
An electric turn coordinator provides a backup in case the vacuum system fails.
Answer (B) is incorrect because both the vacuum-driven and electrically driven
indicators are reliable, and both can tumble. Answer (C) is incorrect because both
the vacuum-driven and electrically driven indicators are reliable, and both can
tumble.
B. never-exceed speed.
Answer (A) is incorrect because this speed is not defined in 14 CFR Part 1. Answer
(C) is incorrect because this is V(LE).
126. Maximum structural cruising speed is the maximum speed at which an airplane
can be operated during
A. abrupt maneuvers.
B. normal operations.
The maximum structural cruising speed (V(NO)) is the speed at which exceeding
the load limit factor may cause permanent deformation of the airplane structure.
This is the maximum speed for normal operation.
Answer (A) is incorrect because design maneuvering speed (V(A)) is the maximum
speed for abrupt maneuvers. Answer (C) is incorrect because the yellow arc
identifies the range where flight is only recommended in smooth air.
127. An airplane is located at an airport with an elevation of 5,000 feet MSL and a
temperature of 90 degrees F. The altimeter is set to airport elevation. Later that
night the temperature plummets to 50 degrees F. Unless the altimeter setting is
changed, it will read
A. 4,800 feet.
B. 5,000 feet.
C. 5,200 feet.
https://online.prepware.com/quiz_review_graded?quiz_id=6393576 6/3/2024, 5 53 pm
Page 41 of 218
:
A decrease in air temperature will increase the density of the air and decrease the
density altitude of a given airport. If the altimeter setting isn't adjusted for the
change in pressure, the altimeter will read higher than the field elevation.
129. Which is the correct symbol for the stalling speed or the minimum steady flight
speed at which the airplane is controllable?
A. V(S).
B. V(S1).
C. V(S0).
V(S) is the stalling speed or the minimum steady flight speed at which the airplane
is controllable.
Answer (B) is incorrect because V(S1) is the stalling speed or the minimum steady
flight speed obtained in a specified configuration. Answer (C) is incorrect because
V(S0) is the stalling speed or the minimum steady flight speed in the landing
configuration.
Answer (B) is incorrect because V(LO) is the maximum landing gear operating
speed. Answer (C) is incorrect because maximum leading edge flaps extended
speed is not defined in 14 CFR Part 1.
https://online.prepware.com/quiz_review_graded?quiz_id=6393576 6/3/2024, 5 53 pm
Page 42 of 218
:
B. Design limit load factors may be exceeded, if gusts are encountered.
Any speed above V(NE) can cause damage; therefore, flight above this speed
should be avoided even in smooth air.
Answer (A) is incorrect because induced drag decreases with increased airspeed.
Answer (C) is incorrect because control effectiveness increases with increased
airspeed.
132. Newer airplanes have a design maneuvering speed that can generally be
calculated as follows:
A. 1.2 Vso.
B. 1.7 Vso.
The maximum speed at which an aircraft may be stalled safely is now determined
for all new designs. This speed is called the "design maneuvering speed" (VA) and
must be entered in the AFM/POH of all recently designed aircraft. For older
general aviation aircraft, this speed is approximately 1.7 times the normal stalling
speed. For example, an older aircraft which normally stalls at 60 knots must never
be stalled at above 102 knots (60 knots x 1.7 = 102 knots). An aircraft with a
normal stalling speed of 60 knots stalled at 102 knots undergoes a load factor
equal to the square of the increase in speed, or 2.89 Gs (1.7 x 1.7 = 2.89 Gs).
133. A pilot is entering an area where significant clear air turbulence has been reported.
Which action is appropriate upon encountering the first ripple?
In an area where significant clear air turbulence (CAT) has been reported or is
forecast, it is suggested that the pilot adjust the speed to fly at the recommended
rough air speed on encountering the first ripple, since the intensity of such
turbulence may build up rapidly.
134. What is a consideration when using a hand-held GPS for VFR navigation?
https://online.prepware.com/quiz_review_graded?quiz_id=6393576 6/3/2024, 5 53 pm
Page 43 of 218
:
A. Position accuracy may degrade without notification.
While a hand-held GPS receiver can provide excellent navigation capability to VFR
pilots, be prepared for intermittent loss of navigation signal, possibly with no RAIM
warning to the pilot.
135. Structural damage or failure is more likely to occur in smooth air at speeds above
A. V(NO).
B. V(A).
C. V(NE).
Any speed above V(NE) can cause damage; therefore, flight above this speed
should be avoided even in smooth air.
136. What is an operational difference between the turn coordinator and the turn-and-
slip indicator? The turn coordinator
indicates bank angle only; the turn-and-slip indicator indicates rate of turn and
B.
coordination.
indicates roll rate, rate of turn, and coordination; the turn-and-slip indicator
C.
indicates rate of turn and coordination.
The turn coordinator indicates roll rate in addition to rate of turn and coordination.
The turn-and-slip indicator only indicates rate of turn and coordination.
Answer (A) is incorrect because both these instruments are usually electrically
driven. Answer (B) is incorrect because a turn coordinator does not indicate bank
angle.
A. Propeller stall.
B. Reduction in drag.
Critical Mach number is the boundary between subsonic and transonic flight.
When shock waves form on the aircraft, airflow separation followed by buffet and
aircraft control difficulties can occur. Shock waves, buffet, and airflow separation
https://online.prepware.com/quiz_review_graded?quiz_id=6393576 6/3/2024, 5 53 pm
Page 44 of 218
:
take place above critical Mach number.
138. If severe turbulence is encountered during flight, the pilot should reduce the
airspeed to
B. design-maneuvering speed.
Design maneuvering speed (V(A)) is the maximum speed at which the maximum
load limit can be imposed (either by gust or full deflection of the control surfaces)
without causing structural damage.
Answer (A) is incorrect because in turbulence, minimum control speed would result
in the airplane stalling or significant control problems. Answer (C) is incorrect
because the maximum structural cruising speed is faster than V(A).
139. (Refer to Figure 5.) The vertical line from point D to point G is represented on the
airspeed indicator by the maximum speed limit of the
A. green arc.
B. yellow arc.
C. white arc.
The high speed limit of the green arc is the maximum speed for normal operation.
This is designated as maximum structural cruising speed (V(NO)), which is line D
to G.
Answer (B) is incorrect because the high speed limit of the yellow arc is the never
exceed speed (V(NE)), which is line E to F. Answer (C) is incorrect because the high
speed limit of the white arc is the maximum flap extended speed (V(FE)), which is
not indicated on the chart.
Answer (A) is incorrect because this is not defined in 14 CFR Part 1. Answer (B) is
incorrect because this is V(X).
https://online.prepware.com/quiz_review_graded?quiz_id=6393576 6/3/2024, 5 53 pm
Page 45 of 218
:
141. True airspeed is best described as calibrated airspeed corrected for
B. non-standard temperature.
True airspeed is indicated airspeed after it has been corrected for nonstandard
temperature and pressure altitude.
143. If a standard rate turn is maintained, how long would it take to turn 360°?
A. 1 minute.
B. 2 minutes.
C. 3 minutes.
A standard rate turn means the aircraft is turning at a rate of 3° per second. 360°
divided by 3° per second is equal to 120 seconds, or 2 minutes.
144. Which airspeed would a pilot be unable to identify by the color coding of an
airspeed indicator?
Answer (A) is incorrect because the never-exceed speed is identified by the red
line at the high-speed end of the yellow arc. Answer (B) is incorrect because the
power-off stall speed is identified by the low-speed end of the white arc for
https://online.prepware.com/quiz_review_graded?quiz_id=6393576 6/3/2024, 5 53 pm
Page 46 of 218
:
landing configuration (V(SO)), and the low-speed end of the green arc for clean
configuration (V(S1)).
145. To determine pressure altitude prior to takeoff, the altimeter should be set to
C. the field elevation and the pressure reading in the altimeter setting window noted.
Pressure altitude can be determined by setting the altimeter to 29.92 ''Hg and
reading the altimeter indication.
Answer (A) is incorrect because these would indicate field elevation, or true
altitude. Answer (C) is incorrect because these would indicate field elevation, or
true altitude.
Deviation depends, in part, on the heading of the aircraft. The difference between
the direction indicated by a magnetic compass not installed in an airplane, and
one that is installed in an airplane, is deviation.
Answer (A) is incorrect because variation varies over time as the agonic line shifts.
Answer (C) is incorrect because deviation varies from aircraft to aircraft.
147. What period of time must a person be hospitalized before an injury may be defined
by the NTSB as a "serious injury"?
"Serious injury" means any injury requiring hospitalization for more than 48 hours,
commencing within 7 days from the date the injury was received.
https://online.prepware.com/quiz_review_graded?quiz_id=6393576 6/3/2024, 5 53 pm
Page 47 of 218
:
A. receive and log ground and flight training from a qualified pilot in such an airplane.
obtain an endorsement from a qualified pilot stating that the person is proficient to
B.
operate such an airplane.
receive and log ground and flight training from an authorized instructor in such an
C.
airplane.
To act as pilot-in-command of a high-performance airplane (an airplane with an
engine of more than 200 horsepower), the person must receive and log ground
and flight training from an authorized instructor in a high-performance airplane and
has been found proficient in the operation and systems of the airplane. In addition,
the pilot must receive a one-time endorsement in the pilot's logbook from an
authorized instructor who certifies that the person is proficient to operate a high-
performance airplane. The training and endorsement is not required if the person
has logged flight time as pilot-in-command of a high-performance airplane prior to
August 4, 1997.
Answer (A) is incorrect because the required flight and ground training must be
given by an authorized instructor, not a qualified pilot. Answer (B) is incorrect
because the required flight and ground training must be given by an authorized
instructor, not a qualified pilot.
Operations for compensation or hire are not authorized in limited category aircraft,
experimental aircraft, and restricted category aircraft. Operations using utility
category aircraft (such as Cessna 152) are authorized.
Answer (A) is incorrect because the carriage of passengers for hire is permitted in
normal and utility category aircraft. Answer (C) is incorrect because the carriage of
passengers for hire is not permitted in restricted category aircraft.
150. During preflight, you discover one of the passenger seats has a defective shoulder
harness. All of the seats will be occupied. This flight is
A. allowed.
B. not allowed.
No pilot may cause to be moved on the surface, takeoff, or land a U.S.- registered
https://online.prepware.com/quiz_review_graded?quiz_id=6393576 6/3/2024, 5 53 pm
Page 48 of 218
:
civil aircraft unless the PIC of that aircraft ensures that each person on board has
been notified to fasten their safety belt and, if installed, their shoulder harness.
151. No person may operate a large civil aircraft of U.S. registry which is subject to a
lease, unless the lessee has mailed a copy of the lease to the FAA Aircraft
Registration Branch, Technical Section, Oklahoma City, OK within how many hours
of its execution?
A. 24.
B. 48.
C. 72.
The lessee must mail a copy of the lease to the Aircraft Registry Technical Section,
Box 25724, Oklahoma City, Oklahoma, 73125, within 24 hours of its execution.
152. The operator of an aircraft that has been involved in an incident is required to
submit a report to the nearest field office of the NTSB
A. within 7 days.
B. within 10 days.
Answer (A) is incorrect because 7 days is the time limitation for reporting overdue
(missing) aircraft. Answer (B) is incorrect because 10 days is the limitation on filing
a report for accidents.
153. You are PIC of a flight and determine that the aircraft you planned to fly has an
overdue Airworthiness Directive (AD). Which of the following is an appropriate
decision?
A. No maintenance is available so you wait until after the trip to comply with the AD.
C. You cancel the flight and have the aircraft scheduled for maintenance.
154. Before beginning any flight under IFR, the pilot in command must become familiar
with all available information concerning that flight. In addition, the pilot must
https://online.prepware.com/quiz_review_graded?quiz_id=6393576 6/3/2024, 5 53 pm
Page 49 of 218
:
A. be familiar with all instrument approaches at the destination airport.
list an alternate airport on the flight plan and confirm adequate takeoff and landing
B.
performance at the destination airport.
be familiar with the runway lengths at airports of intended use, weather reports,
C.
fuel requirements, the alternatives available, if the flight cannot be completed.
For a flight under IFR or flight not in the vicinity of an airport, the pilot must be
familiar with weather reports and forecasts, fuel requirements, alternatives
available if the planned flight cannot be completed, and any known traffic delays
of which the pilot has been advised by ATC. For any flight, the pilot must be
familiar with runway lengths at airports of intended use, and the following takeoff
and landing distance information: For civil aircraft for which an approved airplane
or rotorcraft flight manual containing takeoff and landing distance data is required,
the takeoff and landing distance data contained therein.
Answer (A) is incorrect because only pilots filing an IFR flight must be familiar with
the instrument approaches at their destination airport and possible alternates.
Answer (B) is incorrect because an alternate is not required if weather is VFR at the
destination.
155. To act as pilot in command of an airplane that is equipped with retractable landing
gear, flaps, and controllable-pitch propeller, a person is required to
make at least six takeoffs and landings in such an airplane within the preceding 6
A.
months.
receive and log ground and flight training in such an airplane, and obtain a
B.
logbook endorsement certifying proficiency.
No person may act as PIC of a complex airplane (an airplane that has a retractable
landing gear, flaps, and a controllable pitch propeller), unless the person has
received and logged ground and flight training from an authorized instructor in a
complex airplane, or in a flight simulator or flight training device that is
representative of a complex airplane, and has been found proficient in the
operation and systems of the airplane; and received a one-time endorsement in
the pilot's logbook from an authorized instructor who certifies the person is
proficient to operate a complex airplane.
156. A pilot convicted for the violation of any Federal or State statute relating to the
process, manufacture, transportation, distribution, or sale of narcotic drugs is
grounds for
a written report to be filed with the FAA Civil Aviation Security Division (AMC-700)
A.
not later than 60 days after the conviction.
https://online.prepware.com/quiz_review_graded?quiz_id=6393576 6/3/2024, 5 53 pm
Page 50 of 218
:
notification of this conviction to the FAA Civil Aeromedical Institute (CAMI) within
B.
60 days after the conviction.
suspension or revocation of any certificate, rating, or authorization issued under
C.
14 CFR Part 61.
A conviction for the violation of any federal or state statute relating to the growing,
processing, manufacture, sale, disposition, possession, transportation, or
importation of narcotic drugs, marijuana, or depressant or stimulant drugs or
substances is grounds for: (1) denial of an application for any certificate, rating, or
authorization for a period of up to 1 year after the date of final conviction; or (2)
suspension or revocation of any certificate, rating, or authorization.
157. During a night operation, the pilot of aircraft #1 sees only the green light of aircraft
#2. If the aircraft are converging, which pilot has the right-of-way? The pilot of
aircraft
When aircraft of the same category are converging at approximately the same
altitude (except head-on, or nearly so), the aircraft to the other's right has the
right-of-way. The green light indicates aircraft #1 is looking at the right wing of
aircraft #2, which means aircraft #1 is to the right of aircraft #2.
159. In what type of operation, not regulated by 14 CFR Part 119, may a commercial
pilot act as pilot in command and receive compensation for services?
https://online.prepware.com/quiz_review_graded?quiz_id=6393576 6/3/2024, 5 53 pm
Page 51 of 218
:
Nonstop flights within a 25 SM radius of an airport to carry persons for intentional
B.
parachute jumps.
Part 119 does not apply to nonstop flights conducted within a 25 SM radius of the
airport of takeoff carrying persons or objects for the purpose of conducting
intentional parachute operations.
Answer (A) is incorrect because these operations are regulated by Part 119.
Answer (C) is incorrect because these operations are regulated by Part 119.
C. only during takeoff and landing when passengers are aboard the aircraft.
Required flight crewmembers must keep their safety belts fastened while at their
stations.
161. Portable electronic devices which may cause interference with the navigation or
communication system may not be operated on a U.S.-registered civil aircraft
being flown
Answer (A) is incorrect because this rule pertains only to IFR and commercial
flights. Answer (B) is incorrect because this rule pertains only to IFR and
commercial flights.
https://online.prepware.com/quiz_review_graded?quiz_id=6393576 6/3/2024, 5 53 pm
Page 52 of 218
:
A person holding at least a Private Pilot Certificate may approve an aircraft for
return to service after performing preventative maintenance under the provisions
of 14 CFR §43.3(g) and shall make an entry in the maintenance record of that
maintenance.
163. To act as pilot in command of an airplane towing a glider, the tow pilot is required
to have
a logbook endorsement from an authorized glider instructor certifying receipt of
A. ground and flight training in gliders, and be proficient with techniques and
procedures for safe towing of gliders.
at least a private pilot certificate with a category rating for powered aircraft, and
B. made and logged at least three flights as pilot or observer in a glider being towed
by an airplane.
a logbook record of having made at least three flights as sole manipulator of the
C.
controls of a glider being towed by an airplane.
To act as pilot-in-command of an aircraft towing a glider, a person must have a
logbook endorsement from an authorized instructor who certifies that the person
has received ground and flight training in gliders and is proficient in the techniques
and procedures essential to the safe towing of gliders, including airspeed
limitations.
Answer (B) is incorrect because the PIC must have logged at least three flights as
the sole manipulator of the controls, not as an observer, of an aircraft towing a
glider or simulating glider-towing flight procedures while accompanied by an
authorized pilot. Answer (C) is incorrect because any person who, before May 17,
1967, has made and logged 10 or more flights as PIC of an aircraft towing a glider
in accordance with a certificate of waiver need not comply with this rule.
A safety link at each end of the towline which has a breaking strength not less than
C.
80% of the aircraft's gross weight.
Answer (A) is incorrect because approval from ATC to operate in Class E airspace
is only required in IFR conditions. Answer (C) is incorrect because the towline
breaking strength requirement is for towing gliders.
https://online.prepware.com/quiz_review_graded?quiz_id=6393576 6/3/2024, 5 53 pm
Page 53 of 218
:
165. Regulations which refer to the operational control of a flight are in relation to
Operational control, with respect to a flight, means the exercise of authority over
initiating, conducting, or terminating a flight.
Answer (A) is incorrect because assigning specific duties to a crew member is only
a small part of exercising operational control. Answer (B) is incorrect because
acting as sole manipulator of an aircraft is a flight crew responsibility.
166. Portable electronic devices which may cause interference with the navigation or
communication system may not be operated on U.S.-registered civil aircraft being
operated
A. under IFR.
Any portable electronic device that the operator of the aircraft has determined
could cause interference with the navigation or communication systems may not
be used on any aircraft operated under IFR.
167. You are pilot-in-command of a VFR flight that you think will be within the fuel
range of your aircraft. As part of your preflight planning you must
list an alternate airport on the flight plan and confirm adequate takeoff and landing
B.
performance at the destination airport.
C. obtain weather reports, forecasts, and fuel requirements for the flight.
Before beginning a flight under IFR or a flight not in the vicinity of an airport, each
PIC shall become familiar with all available information concerning that flight,
including weather reports and forecasts, fuel requirements, available alternatives if
the planned flight cannot be completed, and any known traffic delays of which the
pilot in command has been advised by ATC.
168. You are taking a 123 nautical mile VFR flight from one airport to another. Which of
the following actions must the pilot in command take?
https://online.prepware.com/quiz_review_graded?quiz_id=6393576 6/3/2024, 5 53 pm
Page 54 of 218
:
B. Verify the airworthiness certificate is legible to passengers.
169. Which is true with respect to formation flights? Formation flights are
A. not authorized, except by arrangement with the pilot in command of each aircraft.
not authorized, unless the pilot in command of each aircraft is trained and found
B.
competent in formation.
authorized when carrying passengers for hire, with prior arrangement with the pilot
C.
in command of each aircraft in the formation.
170. Unless otherwise authorized, the pilot in command is required to hold a type rating
when operating any
A person may not act as pilot-in-command of any of the following aircraft unless
they hold a type rating for that aircraft:
1. A large aircraft (except lighter-than-air), more than 12,500 pounds maximum
certificated takeoff weight.
2. A helicopter, for operations requiring an airline transport pilot certificate.
3. A turbojet-powered airplane.
4. Other aircraft specified by the Administrator through aircraft type certificate
procedures.
171. The pilot in command of an aircraft operated under IFR, in controlled airspace,
shall report as soon as practical to ATC when
https://online.prepware.com/quiz_review_graded?quiz_id=6393576 6/3/2024, 5 53 pm
Page 55 of 218
:
A. climbing or descending to assigned altitudes.
172. What transponder equipment is required for airplane operations within Class B
airspace? A transponder
with 4096 code capability is required except when operating at or below 1,000 feet
B.
AGL under the terms of a letter of agreement.
173. Which is true with respect to operating near other aircraft in flight? They are
not authorized, when operated so close to another aircraft they can create a
A.
collision hazard.
not authorized, unless the pilot in command of each aircraft is trained and found
B.
competent in formation.
authorized when carrying passengers for hire, with prior arrangement with the pilot
C.
in command of each aircraft in the formation.
No person may operate an aircraft so close to another aircraft as to create a
collision hazard. No person may operate an aircraft in formation flight except by
arrangement with the pilot-in-command of each aircraft in the formation. No
person may operate an aircraft, carrying passengers for hire, in formation flight.
174. Your transponder is inoperative. In order to enter Class B airspace, you must
submit a request for a deviation from the
https://online.prepware.com/quiz_review_graded?quiz_id=6393576 6/3/2024, 5 53 pm
Page 56 of 218
:
B. nearest FSDO 24 hours before the proposed operation.
C. controlling ATC facility at any time prior to entering the controlled airspace.
A. ADs are advisory in nature and are, generally, not addressed immediately.
Answer (A) is incorrect because ADs are mandatory. Answer (C) is incorrect
because compliance with ADs, along with other maintenance regulations, is the
responsibility of the owner/operator.
176. Approved flotation gear, readily available to each occupant, is required on each
aircraft if it is being flown for hire over water,
Answer (A) is incorrect because the flotation gear requirement applies to all aircraft
operated for hire when flying beyond power-off gliding distance from shore.
Answer (C) is incorrect because flotation gear is not required if the aircraft remains
within power-off gliding distance from shore.
177. To act as pilot in command of a tailwheel airplane, without prior experience, a pilot
must
https://online.prepware.com/quiz_review_graded?quiz_id=6393576 6/3/2024, 5 53 pm
Page 57 of 218
:
receive and log flight training from an authorized instructor as well as receive a
C.
logbook endorsement from an authorized instructor who finds the person
proficient in a tailwheel airplane.
No person may act as pilot-in-command of a tailwheel airplane unless that person
has received and logged flight training from an authorized instructor in a tailwheel
airplane and received an endorsement in the person's logbook from an authorized
instructor who found the person proficient in the operation of a tailwheel airplane.
B. after dark.
No person may during the period from sunset to sunrise operate an aircraft that is
equipped with an anticollision light system, unless it has lighted anticollision lights.
However, the anticollision lights need not be lighted when the pilot in command
determines that, because of operating conditions, it would be in the interest of
safety to turn the lights off.
179. Aircraft maintenance records must include the current status of the
Each registered owner or operator shall keep records containing the following
information: the current status of life-limited parts of each airframe, engine,
propeller, rotor, and appliance.
180. Which incident would require that the nearest NTSB field office be notified
immediately?
A. In-flight fire.
Fire of the primary aircraft while in a hangar which results in damage to other
C.
property of more than $25,000.
The operator of an aircraft shall immediately and by the most expeditious means
available notify the nearest NTSB field office of any inflight fire.
https://online.prepware.com/quiz_review_graded?quiz_id=6393576 6/3/2024, 5 53 pm
Page 58 of 218
:
Answer (B) is incorrect because the regulation specifies inflight fires only. Answer
(C) is incorrect because the regulation specifies inflight fires only.
181. Two aircraft of the same category are approaching an airport for the purpose of
landing. The right-of-way belongs to the aircraft
at the lower altitude, but the pilot shall not take advantage of this rule to cut in
B.
front of or to overtake the other aircraft.
that is more maneuverable, and that aircraft may, with caution, move in front of or
C.
overtake the other aircraft.
When two or more aircraft are approaching an airport for the purpose of landing,
the lower aircraft has the right-of-way. However, a pilot shall not take advantage of
this rule to overtake or cut in front of another aircraft that is on final approach to
land.
182. While in flight a helicopter and an airplane are converging at a 90° angle, and the
helicopter is located to the right of the airplane. Which aircraft has the right-of-
way, and why?
When aircraft of the same category are converging at approximately the same
altitude (except head-on, or nearly so) the aircraft to the other's right has the right-
of-way. Aircraft and rotorcraft are treated as equally maneuverable, so neither
aircraft has right-of-way over the other.
183. The maximum cumulative time that an emergency locator transmitter may be
operated before the rechargeable battery must be recharged is
A. 30 minutes.
B. 45 minutes.
C. 60 minutes.
ELT batteries must be replaced or recharged when the transmitter has been in use
for more than one cumulative hour.
https://online.prepware.com/quiz_review_graded?quiz_id=6393576 6/3/2024, 5 53 pm
Page 59 of 218
:
184. What are the oxygen requirements when operating at cabin pressure altitudes
above 15,000 feet MSL?
Answer (A) is incorrect because the flight crew must use oxygen above 14,000 feet
MSL. Answer (B) is incorrect because oxygen requirements apply to all aircraft.
185. On a post flight inspection of your aircraft after an aborted takeoff due to an
elevator malfunction, you find that the elevator control cable has broken.
According to NTSB 830, you
Immediate notification is required by the operator of any civil aircraft when a flight
control system malfunction or failure occurs.
186. What is the general direction of movement of the other aircraft if during a night
flight you observe a steady white light and a rotating red light ahead and at your
altitude? The other aircraft is
The pilot is seeing the rotating beacon and the white light on the tail. The other
aircraft is headed away.
https://online.prepware.com/quiz_review_graded?quiz_id=6393576 6/3/2024, 5 53 pm
Page 60 of 218
:
Answer (B) is incorrect because you would see a steady red light if the other
aircraft was crossing to your left. Answer (C) is incorrect because you would see
both the red and green wing-tip position lights if the other aircraft were
approaching you head-on.
187. What person is directly responsible for the final authority as to the operation of the
airplane?
A. Certificate holder.
B. Pilot in command.
C. Airplane owner/operator.
188. Which is required equipment for powered aircraft during VFR night flights?
An approved anticollision light system is required for flight under VFR between
sunset and sunrise.
Each aircraft that is being overtaken has the right-of-way, and each pilot of an
overtaking aircraft shall alter course to the right to pass well clear.
190. To act as PIC of a high performance airplane, which training or experience would
meet the additional requirements?
https://online.prepware.com/quiz_review_graded?quiz_id=6393576 6/3/2024, 5 53 pm
Page 61 of 218
:
airplane in the last 12 calendar months.
Received and logged ground and flight training in an airplane with retractable
B.
landing gear, flaps, and controllable-pitch propeller.
Received and logged ground and flight training in a high performance airplane and
C.
a received a logbook endorsement.
To act as pilot-in-command of a high-performance airplane (an airplane with an
engine of more than 200 horsepower), the pilot must receive and log ground and
flight training from an authorized instructor in a high-performance airplane and
must be found proficient in the operation and systems of the airplane. In addition,
the pilot must receive a one-time endorsement in the pilot's logbook from an
authorized instructor who certifies that the pilot is proficient in operating a high-
performance airplane. The training and endorsement is not required if the pilot has
logged flight time as pilot-in-command of a high-performance airplane prior to
August 4, 1997.
Aircraft class ratings, with respect to airmen, include single-engine land, multi-
engine land, single-engine sea, and multi-engine sea, helicopter, gyroplane,
airship, and free balloon.
Answer (A) is incorrect because these are aircraft categories with respect to the
certification of aircraft. Answer (B) is incorrect because these are categories of
aircraft with respect to airmen.
https://online.prepware.com/quiz_review_graded?quiz_id=6393576 6/3/2024, 5 53 pm
Page 62 of 218
:
night is prohibited.'
C. An Owner's Manual.
194. No person may operate an aircraft in simulated instrument flight conditions unless
the
B. pilot has filed an IFR flight plan and received an IFR clearance.
other control seat is occupied by a safety pilot, who holds at least a private pilot
C.
certificate and is appropriately rated.
No person may operate a civil aircraft in simulated instrument flight unless the
other control seat is occupied by a safety pilot who possesses at least a private
pilot certificate with category and class ratings appropriate to the aircraft being
flown.
195. A coded transponder equipped with altitude reporting equipment is required for
1. Class A, Class B, and Class C airspace areas.
2. all airspace of the 48 contiguous U.S. and the District of Columbia at and above
10,000 feet MSL (including airspace at and below 2,500 feet above the surface).
A. 1.
B. 2.
C. Both 1 and 2.
https://online.prepware.com/quiz_review_graded?quiz_id=6393576 6/3/2024, 5 53 pm
Page 63 of 218
:
excluding airspace at and below 2,500 feet AGL. Answer (C) is incorrect because a
transponder is required in all airspace of the 48 contiguous U.S. and the District of
Columbia at and above 10,000 feet MSL, excluding airspace at and below 2,500
feet AGL.
196. Pilots who change their permanent mailing address and fail to notify the FAA
Airmen Certification Branch of this change, are entitled to exercise the privileges of
their pilot certificate for a period of
A. 30 days.
B. 60 days.
C. 90 days.
The holder of a pilot or flight instructor certificate who has made a change in their
permanent mailing address may not after 30 days from the date they moved,
exercise the privileges of their certificate unless they have notified the FAA.
197. After an annual inspection has been completed and the aircraft has been returned
to service, an appropriate notation should be made
198. A person with a commercial pilot certificate may act as pilot in command of an
aircraft carrying persons or property for compensation or hire, if that person
holds appropriate category, class ratings, and meets the recent flight experience
A.
requirements of 14 CFR Part 61.
is qualified in accordance with 14 CFR Part 61 and with the applicable parts that
B.
apply to the operation.
is qualified in accordance with 14 CFR Part 61 and has passed a pilot competency
C.
check given by an authorized check pilot.
Not only must the pilot be qualified in accordance with Part 61, the pilot must also
https://online.prepware.com/quiz_review_graded?quiz_id=6393576 6/3/2024, 5 53 pm
Page 64 of 218
:
comply with Part 91, the regulations which govern the operation of the flight.
199. Commercial pilots are required to have a valid and appropriate pilot certificate in
their physical possession or readily accessible in the aircraft when
Answer (A) is incorrect because having the appropriate pilot and medical
certificates is required regardless of the type of operation. Answer (B) is incorrect
because having the appropriate pilot and medical certificates is required
regardless of the type of operation.
200. If an ATC transponder installed in an aircraft has not been tested, inspected, and
found to comply with regulations within a specified period, what is the limitation on
its use?
No person may use an ATC transponder unless, within the preceding 24 calendar
months, that ATC transponder has been tested, inspected and found to comply
with regulations.
201. You have accomplished 25 takeoffs and landings in multi-engine land airplanes in
the previous 45 days. For a flight you plan to conduct today, this meets the PIC
recency of experience requirements to carry passengers in which airplanes?
No person may act as PIC of an aircraft carrying passengers unless, within the
preceding 90 days, he/she has made three takeoffs and landings as the sole
manipulator of the controls in an aircraft of the same category, class and type (if
https://online.prepware.com/quiz_review_graded?quiz_id=6393576 6/3/2024, 5 53 pm
Page 65 of 218
:
required).
202. For the purpose of airworthiness, a dealer registration certificate is the same as
the owner's certificate when
203. You are planning a trip and one of your passengers states that he prefers not to
use his shoulder harness because it is uncomfortable. You should
explain that it is a mandatory requirement and that he use the shoulder harness
A.
during takeoff, landing, and movement on the surface.
B. allow him to use his seat belt for the entire trip without the shoulder harness.
allow him to use his seat belt for takeoff and landing and the shoulder harness
C.
while en route.
If installed, a shoulder harness must be worn during movement on the surface,
takeoff, and landing.
204. If weather conditions are such that it is required to designate an alternate airport
on your IFR flight plan, you should plan to carry enough fuel to arrive at the first
airport of intended landing, fly from that airport to the alternate airport, and fly
thereafter for
Civil airplanes in IFR conditions must have sufficient fuel (considering weather
reports, forecasts, and conditions) to fly to the first airport of intended landing,
then to an alternate, then to fly for 45 minutes at normal cruising speed.
205. With U.S.-registered civil airplanes, the use of safety belts is required during
movement on the surface, takeoffs, and landings for
https://online.prepware.com/quiz_review_graded?quiz_id=6393576 6/3/2024, 5 53 pm
Page 66 of 218
:
B. each person over 2 years of age on board.
No pilot may cause to be moved on the surface, takeoff, or land a U.S.- registered
civil aircraft unless the PIC of that aircraft ensures that each person on board has
been notified to fasten their safety belt and, if installed, their shoulder harness. A
person who has not reached their second birthday may be held by an adult.
206. When may an aircraft equipped with ADS-B Out not operate in transmit mode?
If the aircraft is being operated for law enforcement purposes and transmitting
B.
would jeopardize the safety of the mission.
Answer (A) is incorrect because regardless of airspace or whether the aircraft was
originally certificated with an electrical system, it must operate any installed ADS-
B Out equipment in transmit mode. Answer (C) is incorrect because regardless of
airspace or whether the aircraft was originally certificated with an electrical
system, it must operate any installed ADS-B Out equipment in transmit mode.
208. Which is required equipment for powered aircraft during VFR night flights?
An electric landing light is required only if the night flight is for hire.
https://online.prepware.com/quiz_review_graded?quiz_id=6393576 6/3/2024, 5 53 pm
Page 67 of 218
:
Answer (A) is incorrect because there is no specific requirement for flashlights and
the color of the lens. Answer (C) is incorrect because sensitive altimeters are only
required for IFR flight.
209. Unless otherwise authorized by ATC, which airspace requires the appropriate
Automatic Dependent Surveillance-Broadcast (ADS-B) Out equipment installed?
Above the ceiling and within the lateral boundaries of Class D airspace up to
B.
10,000 feet MSL.
Any airspace that requires the use of a transponder also requires aircraft to be
equipped with an ADS-B Out system. ADS-B is required when operating within 30
NM of an airport listed in appendix D of Part 91 from the surface up to 10,000 feet
MSL.
210. Which of the following preflight actions is the pilot in command required to take in
order to comply with the United States Code of Federal Regulations regarding day
Visual Flight Rules (VFR)?
Answer (A) is incorrect because it is not mandatory to file a flight plan. Answer (C)
is incorrect because regulations only specify approved position lights must be
installed.
211. When is preflight action required, relative to alternatives available, if the planned
flight cannot be completed?
https://online.prepware.com/quiz_review_graded?quiz_id=6393576 6/3/2024, 5 53 pm
Page 68 of 218
:
For a flight under IFR or flight not in the vicinity of an airport, the pilot must be
familiar with weather reports and forecasts, fuel requirements, alternatives
available if the planned flight cannot be completed, and any known traffic delays
of which the pilot has been advised by ATC.
A. 6 calendar months.
B. 12 calendar months.
C. 24 calendar months.
213. In accordance with 14 CFR Part 91, supplemental oxygen must be used by the
required minimum flightcrew for that time exceeding 30 minutes while at cabin
pressure altitudes of
214. Each required flight crewmember is required to keep his or her shoulder harness
fastened
A. during takeoff and landing only when passengers are aboard the aircraft.
while the crewmembers are at their stations, unless he or she is unable to perform
https://online.prepware.com/quiz_review_graded?quiz_id=6393576 6/3/2024, 5 53 pm
Page 69 of 218
:
B. required duties.
C. during takeoff and landing, unless he or she is unable to perform required duties.
Answer (A) is incorrect because the shoulder harness must be used on takeoff and
landing regardless of the passenger complement. Answer (B) is incorrect because
the regulation specifies shoulder harness use during takeoff and landing.
215. While taxiing on the parking ramp, the landing gear, wheel, and tire are damaged
by striking ground equipment. What action would be required to comply with
NTSB Part 830?
An immediate notification must be filed by the operator of the aircraft with the
A.
nearest NTSB field office.
B. A report must be filed with the nearest FAA field office within 7 days.
216. You are conducting your preflight of an aircraft and notice that the last inspection
of the emergency locator transmitter was 11 calendar months ago. You may
ELTs must be inspected within 12 calendar months after the last inspection.
217. An airplane is converging with a helicopter. Which aircraft has the right-of-way?
https://online.prepware.com/quiz_review_graded?quiz_id=6393576 6/3/2024, 5 53 pm
Page 70 of 218
:
B. The aircraft on the right.
If aircraft of the same category (or an airplane and rotorcraft) are converging, the
aircraft to the other's right has the right-of-way.
written report to be filed with the FAA Civil Aviation Security Division (AMC-700)
A.
not later than 60 day after the conviction.
written notification to the FAA Civil Aeromedical Institute (CAMI) within 60 days
B.
after the conviction.
denial of an application for an FAA certificate, rating, or authorization issued under
C.
14 CFR part 61.
No person may act or attempt to act as a crewmember of a civil aircraft within 8
hours after the consumption of any alcoholic beverage, while under the influence
of alcohol, while using any drug that affects the person's faculties in any way
contrary to safety; or while having 0.04 percent by weight or more alcohol in the
blood. Committing any of these acts is grounds for: (1) denial of an application for
a certificate, rating, or authorization for a period of up to 1 year after the date of
that act; or (2) suspension or revocation of any certificate, rating, or authorization.
219. What flight time must be documented and recorded, by a pilot exercising the
privileges of a commercial certificate?
Flight time showing training and aeronautical experience to meet requirements for
A.
a certificate, rating, or flight review.
Only flight time for compensation or hire with passengers aboard which is
C.
necessary to meet the recent flight experience requirements.
The aeronautical training and experience needed to meet the requirements for a
certificate or rating, and/or the recent flight experience requirements, must be
documented and recorded. The logging of other flight time is not required.
A. 125 feet.
B. 50 feet.
C. 25 feet.
https://online.prepware.com/quiz_review_graded?quiz_id=6393576 6/3/2024, 5 53 pm
Page 71 of 218
:
The difference between the automatic reporting output and the altitude displayed
at the altimeter shall not exceed 125 feet.
within the last 12 months become familiar with the required information, and
C. perform and log pilot time in the type of airplane for which privileges are
requested.
Under Part 91 no person may serve as SIC of an aircraft type certificated for more
than one required pilot flight crewmember unless that person has within the
previous 12 calendar months become familiar with information for the specific type
aircraft for which SIC privileges are requested and performed and logged pilot
time in the type of aircraft or in a flight simulator that represents the type of aircraft
for which SIC privileges are requested. Also, no person may serve as a SIC of an
aircraft type certificated for more than one required pilot flight crewmember unless
that person holds at least a current Private Pilot Certificate with the appropriate
category and class rating.
222. What is the maximum indicated airspeed authorized in the airspace underlying
Class B airspace?
A. 156 knots.
B. 200 knots.
C. 230 knots.
223. What action must be taken when a pilot in command deviates from any rule in 14
CFR Part 91?
Upon landing, report the deviation to the nearest FAA Flight Standards District
A.
Office.
Upon the request of the Administrator, send a written report of that deviation to
C.
the Administrator.
https://online.prepware.com/quiz_review_graded?quiz_id=6393576 6/3/2024, 5 53 pm
Page 72 of 218
:
Each pilot-in-command who deviates from a rule in an emergency shall, upon
request, send a written report of that deviation to the Administrator.
224. You are acting as a commercial pilot, but are not operating under the regulations
of 14 CFR Part 119. Which of these operations are you authorized to conduct?
Part 119 does not apply to crop dusting, seeding, spraying, and bird chasing.
Answer (B) is incorrect because these operations are regulated by Part 119.
Answer (C) is incorrect because these operations are regulated by Part 119.
B. Yes, it expires at the end of the 24th month after the month in which it was issued.
226. Regulations which refer to "commercial operators" relate to that person who
227. A pilot convicted of a motor vehicle offense involving alcohol or drugs is required
to provide a written report to the
nearest FAA Flight Standards District Office (FSDO) within 60 days after such
A.
action.
https://online.prepware.com/quiz_review_graded?quiz_id=6393576 6/3/2024, 5 53 pm
Page 73 of 218
:
B. FAA Civil Aeromedical Institute (CAMI) within 60 days after the conviction.
C. FAA Civil Aviation Security Division (AMC-700) within 60 days after such action.
Each person holding a pilot certificate shall provide a written report of each motor
vehicle action to the FAA, Civil Aviation Security Division (AMC-700), not later than
60 days after the motor vehicle action.
228. Who is responsible for determining if an aircraft is in condition for safe flight?
No person may carry any person (meaning passengers) other than crewmembers
in an aircraft that has been maintained, rebuilt, or altered in a manner that may
have appreciably changed its flight characteristics or substantially affected its
operation in flight, until an appropriately rated pilot with at least a Private Pilot
Certificate flies the aircraft, makes an operational check of the maintenance
performed or alteration made, and logs the flight in the aircraft records.
230. Which list accurately reflects some of the documents required to be current and
carried in a U.S. registered civil airplane flying in the United States under day
Visual Flight Rules (VFR)?
Proof of insurance certificate, VFR flight plan or flight itinerary, and the aircraft
A.
logbook.
VFR sectional(s) chart(s) for the area in which the flight occurs, aircraft logbook,
B.
and engine logbook.
https://online.prepware.com/quiz_review_graded?quiz_id=6393576 6/3/2024, 5 53 pm
Page 74 of 218
:
C. certificate.
231. According to 14 CFR Part 91, at what minimum altitude may an airplane be
operated unless necessary for takeoff and landing?
In congested areas, you must maintain 500 feet over obstacles, and no closer than
A.
500 feet to any person, vessel, vehicle, or structure.
In uncongested areas, 1,000 feet over any obstacle within a horizontal radius of
B.
2,000 feet.
An altitude allowing for an emergency landing without undue hazard, if a power
C.
unit fails.
Except when necessary for takeoff or landing, no person may operate an aircraft
below an altitude allowing, if a power unit fails, an emergency landing without
undue hazard to persons or property on the surface.
Answer (A) is incorrect because except when necessary for takeoff or landing, no
person may operate an aircraft over any congested area of a city, town, or
settlement, or over any open air assembly of persons, below an altitude of 1,000
feet above the highest obstacle within a horizontal radius of 2,000 feet of the
aircraft. Answer (B) is incorrect because except when necessary for takeoff or
landing, no person may operate an aircraft over other than congested areas below
an altitude of 500 feet above the surface except over open water or sparsely
populated areas. In that case, the aircraft may not be operated closer than 500
feet to any person, vessel, vehicle, or structure.
232. A Second Class Medical Certificate issued to a commercial pilot on April 10, this
year, permits the pilot to exercise which of the following privileges?
C. Private pilot privileges through, but not after, March 31, next year.
A Second Class Medical Certificate expires at the end of the last day of:
1. The 12th month after the month of the date of examination shown on the
certificate, for operations requiring a Commercial Pilot Certificate or an Air Traffic
https://online.prepware.com/quiz_review_graded?quiz_id=6393576 6/3/2024, 5 53 pm
Page 75 of 218
:
Control Tower Operator Certificate, and
2. The 24th or 60th (depending on the applicant's age) month after the month of
the date of examination shown on the certificate, for operations requiring a
Private, Recreational, or Student Pilot Certificate.
Answer (B) is incorrect because a Second Class Medical Certificate is valid for
commercial operations for 12 months, and expires on the last day of the month.
Answer (C) is incorrect because a Second Class Medical Certificate is valid for
private pilot operations for 24 or 60 months (depending on the applicant's age).
233. When is the pilot in command required to hold a category and class rating
appropriate to the aircraft being flown?
Unless a person holds a category, class, and type rating (if a class and type rating
is required) that applies to the aircraft, that person may not act as pilot-in-
command of an aircraft that is carrying another person, or is operated for
compensation or hire.
Answer (A) is incorrect because a pilot can solo an aircraft with an authorized
instructor's logbook endorsement. Answer (B) is incorrect because a pilot may act
as pilot-in-command when testing for a category and class rating with a duly
authorized examiner.
All flight time when qualified and occupying a crewmember station in an aircraft
B.
that requires more than one pilot.
Only that flight time during which the second-in-command is the sole manipulator
C.
of the controls.
A person may log SIC time only for that flight time during which that person is
qualified in accordance with the SIC requirements of this part, and occupies a
crewmember station in an aircraft that requires more than one pilot by the
aircraft's type certificate.
235. Who is responsible for filing a Near Midair Collision (NMAC) Report?
https://online.prepware.com/quiz_review_graded?quiz_id=6393576 6/3/2024, 5 53 pm
Page 76 of 218
:
B. Local law enforcement.
It is the responsibility of the pilot and/or flight crew to determine whether a near
midair collision did actually occur and, if so, to initiate an NMAC report.
236. During flight a fire which was extinguished burned the insulation from a transceiver
wire. What action is required by regulations?
A report must be filed with the avionics inspector at the nearest FAA Flight
B.
Standards District Office within 48 hours.
An immediate notification by the operator of the aircraft to the nearest NTSB field
C.
office.
The operator of an aircraft shall immediately and by the most expeditious means
available notify the nearest NTSB field office of any in-flight fire.
237. No pilot may act as pilot in command of an aircraft under IFR or in weather
conditions less than the minimums prescribed for VFR unless that pilot has, within
the past 6 months, performed and logged under actual or simulated instrument
conditions, at least
six instrument approaches, holding procedures, intercepting and tracking courses,
A. or passed an instrument proficiency check in an aircraft that is appropriate to the
aircraft category.
To act as pilot-in-command under IFR, the pilot must, within the preceding six
calendar months, have performed and logged under actual or simulated
instrument conditions at least six instrument approaches, holding procedures, and
intercepting and tracking of courses through the use of navigation systems.
https://online.prepware.com/quiz_review_graded?quiz_id=6393576 6/3/2024, 5 53 pm
Page 77 of 218
:
C. Owner or operator of the aircraft.
Answer (A) is incorrect because mechanics work under the advisement of the
owner/operator. Answer (B) is incorrect because the pilot is only responsible for
determining airworthiness before flight.
A. pilot in command and the FAA certificated mechanic assigned to that aircraft.
Answer (A) is incorrect because the pilots are responsible for determining
airworthiness before flight, and the mechanics work under the advisement of the
owner/operator. Answer (B) is incorrect because the pilots are responsible for
determining airworthiness before flight, and the mechanics work under the
advisement of the owner/operator.
240. Notification to the NTSB is required when there has been substantial damage
The operator of an aircraft shall immediately, and by the most expeditious means
available, notify the nearest NTSB field office when an aircraft accident occurs. An
aircraft accident is an occurrence associated with the operation of an aircraft
which takes place between the time any person boards the aircraft with the
intention of flight and all such persons have disembarked, and in which any
person suffers death or serious injury, or in which the aircraft receives substantial
damage which adversely affects the structural strength, performance, or flight
characteristics of the aircraft.
241. The required preflight action relative to weather reports and fuel requirements is
applicable to
https://online.prepware.com/quiz_review_graded?quiz_id=6393576 6/3/2024, 5 53 pm
Page 78 of 218
:
A. any flight conducted for compensation or hire.
Before beginning a flight under IFR or a flight not in the vicinity of an airport, each
PIC shall become familiar with all available information concerning that flight,
including weather reports and forecasts, fuel requirements, alternatives available if
the planned flight cannot be completed, and any known traffic delays of which the
pilot in command has been advised by ATC.
242. Pilots and/or flight crew members involved in near midair collision (NMAC)
occurrences are urged to report each incident immediately
by cell phone to the nearest Flight Standards District Office, as this is an
A.
emergency.
The primary purpose of the NMAC Reporting Program is to provide information for
use in enhancing the safety and efficiency of the National Airspace System. Pilots
and/or flight crew members involved in NMAC occurrences are urged to report
each incident immediately by radio or telephone to the nearest FAA ATC facility or
FSS.
243. An ATC transponder is not to be used unless it has been tested, inspected, and
found to comply with regulations within the preceding
A. 30 days.
B. 12 calendar months.
C. 24 calendar months.
No person may use an ATC transponder unless, within the preceding 24 calendar
months, that ATC transponder has been tested, inspected, and found to comply
with regulations.
244. If a pilot does not meet the recency of experience requirements for night flight and
official sunset is 1900 CST, the latest time passengers should be carried is
A. 1959 CST.
B. 1900 CST.
https://online.prepware.com/quiz_review_graded?quiz_id=6393576 6/3/2024, 5 53 pm
Page 79 of 218
:
C. 1800 CST.
No person may act as PIC of an aircraft carrying passengers during the period
beginning 1 hour after sunset and ending 1 hour before sunrise, unless within the
preceding 90 days that person has made at least three takeoffs and three landings
to a full stop during the period beginning 1 hour after sunset and ending 1 hour
before sunrise.
245. What type of ADS-B equipment is required for aircraft operating in class A
airspace?
A. 1090-ES.
For flights at and above 18,000 feet MSL in Class A airspace, the aircraft must be
equipped with 1090-ES and meet the performance requirements outlined in 14
CFR 91.227. For flights below 18,000 feet MSL, either 1090-ES or UAT equipment
can be used.
246. In the contiguous U.S., excluding the airspace at and below 2,500 feet AGL, an
operable coded transponder equipped with Mode C capability is required in all
airspace above
247. A new maintenance record being used for an aircraft engine rebuilt by the
manufacturer must include previous
Each manufacturer or agency that grants zero time to an engine rebuilt by it shall
enter, in the new record, each change made as required by Airworthiness
Directives.
https://online.prepware.com/quiz_review_graded?quiz_id=6393576 6/3/2024, 5 53 pm
Page 80 of 218
:
Answer (A) is incorrect because a rebuilt engine is considered to start with zero
time. Answer (B) is incorrect because a record of previous maintenance is not
required for a rebuilt engine.
248. How many days after an accident is a report required to be filed with the nearest
NTSB field office?
A. 2.
B. 7.
C. 10.
The operator of an aircraft shall file a report within 10 days after an accident.
Answer (A) is incorrect because 2 days is not a reporting requirement in NTSB Part
830. Answer (B) is incorrect because 7 days is the limitation with respect to an
overdue aircraft that is missing.
249. To act as pilot in command of an airplane towing a glider, a pilot must have
accomplished, within the preceding 24 months, at least
A. three actual glider tows under the supervision of a qualified tow pilot.
B. three actual or simulated glider tows while accompanied by a qualified tow pilot.
To act as PIC, the pilot must make at least three actual or simulated glider tows
while accompanied by a qualified pilot or make at least three flights as pilot-in-
command of a glider towed by an aircraft.
250. Which airborne incident would require that the nearest NTSB field office be
notified immediately?
Immediate notification is required by the operator of any civil aircraft when a flight
control system malfunction or failure occurs.
Answer (A) is incorrect because neither a cargo compartment door malfunction nor
a cabin door opening would require immediate notification to the NTSB, unless
substantial damage affecting the structural integrity of the aircraft resulted. Answer
(B) is incorrect because neither a cargo compartment door malfunction nor a cabin
door opening would require immediate notification to the NTSB, unless substantial
https://online.prepware.com/quiz_review_graded?quiz_id=6393576 6/3/2024, 5 53 pm
Page 81 of 218
:
damage affecting the structural integrity of the aircraft resulted.
Operate, with respect to aircraft, means use, cause to use, or authorize to use
aircraft.
Answer (A) is incorrect because the pilot-in-command may not necessarily be the
operator of the aircraft. Answer (B) is incorrect because the sole manipulator may
not necessarily be the operator of the aircraft.
252. Which is true with respect to operating limitations of a 'primary' category airplane?
A 'primary' category airplane is limited to a specified operating radius from its
A.
home base.
No person may operate a 'primary' category airplane carrying passengers or
B.
property for compensation or hire.
253. When should notification of an aircraft accident be made to the NTSB if there was
substantial damage and no injuries?
A. Immediately.
B. Within 10 days.
C. Within 30 days.
The operator of an aircraft shall immediately, and by the most expeditious means
available, notify the nearest NTSB field office when an aircraft accident occurs. An
aircraft accident is an occurrence associated with the operation of an aircraft
which takes place between the time any person boards the aircraft with the
intention of flight and all such persons have disembarked, and in which any
person suffers death or serious injury, or in which the aircraft receives substantial
damage which adversely affects the structural strength, performance, or flight
characteristics of the aircraft.
https://online.prepware.com/quiz_review_graded?quiz_id=6393576 6/3/2024, 5 53 pm
Page 82 of 218
:
Answer (B) is incorrect because 10 days is the time specified to file a detailed
aircraft accident report with the NTSB. Answer (C) is incorrect because 30 days is
not a deadline specified in NTSB Part 830.
B. Airplane; the airplane pilot should alter course to the left to pass.
255. If not equipped with required position lights, an aircraft must terminate flight
A. at sunset.
No person may, during the period from sunset to sunrise (or, in Alaska, during the
period a prominent unlighted object cannot be seen from a distance of 3 SM or
the sun is more than 6° below the horizon) operate an aircraft unless it has lighted
position lights.
256. If the minimum safe speed for any particular operation is greater than the
maximum speed prescribed in 14 CFR Part 91, the
operator must have a Memorandum of Agreement (MOA) with the controlling
A.
agency.
If the minimum safe airspeed for any particular operation is greater than the
maximum speed prescribed in Part 91, the aircraft may be operated at that
minimum speed.
https://online.prepware.com/quiz_review_graded?quiz_id=6393576 6/3/2024, 5 53 pm
Page 83 of 218
:
A. FAA-approved aircraft flight manual.
aircraft flight manual, approved manual material, markings and placards, or any
C.
combination thereof.
As required by 14 CFR 21.5, no person may operate a U.S.-registered civil unless
there is a current approved flight manual, approved manual material, markings and
placards, or any combination thereof available in the aircraft.
258. NTSB Part 830 requires an immediate notification as a result of which incident?
Any required flight crewmember being unable to perform flight duties because of
C.
illness.
The operator of an aircraft shall immediately, and by the most expeditious means
available, notify the nearest NTSB field office of the inability of any required flight
crew member to perform normal flight duties as a result of injury or illness.
Answer (A) is incorrect because these are not considered substantial damage
requiring immediate notification. Answer (B) is incorrect because these are not
considered substantial damage requiring immediate notification.
259. Unless otherwise authorized or required by ATC, the maximum indicated airspeed
permitted when at or below 2,500 feet AGL within 4 NM of the primary airport
within Class C or D airspace is
A. 180 knots.
B. 200 knots.
C. 230 knots.
260. Under what condition could an aircraft's engine logbook show no previous
operating history?
https://online.prepware.com/quiz_review_graded?quiz_id=6393576 6/3/2024, 5 53 pm
Page 84 of 218
:
C. After the aircraft's engine has been rebuilt by the manufacturer.
The owner or operator may use a new maintenance record without previous
operating history for an aircraft engine rebuilt by the manufacturer or by an agency
approved by the manufacturer.
261. The holder of a Commercial Pilot Certificate while exercising commercial pilot
privileges is restricted from operating under BasicMed
A. at anytime.
A Third Class Medical Certificate or compliance with BasicMed does not authorize
the PIC to exercise commercial pilot privileges. To operate as a commercial pilot
you are required to hold either a First or Second Class Medical Certificate.
262. Except when necessary for takeoff or landing or unless otherwise authorized by
the Administrator, the minimum altitude for IFR flight is
B. 3,000 feet over designated mountainous terrain; 2,000 feet over terrain elsewhere.
2,000 feet above the highest obstacle over designated mountainous terrain; 1,000
C.
feet above the highest obstacle over terrain elsewhere.
At a minimum, IFR flight must remain at 2,000 feet AGL in mountainous areas and
1,000 feet AGL elsewhere.
263. A pilot in command (PIC) of a civil aircraft may not allow any object to be dropped
from that aircraft in flight
B. unless the PIC has permission to drop any object over private property.
264. An aircraft carrying passengers for hire has been on a schedule of inspection every
https://online.prepware.com/quiz_review_graded?quiz_id=6393576 6/3/2024, 5 53 pm
Page 85 of 218
:
100 hours of time in service. Under which condition, if any, may that aircraft be
operated beyond 100 hours without a new inspection?
The aircraft may be flown for any flight as long as the time in service has not
A.
exceeded 110 hours.
The aircraft may be dispatched for a flight of any duration as long as 100 hours
B.
has not been exceeded at the time it departs.
The 100-hour limitation may be exceeded by not more than 10 hours if necessary
C.
to reach a place at which the inspection can be done.
The 100-hour limitation may be exceeded by not more than 10 hours if necessary
to reach a place the inspection can be done. The excess time, however, is
included in computing the next 100 hours of time in service.
Answer (A) is incorrect because the 10-hour grace period only applies to ferrying
an aircraft to a maintenance facility, for the 100-hour inspection. Answer (B) is
incorrect because the 10-hour grace period only applies to ferrying an aircraft to a
maintenance facility, for the 100-hour inspection.
265. As a commercial pilot, you decide to start a small business flying non-stop tours
to look at Christmas lights during the holiday season. What authorizations, if any,
are required to conduct Christmas light tours?
No authorizations or approvals are required if you hold the appropriate category
A.
and class rating for the aircraft that will be flown.
You must apply for and receive a Letter of Authorization from a Flight Standards
B.
District Office.
You must apply to the FAA to receive an exemption to carry passengers at night
C.
within a 50 mile radius of your departure airport.
A Letter of Authorization (LOA) must be obtained from your local Flight Standards
District Office (FSDO) prior to conducting commercial air tours. These regulations
can be found under 14 CFR 91.146 and 91.147.
https://online.prepware.com/quiz_review_graded?quiz_id=6393576 6/3/2024, 5 53 pm
Page 86 of 218
:
No person may operate a restricted category civil aircraft carrying persons or
property for compensation or hire.
C. ADS-B.
268. To serve as pilot in command of an airplane that is certified for more than one pilot
crewmember, and operated under Part 91, a person must
B. receive and log ground and flight training from an authorized flight instructor.
269. What regulations are in the terms and conditions of a Standard Airworthiness
Certificate?
https://online.prepware.com/quiz_review_graded?quiz_id=6393576 6/3/2024, 5 53 pm
Page 87 of 218
:
270. While taxiing for takeoff, a small fire burned the insulation from a transceiver wire.
What action would be required to comply with NTSB Part 830?
A report must be filed with the avionics inspector at the nearest FAA field office
B.
within 48 hours.
An immediate notification must be filed by the operator of the aircraft with the
C.
nearest NTSB field office.
The operator of an aircraft shall immediately and by the most expeditious means
available notify the nearest NTSB field office of any inflight fire. This rule specifies
inflight fires only.
The pilot of the multiengine airplane should give way; the single-engine airplane is
A.
to its right.
The pilot of the single-engine airplane should give way; the other airplane is to the
B.
left.
When aircraft of the same category are converging at approximately the same
altitude (except head-on, or nearly so), the aircraft to the other's right has the
right-of-way.
C. the same category, class, and type of aircraft (if a type rating is required).
https://online.prepware.com/quiz_review_graded?quiz_id=6393576 6/3/2024, 5 53 pm
Page 88 of 218
:
273. A standard airworthiness certificate remains in effect as long as the aircraft
receives
B. an annual inspection.
274. Which is true with respect to formation flights? Formation flights are
authorized when carrying passengers for hire with prior arrangement with the pilot
A.
in command of each aircraft in the formation.
No person may operate an aircraft carrying passengers for hire in formation flight.
Answer (A) is incorrect because when carrying passengers for hire, formation
flights are prohibited. Answer (B) is incorrect because formation flights are
authorized regardless of visibility.
275. Operating regulations for U.S.-registered civil airplanes require that during
movement on the surface, takeoffs, and landings, a seat belt and shoulder harness
(if installed) must be properly secured about each
B. person on board.
https://online.prepware.com/quiz_review_graded?quiz_id=6393576 6/3/2024, 5 53 pm
Page 89 of 218
:
276. A pilot convicted of operating a motor vehicle while either intoxicated by, impaired
by, or under the influence of alcohol or a drug is required to provide a
written report to the FAA Civil Aeromedical Institute (CAMI) within 60 days after the
A.
motor vehicle action.
written report to the FAA Civil Aviation Security Division (AMC-700) not later than
B.
60 days after the conviction.
notification of the conviction to an FAA Aviation Medical Examiner (AME) not later
C.
than 60 days after the motor vehicle action.
Each person holding a pilot certificate shall provide a written report of each motor
vehicle action to the FAA, Civil Aviation Security Division (AMC-700), not later than
60 days after the motor vehicle action.
277. An early part of the Aeronautical Decision Making (ADM) process involves
Each ADM student should take the Self-Assessment Hazardous Attitude Inventory
Test in order to gain a realistic perspective on his/her attitudes toward flying.
278. Hazardous attitudes which contribute to poor pilot judgment can be effectively
counteracted by
279. The Aeronautical Decision Making (ADM) process identifies the steps involved in
good decision making. One of these steps includes a pilot
https://online.prepware.com/quiz_review_graded?quiz_id=6393576 6/3/2024, 5 53 pm
Page 90 of 218
:
Steps for good decision making are: identifying personal attitudes hazardous to
safe flight, learning behavior modification techniques, learning how to recognize
and cope with stress, developing risk assessment skills, using all resources in a
multicrew situation, and evaluating the effectiveness of one's ADM skills.
280. To operate an airplane under SPECIAL VFR (SVFR) within Class D airspace at
night, which is required?
The pilot must hold an instrument rating, but the airplane need not be equipped
A. for instrument flight, as long as the weather will remain at or above SVFR
minimums.
The pilot must hold an instrument rating and the airplane must be equipped for
C.
instrument flight.
No person may operate an airplane in Class D airspace under Special VFR at night
unless that person is instrument rated, and the airplane is equipped for instrument
flight.
Answer (A) is incorrect because the airplane must be equipped for instrument
flight. Answer (B) is incorrect because there is no such designation as 'night SVFR
area.'
282. When turning onto a taxiway from another taxiway, what is the purpose of the
taxiway directional sign?
https://online.prepware.com/quiz_review_graded?quiz_id=6393576 6/3/2024, 5 53 pm
Page 91 of 218
:
Answer (A) is incorrect because this is the purpose of the runway location sign.
Answer (B) is incorrect because this is the purpose of the destination sign.
283. (Refer to Figure 54, point 6.) The Class C airspace at Metropolitan Oakland
International (OAK) which extends from the surface upward has a ceiling of
The letter 'T' denotes the ceiling of the Class C airspace which extends up to, but
does not include the floor of the overlying Class B airspace. The Class C airspace
normally extends upward to 4,000 feet AGL. However, in this case the Class C
airspace extends upward to the base of the Class B airspace. The overlying Class
B airspace has bases of 2,100 feet MSL, and 3,000 feet MSL.
Answer (B) is incorrect because 10,000 feet is the ceiling of the Class B airspace
over OAK. Answer (C) is incorrect because the Class C airspace ceiling on the
west side of OAK is 2,100 feet MSL, and the ceiling on the east side is 3,000 feet
MSL.
284. (Refer to Figure 51.) Which symbol does not directly address runway incursion
with other aircraft?
A. Illustration D.
B. Illustration G.
C. Illustration H.
Illustration D prohibits an aircraft from entering an area. This sign would typically
be located on one-way taxiways or a vehicle roadway. Thus, this sign does not
directly address runway incursions with other aircraft.
Answer (B) is incorrect because Illustration G is used to indicate when you are
clear of the runway. Answer (C) is incorrect because Illustration H is used to
indicate when you are clear of the ILS critical area.
285. You attended a party last night and consumed several glasses of wine. You are
planning to fly your aircraft home and have been careful to make sure 8 hours
have passed since your last alcoholic drink. You can make the flight now only if
you are not under the influence of alcohol and your blood alcohol level is
A. below .04%.
https://online.prepware.com/quiz_review_graded?quiz_id=6393576 6/3/2024, 5 53 pm
Page 92 of 218
:
B. below .08%.
C. 0.0%.
286. Excluding Hawaii, the vertical limits of the Federal Low Altitude airways extend
from
A. 700 feet AGL up to, but not including, 14,500 feet MSL.
B. 1,200 feet AGL up to, but not including, 18,000 feet MSL.
C. 1,200 feet AGL up to, but not including, 14,500 feet MSL.
Each Federal airway includes that airspace extending upward from 1,200 feet
above the surface up to but not including 18,000 feet MSL. Federal airways for
Hawaii have no upper limits.
287. (Refer to Figure 53, area 4.) You plan to depart on a day VFR flight from the
Firebaugh (F34) airport. What is the floor of controlled airspace above this airport?
Firebaugh (F34) is inside the magenta shading, which indicates the Class E
airspace begins at 700 feet AGL.
288. When planning a night cross-country flight, a pilot should check for the availability
and status of
289. What are some of the hazardous attitudes dealt with in Aeronautical Decision
Making (ADM)?
https://online.prepware.com/quiz_review_graded?quiz_id=6393576 6/3/2024, 5 53 pm
Page 93 of 218
:
Antiauthority (don't tell me), impulsivity (do something quickly without thinking),
A.
macho (I can do it).
ADM addresses the following five hazardous attitudes: antiauthority (don't tell
me!), impulsivity (do something quickly!), invulnerability (it won't happen to me),
macho (I can do it), resignation (what's the use?).
290. (Refer to Figure 61.) Ground control has instructed you to taxi Alfa to Foxtrot to the
active runway. According to the sign in the figure, which direction would you turn
at this intersection to comply with ATC?
A. No turn is required.
Orientation of signs is from left to right. Left turn signs are on the left of the
location sign and right turn signs are on the right side of the location sign. Figure
61 indicates taxiway Foxtrot will be a left turn from Alfa.
291. The basic drive for a pilot to demonstrate the 'right stuff' can have an adverse
effect on safety, by
generating tendencies that lead to practices that are dangerous, often illegal, and
B.
may lead to a mishap.
https://online.prepware.com/quiz_review_graded?quiz_id=6393576 6/3/2024, 5 53 pm
Page 94 of 218
:
When approaching to land at an airport without an operating control tower in a
Class G airspace area each pilot of an airplane must make all turns of that airplane
to the left unless the airport displays approved light signals or visual markings
indicating that turns should be made to the right, in which case the pilot must
make all turns to the right.
293. (Refer to Figure 53.) What is indicated by the star next to the "L" in the airport
information box for the MADERA (MAE) airport north of area 2?
The "L" with an asterisk indicates lighting limitations exist. Pilots should refer to
the Chart Supplement U.S. for this airport for details on these limitations.
294. (Refer to Figure 53, point 1.) This thin black shaded line is most likely
A. an arrival route.
The thin black shaded line is most likely a Military Training Route (MTR). MTRs are
normally labeled on sectional charts with either IR (IFR operations) or VR (VFR
operations), followed by either three or four numbers.
Answer (A) is incorrect because arrival routes are found on IFR charts. Answer (C)
is incorrect because state boundaries are indicated by a thin black broken line.
295. (Refer to Figure 52, area 8). The traffic pattern altitude at the Auburn Airport (AUN)
is 1,000 feet AGL. May you practice landings under VFR when the AWOS is
reporting a ground visibility of 2 miles?
Auburn is inside the magenta shading, which indicates that the Class E airspace
begins at 700 feet AGL. Therefore you are operating in a combination of Class E
and G airspace since the traffic pattern altitude is given to be 1,000 feet AGL. With
no other visibility information given, other than 2 SM reported by the AWOS you
will need to assume that visibility in the Class E airspace above AUB will be less
https://online.prepware.com/quiz_review_graded?quiz_id=6393576 6/3/2024, 5 53 pm
Page 95 of 218
:
than the prescribed 3 SM minimum for VFR flight.
The black and yellow diagonal striped sign is a taxiway ending marker, which
indicates the taxiway does not continue.
297. To rid itself of all the alcohol contained in one mixed drink, the human body
requires about
A. 1 hour.
B. 2 hours.
C. 3 hours.
As little as one ounce of liquor, one bottle of beer or four ounces of wine can
impair flying skills, with the alcohol consumed in these drinks being detectable in
the breath and blood for at least 3 hours.
298. (Refer to Figure 53.) You are planning a VFR westbound flight departing the
FRESNO CHANDLER EXECUTIVE (FCH) airport and you will be passing through
the active Lemoore C and A MOAs. What action should you take?
B. Avoid the MOA, VFR, and IFR flights are prohibited during day light hours.
C. Contact the aircraft operating in the MOA on the Guard frequency of 121.5.
Pilots operating under VFR should exercise extreme caution while flying within an
MOA when military activity is being conducted. The activity status (active/inactive)
of MOAs may change frequently. Therefore, pilots should contact any FSS within
100 miles of the area to obtain accurate real-time information concerning the MOA
hours of operation. Prior to entering an active MOA, pilots should contact the
controlling agency for traffic advisories.
https://online.prepware.com/quiz_review_graded?quiz_id=6393576 6/3/2024, 5 53 pm
Page 96 of 218
:
C. runways that intersect other taxiways.
300. A pilot and friends are going to fly to an out-of-town football game. When the
passengers arrive, the pilot determines that they will be over the maximum gross
weight for takeoff with the existing fuel load. Which of the following alternatives
best illustrates the RESIGNATION reaction?
The resignation attitude is found in pilots who think, 'what's the use?' They do not
see themselves as being able to make a great deal of difference in what happens
to them. When things go well, the pilot is apt to think that's good luck. When
things go badly, the pilot may feel someone is out to get them, or attribute it to
bad luck. The pilot will leave the action to others, for better or worse. Sometimes,
such pilots will even go along with unreasonable requests just to be a 'nice guy.'
301. When operating an airplane for the purpose of landing or takeoff within Class D
airspace under special VFR, what minimum distance from clouds and what
visibility are required?
A. Remain clear of clouds, and the ground visibility must be at least 1 SM.
B. 500 feet beneath clouds, and the ground visibility must be at least 1 SM.
C. Remain clear of clouds, and the flight visibility must be at least 1 NM.
No person may operate an airplane within Class D airspace under Special VFR
unless they remain clear of clouds and the ground visibility must be at least 1 SM.
Answer (B) is incorrect because the cloud clearance for Special VFR is clear of
clouds. Answer (C) is incorrect because if ground visibility is not reported, flight
visibility during landing or takeoff must be at least 1 SM.
302. For night flying operations, the best night vision is achieved when the
https://online.prepware.com/quiz_review_graded?quiz_id=6393576 6/3/2024, 5 53 pm
Page 97 of 218
:
cones in the eyes have become adjusted to the darkness in approximately 5
C.
minutes.
When entering a dark room, it is difficult to see anything until the eyes become
adjusted to the darkness. After approximately 5 to 10 minutes, the cones become
adjusted to the dim light and the eyes become 100 times more sensitive to the
light than they were before the dark room was entered. Much more time, about 30
minutes, is needed for the rods to become adjusted to darkness; but when they
do adjust, they are about 100,000 times more sensitive to light than they were in
the lighted area.
303. When a pilot recognizes a hazardous thought, he or she then should correct it by
applying the corresponding antidote. Which of the following is the antidote for
ANTIAUTHORITY hazardous attitude?
C. Don't tell me. Follow the rules. They are usually right.
The antiauthority (don't tell me!) attitude is found in people who do not like anyone
telling them what to do. The antidote for this attitude is: follow the rules, they are
usually right.
Answer (A) is incorrect because this is the antidote for the impulsivity attitude.
Answer (B) is incorrect because this is the antidote for the invulnerability attitude.
304. Which is true concerning the blue and magenta colors used to depict airports on
Sectional Aeronautical Charts?
Airports with control towers underlying Class A, B, and C airspace are shown in
A.
blue, Class D and E airspace are magenta.
Airports with control towers underlying Class C, D, and E airspace are shown in
B.
magenta.
Airports with control towers underlying Class B, C, D, and E airspace are shown in
C.
blue.
Airports having control towers (Class B, C, D or E airspace) are shown in blue. All
others are shown in magenta.
305. What is the first indication of flying into restricted visibility conditions when
operating VFR at night?
Ground lights begin to take on an appearance of being surrounded by a halo or
A.
glow.
https://online.prepware.com/quiz_review_graded?quiz_id=6393576 6/3/2024, 5 53 pm
Page 98 of 218
:
B. A gradual disappearance of lights on the ground.
Answer (A) is incorrect because ground (not cockpit) lights taking on the
appearance of a halo or glow indicate ground fog. Answer (C) is incorrect because
ground (not cockpit) lights taking on the appearance of a halo or glow indicate
ground fog.
B. a pilot should remain clear of an airport traffic pattern and continue circling.
307. (Refer to Figure 57.) You are directed to taxi to runway 10. You see this sign at a
taxiway intersection while taxiing. Which way should you proceed?
A. Left.
B. Right.
C. Straight ahead.
B. Develop this hazardous thought and follow through with modified action.
Label that thought as hazardous, then correct that thought by stating the
C.
corresponding learned antidote.
When a pilot recognizes a thought as hazardous, the pilot should label that
thought as hazardous, then correct that thought by stating the corresponding
antidote.
https://online.prepware.com/quiz_review_graded?quiz_id=6393576 6/3/2024, 5 53 pm
Page 99 of 218
:
309. (Refer to Figure 53, point 2.) The 1^6 indicates
The number 1^6 is a maximum elevation figure (MEF) which approximate and
round-up from the highest known feature within each quadrangle.
310. (Refer to Figure 54.) What is the ceiling of the Class D Airspace of the Byron (C83)
airport (area 2)?
A. 2,900 feet.
B. 7,600 feet.
311. (Refer to Figure 54, area 3.) What is the significance of R-2531? This is a restricted
area
312. What should you expect when you are told that LAHSO operations are in effect at
your destination airport?
A. All aircraft must operate on an IFR clearance due to high traffic volume.
That ATC will give you a clearance to land and hold short of a specified point on
B.
the runway.
https://online.prepware.com/quiz_review_graded?quiz_id=6393576 6/3/2024, 5 53 pm
Page 100 of 218
:
Land and hold short operations (LAHSO) is an air traffic control procedure to
balance the needs for increased airport capacity and system efficiency, consistent
with safety.
Answer (A) is incorrect because LAHSO clearances are also available to aircraft
operating VFR. Answer (C) is incorrect because LAHSO are found at high density
airports and are not due to IFR conditions.
313. At some airports located in Class D airspace where ground visibility is not
reported, takeoffs and landings under special VFR are
A. not authorized.
No person may operate an airplane within Class D airspace under Special VFR
unless they remain clear of clouds and the ground visibility must be at least 1 SM.
If ground visibility is not reported at that airport, flight visibility during landing or
takeoff must be at least 1 SM.
314. (Refer to Figure 64.) You see this sign when holding short of the runway. You
receive clearance to back taxi on the runway for a full-length runway 8 departure.
Which way should you turn when first taxiing on to the runway for takeoff?
A. Left.
B. Right.
The runway holding position sign contains the designation of the intersecting
runways. The runway numbers are arranged to correspond to the respective
runway threshold. For example, "26-8" indicates that the threshold for Runway 26
is to the left and the threshold for Runway 8 is to the right.
315. When a pilot recognizes a hazardous thought, he or she then should correct it by
stating the corresponding antidote. Which of the following is the antidote for
MACHO?
https://online.prepware.com/quiz_review_graded?quiz_id=6393576 6/3/2024, 5 53 pm
Page 101 of 218
:
B. Not so fast. Think first.
Macho (I can do it) pilots who are always trying to prove that they are better than
anyone else are thinking 'I can do it - I'll show them.' Pilots with this type of
attitude will try to prove themselves by taking risks in order to impress others.
While this pattern is thought to be a male characteristic, women are equally
susceptible. The antidote for this attitude is: taking chances is foolish.
Answer (A) is incorrect because this is the antidote for an anti-authority attitude.
Answer (B) is incorrect because this is the antidote for an impulsivity attitude.
316. What is the minimum visibility and ceiling required for a pilot to receive a "land and
hold short" clearance?
Pilots should only receive a LAHSO clearance when there is a minimum ceiling of
1,000 feet and 3 statute miles visibility. The intent of having 'basic' VFR weather
conditions is to allow pilots to maintain visual contact with other aircraft and
ground vehicle operations.
317. Which of the following is the first step of the Decide Model for effective risk
management and Aeronautical Decision Making?
A. Detect.
B. Identify.
C. Evaluate.
Hypoxia is the result of insufficient oxygen in the bloodstream going to the brain.
https://online.prepware.com/quiz_review_graded?quiz_id=6393576 6/3/2024, 5 53 pm
Page 102 of 218
:
Answer (A) is incorrect because hypoxia is a lack of oxygen in the bloodstream.
Answer (C) is incorrect because excessive carbon dioxide in the bloodstream is
not a symptom of hypoxia.
319. Which of the following is the final step of the Decide Model for effective risk
management and Aeronautical Decision Making?
A. Estimate.
B. Evaluate.
C. Eliminate.
320. (Refer to Figure 50.) The segmented circle indicates that the airport traffic is
The traffic pattern indicators on a segmented circle are used to indicate the
direction of turns. The traffic pattern indicators, shown as extensions from the
segmented circle, represent the base and final approach legs.
This sign prohibits an aircraft from entering an area. Typically, this sign would be
located on a taxiway intended to be used in only one direction or at the
intersection of vehicle roadways with runways, taxiways, or aprons where the
roadway may be mistaken as a taxiway or other aircraft movement surface.
Answer (A) is incorrect because this refers to Illustration G. Answer (C) is incorrect
because this refers to Illustration H.
322. The minimum flight visibility for VFR flight increases to 5 statute miles beginning at
an altitude of
https://online.prepware.com/quiz_review_graded?quiz_id=6393576 6/3/2024, 5 53 pm
Page 103 of 218
:
A. 14,500 feet MSL.
The only area requiring 5 SM visibility is 10,000 feet MSL and up (when above
1,200 feet AGL).
323. What is the first step in neutralizing a hazardous attitude in the ADM process?
324. You are preflight planning in the morning before an afternoon flight. Where would
you find information regarding an "Airport surface hot spot?"
325. Who has the final authority to accept or decline any "land and hold short" (LAHSO)
clearance?
C. Pilot-in-Command.
The pilot-in-command has the final authority to accept or decline any land and
hold short clearance. The safety and operation of the aircraft remain the
responsibility of the pilot.
https://online.prepware.com/quiz_review_graded?quiz_id=6393576 6/3/2024, 5 53 pm
Page 104 of 218
:
326. (Refer to Figure 51.) When taxiing up to an active runway, you are likely to be clear
of the ILS critical area when short of which sign?
A. Illustration H.
B. Illustration D.
C. Middle yellow.
Answer (B) is incorrect because this is the sign prohibiting aircraft entry into an
area. Answer (C) is incorrect because this is a runway boundary sign.
327. (Refer to Figure 52, point 8.) The floor of the Class E airspace over the town of
Auburn is
Auburn is inside the magenta shading, which indicates the Class E airspace
begins at 700 feet AGL.
Answer (A) is incorrect because the Class E airspace would begin at 1,200 feet
AGL (not MSL) if the airport were outside the magenta shaded area. Answer (C) is
incorrect because the Class E airspace would begin at 1,200 feet AGL (not MSL) if
the airport were outside the magenta shaded area.
Answer (A) is incorrect because body sensations must be ignored. Answer (B) is
incorrect because an increase in breathing rate could cause hyperventilation.
329. Which is true regarding the presence of alcohol within the human body?
https://online.prepware.com/quiz_review_graded?quiz_id=6393576 6/3/2024, 5 53 pm
Page 105 of 218
:
A. A small amount of alcohol increases vision acuity.
Answer (A) is incorrect because all mental and physical activities will be decreased
with even small amounts of alcohol in the bloodstream. Answer (B) is incorrect
because the adverse effects of alcohol are increased as altitude is increased.
332. When planning for an emergency landing at night, on of the primary considerations
should include
B. turning off all electrical switches to save battery power for the landing.
https://online.prepware.com/quiz_review_graded?quiz_id=6393576 6/3/2024, 5 53 pm
Page 106 of 218
:
If the engine fails at night, one of the primary considerations includes selecting an
emergency landing area close to public access if possible. This may facilitate
rescue or help, if needed.
333. Once a pilot-in-command accepts a 'land and hold short' (LAHSO) clearance, the
clearance must be adhered to, just as any other ATC clearance, unless
The common symptoms of hyperventilation are dizziness, nausea, hot and cold
sensations, tingling of the hands, legs and feet, sleepiness, and finally,
unconsciousness.
A. swallow or yawn.
https://online.prepware.com/quiz_review_graded?quiz_id=6393576 6/3/2024, 5 53 pm
Page 107 of 218
:
B. slow the breathing rate.
Answer (A) is incorrect because swallowing or yawning is used to relieve ear block.
Answer (C) is incorrect because the breathing rate should be slowed to increase
the amount of carbon dioxide in the blood.
A. Drowsiness.
C. A sense of well-being.
The common symptoms of hyperventilation are dizziness, nausea, hot and cold
sensations, tingling of the hands, legs, and feet, sleepiness, and finally
unconsciousness.
A. are in violation of the CFRs; you need 3 miles of visibility under VFR.
are required to descend to below 700 feet AGL to remain clear of Class E airspace
B.
and may continue for landing.
may descend to 800 feet AGL (Pattern Altitude) after entering Class E airspace
C.
and continue to the airport.
At 7 NM north of Madera, you are outside the magenta shading, which indicates
the floor of the Class E airspace is 1,200 feet AGL. At 1,000 feet, you are in Class
G airspace. During daylight hours, the minimum flight visibility for VFR flight is 1
SM. Inside the magenta shading, the floor of the Class E airspace drops to 700
https://online.prepware.com/quiz_review_graded?quiz_id=6393576 6/3/2024, 5 53 pm
Page 108 of 218
:
feet. Therefore, to remain VFR, you must remain in Class G airspace, which
requires you to descend below 700 feet before entering the Class E airspace to
continue for landing.
339. (Refer to Figure 52, point 4.) The terrain at the obstruction approximately 8 NM
east southeast of the Lincoln Airport is approximately how much higher than the
airport elevation?
A. 376 feet.
B. 827 feet.
C. 1,135 feet.
1,245 feet MSL obstruction height - 297 feet AGL = 948 feet MSL terrain height at
obstruction - 121 feet MSL airport elevation = 827 feet terrain height higher than
airport elevation.
340. (Refer to Figure 51.) The pilot generally calls ground control after landing when the
aircraft is completely clear of the runway. This is when the aircraft
After landing, the pilot generally calls ground control when the aircraft is
completely clear of the runway. This is when the aircraft is on the solid-line side of
Illustration G. The solid lines always indicate the side on which the aircraft is to
hold.
Answer (A) is incorrect because Illustration D prohibits aircraft entry into an area.
Answer (B) is incorrect because you are still on the runway if you are on the
dashed-line side of Illustration G.
341. What designated airspace associated with an airport becomes inactive when the
control tower at that airport is not in operation?
https://online.prepware.com/quiz_review_graded?quiz_id=6393576 6/3/2024, 5 53 pm
Page 109 of 218
:
C. Class B.
Class D airspace exists only when the control tower is operating. It reverts to
Class E when the tower closes if there is an instrument approach and a weather
observer.
Answer (A) is incorrect because Class D airspace will revert to Class E airspace
when the control tower closes. Answer (C) is incorrect because the primary airport
of Class B airspace will have a control tower that operates full-time.
342. When should pilots decline a "land and hold short" (LAHSO) clearance?
343. You would like to enter Class B airspace and contact the approach controller. The
controller responds to your initial radio call with "N125HF standby." May you enter
the Class B airspace?
You must remain outside Class B airspace until controller gives you a specific
A.
clearance.
B. You may continue into the Class B airspace and wait for further instructions.
You may continue into the Class B airspace without a specific clearance, if the
C.
aircraft is ADS-B equipped.
No. You are not to enter the Class B airspace until you have received authorization
from ATC. "Standby" simply means the controller has your request, it does not
give you permission to enter the airspace.
344. The Decide Model is comprised of a 6-step process to provide a pilot a logical
way of approaching Aeronautical Decision Making. These steps are:
The DECIDE Model, comprised of a six step process, is intended to provide the
pilot with a logical way of approaching decision making: detect, estimate, choose,
identify, do, and evaluate.
https://online.prepware.com/quiz_review_graded?quiz_id=6393576 6/3/2024, 5 53 pm
Page 110 of 218
:
345. VFR cruising altitudes are required to be maintained when flying
In level cruise at more than 3,000 feet AGL, magnetic course determines proper
altitude.
Answer (A) is incorrect because VFR cruising altitudes are based on magnetic
course and apply for flights above 3,000 feet AGL. Answer (C) is incorrect because
VFR cruising altitudes are based on magnetic course and apply for flights above
3,000 feet AGL.
346. When departing from a runway that is covered with snow or slush, what can a pilot
do to prevent damage to the landing gear due to the conditions?
A. Do not retract the landing gear immediately to allow the gear to air-dry.
B. Immediately retract the landing gear so it can be heated in the gear wells.
Fly at a speed above the green arc of the airspeed indicator can remove the snow
C.
and slush.
If departing from an airstrip with wet snow or slush on the takeoff surface, the gear
should not be retracted immediately so that any wet snow or slush is allowed to
air-dry.
347. What should you consider when planning to land at another airport?
https://online.prepware.com/quiz_review_graded?quiz_id=6393576 6/3/2024, 5 53 pm
Page 111 of 218
:
principles.
Pilot management of risk is improved with practice and consistent use of basic
and practical risk management tools.
349. When should pilots decline a "land and hold short" (LAHSO) clearance?
A. slowly sweep the field of vision from one side to the other at intervals.
use a series of short, regularly spaced eye movements that bring successive areas
C.
of the sky into the central visual field.
Because the eyes can focus on only a narrow viewing area, effective scanning is
accomplished with a series of short, regularly spaced eye movements that bring
successive areas of the sky into the central visual field.
351. While on an IFR flight, a pilot emerges from a cloud to find himself within 300 feet
of a helicopter. Which of the following alternatives best illustrates the 'MACHO'
reaction?
The Macho attitude is found in people who are always trying to prove they are
better than anyone else. They are always thinking 'I can do it, I'll show them.'
Pilots with this type of attitude will try to prove themselves by taking risks in order
https://online.prepware.com/quiz_review_graded?quiz_id=6393576 6/3/2024, 5 53 pm
Page 112 of 218
:
to impress others.
B. The pilot must receive an ATC clearance before operating an aircraft in that area.
No person may operate an aircraft within a Class B airspace area unless the
operator receives an ATC clearance from the ATC facility having jurisdiction for
that area before operating an aircraft in that area.
353. Examples of classic behavioral traps that experienced pilots may fall into are:
trying to
354. When a dashed blue circle surrounds an airport on a sectional aeronautical chart,
it will depict the boundary of
B. Class B airspace
https://online.prepware.com/quiz_review_graded?quiz_id=6393576 6/3/2024, 5 53 pm
Page 113 of 218
:
C. Class D airspace.
Class D airspace areas are depicted on Sectional and Terminal charts with blue
segmented lines.
355. Which is true regarding flight operations to or from a satellite airport, without an
operating control tower, within the Class C airspace area?
Prior to entering that airspace, a pilot must establish and maintain communication
A.
with the ATC serving facility.
Prior to takeoff, a pilot must establish communication with the ATC controlling
C.
facility.
No person may take off or land an aircraft at a satellite airport within a Class C
airspace area except in compliance with FAA arrival and departure traffic patterns.
Each person must establish two-way radio communications with the ATC facility
providing air traffic services prior to entering that airspace and thereafter maintain
those communications while within that airspace.
356. How can you determine if another aircraft is on a collision course with your
aircraft?
B. The other aircraft will always appear to get larger and closer at a rapid rate.
There will be no apparent relative motion between your aircraft and the other
C.
aircraft.
https://online.prepware.com/quiz_review_graded?quiz_id=6393576 6/3/2024, 5 53 pm
Page 114 of 218
:
It is essential to remember that if another aircraft appears to have no relative
motion, it is likely to be on a collision course with you. If the other aircraft shows
no lateral or vertical motion, but increases in size, take evasive action.
357. When in the vicinity of a VOR which is being used for navigation on VFR flights, it
is important to
A. make 90° left and right turns to scan for other traffic.
exercise sustained vigilance to avoid aircraft that may be converging on the VOR
B.
from other directions.
pass the VOR on the right side of the radial to allow room for aircraft flying in the
C.
opposite direction on the same radial.
Pilots should exercise increased caution when entering high use airspace; this
includes the airspace around VORs.
Answer (A) is incorrect because 90° turns are not appropriate while en route.
Answer (C) is incorrect because you should try to maintain the center of the radial.
358. Most pilots have fallen prey to dangerous tendencies or behavior problems at
some time. Some of these dangerous tendencies or behavior patterns which must
be identified and eliminated include:
Performance deficiencies from human factors such as, fatigue, illness or emotional
B.
problems.
Peer pressure, get-there-itis, loss of positional or situation awareness, and
C.
operating without adequate fuel reserves.
There are a number of classic behavioral traps into which pilots have been known
to fall. These dangerous tendencies or behavior patterns, which must be identified
and eliminated, include: peer pressure, mind set, get-there-itis, duck-under
syndrome, scud running, continuing visual flight rules into instrument conditions,
getting behind the aircraft, loss of positional or situation awareness, operating
without adequate fuel reserves, descent below the minimum enroute altitude,
flying outside the envelope, neglect of flight planning, preflight inspections,
checklists, etc.
359. (Refer to Figure 52, area 6.) What is the purpose of the star that follows the CT-
120.65 in the information box for Sacramento Mather Airport (MHR)?
https://online.prepware.com/quiz_review_graded?quiz_id=6393576 6/3/2024, 5 53 pm
Page 115 of 218
:
C. There is a rotating beacon on the field.
The star symbol indicates that control tower operation is part time only.
there is a high volume of pilot training activities or an unusual type of aerial activity,
C.
neither of which is hazardous to aircraft.
Alert Areas inform pilots of airspace that may contain a high volume of pilot
training or an unusual type of aerial activity. While pilots should be particularly alert
in these areas, there are no restrictions on flying through them.
Answer (A) is incorrect because this describes a Restricted Area. Answer (B) is
incorrect because this describes a Prohibited Area.
361. (Refer to Figure 52, point 1.) The floor of the Class E airspace above Georgetown
Airport (Q61) is at
A. the surface.
Georgetown Airport is outside the magenta shaded area, which indicates the floor
of Class E airspace is at 1,200 feet AGL. The airport elevation is given in the
airport data as 2,623 feet MSL. Therefore, the Class E airspace above Georgetown
Airport is 3,823 feet MSL (2,623 + 1,200).
Answer (A) is incorrect because Class E airspace only begins at the surface when
surrounded by a magenta segmented circle. Answer (B) is incorrect because Class
E airspace begins at 700 feet AGL inside the magenta shaded areas.
362. What is the minimum visibility and ceiling required for a pilot to receive a "land and
hold short" clearance?
Pilots should only receive a LAHSO clearance when there is a minimum ceiling of
1,000 feet and 3 SM visibility. The intent of having 'basic' VFR weather conditions
https://online.prepware.com/quiz_review_graded?quiz_id=6393576 6/3/2024, 5 53 pm
Page 116 of 218
:
is to allow pilots to maintain visual contact with other aircraft and ground vehicle
operations.
363. After an ATC clearance has been obtained, a pilot may not deviate from that
clearance, unless the pilot
364. The thinner outer magenta circle depicted around Class B Airspace is
an area within which an appropriate transponder must be used from outside of the
B.
Class B Airspace from the surface to 10,000 feet MSL.
365. After experiencing a powerplant failure at night, one of the primary considerations
should include
If the engine fails at night, one of the primary considerations includes planning an
emergency approach and landing to an unlighted portion of the area.
366. While conducting an operational check of the cabin pressurization system, the
pilot discovers that the rate control feature is inoperative. He knows that he can
manually control the cabin pressure, so he elects to disregard the discrepancy.
https://online.prepware.com/quiz_review_graded?quiz_id=6393576 6/3/2024, 5 53 pm
Page 117 of 218
:
Which of the following alternatives best illustrates the INVULNERABILITY
reaction?
The invulnerability attitude is found in people who feel accidents happen to others,
but never to them. They know accidents can happen, and they know that anyone
can be affected, but they never really feel or believe that they will be personally
involved. Pilots who think this way are more likely to take chances and increase
risk.
367. (Refer to Figure 52, point 5.) The floor of the Class E airspace over University
Airport (0O5) is
A. the surface.
University Airport is within the magenta shading, which indicates the floor of the
Class E airspace begins at 700 feet AGL.
Answer (A) is incorrect because the Class E airspace would begin at the surface if
the airport were surrounded by a magenta segmented circle. Answer (C) is
incorrect because the Class E airspace would begin at 1,200 feet AGL if the
airport were outside the magenta shaded area.
Each person operating an aircraft in Class A airspace must conduct that operation
under instrument flight rules.
https://online.prepware.com/quiz_review_graded?quiz_id=6393576 6/3/2024, 5 53 pm
Page 118 of 218
:
369. (Refer to Figure 52, point 7.) The floor of Class E airspace over the town of
Woodland is
A. 700 feet AGL over part of the town and no floor over the remainder.
B. 1,200 feet AGL over part of the town and no floor over the remainder.
Woodland has magenta shading over part of the town. Inside this magenta
shading, Class E airspace begins at 700 feet AGL. Outside the magenta area,
Class E airspace begins at 1,200 feet AGL.
370. (Refer to Figure 58.) You have requested taxi instructions for takeoff using Runway
16. The controller issues the following taxi instructions: "N123, Taxi to Runway
16." Where are you required to stop in order to be in compliance with the
controller's instructions
A. 5 (Five).
B. 6 (Six).
C. 9 (Nine).
When ATC clears an aircraft to "taxi to" an assigned takeoff runway, the absence
of holding instructions does not authorize the aircraft to "cross" all runways which
the taxi route intersects except the assigned takeoff runway. A clearance must be
obtained prior to crossing any runway. It does not include authorization to "taxi
onto" or "cross" the assigned takeoff runway at any point. You should taxi and
hold short of Runway 16, which is position 5.
Answer (B) is incorrect because "taxi to" does not authorize the aircraft to "taxi
onto" the assigned takeoff runway. Answer (C) is incorrect because the airplane
should taxi the most direct route to the assigned runway unless instructed
otherwise; position 9 would not be encountered for the airplane at the west ramp
to taxi to Runway 16.
A. Insufficient oxygen.
As hyperventilation 'blows off' excessive carbon dioxide from the body, a pilot can
experience symptoms of lightheadedness, suffocation, drowsiness, tingling of the
extremities, and coolness and react to them with even greater hyperventilation.
https://online.prepware.com/quiz_review_graded?quiz_id=6393576 6/3/2024, 5 53 pm
Page 119 of 218
:
Answer (A) is incorrect because insufficient oxygen is a symptom of hypoxia.
Answer (B) is incorrect because excessive carbon monoxide will lead to carbon
monoxide poisoning.
372. When operating an aircraft in the vicinity of an airport with an operating control
tower, in Class E airspace, a pilot must establish communications prior to
373. Risk management, as part of the Aeronautical Decision Making (ADM) process,
relies on which features to reduce the risks associated with each flight?
The mental process of analyzing all information in a particular situation and making
A.
a timely decision on what action to take.
Risk management is the part of the decision making process which relies on
situational awareness, problem recognition, and good judgment to reduce risks
associated with each flight.
374. Your VFR flight will be conducted above 10,000 MSL in Class E airspace. What is
the minimum flight visibility?
A. 3 NM.
B. 5 SM.
C. 1 SM.
The only area requiring 5 SM visibility is 10,000 feet MSL and up (when above
1,200 feet AGL).
https://online.prepware.com/quiz_review_graded?quiz_id=6393576 6/3/2024, 5 53 pm
Page 120 of 218
:
A. Aircraft must be equipped with approved distance measuring equipment (DME).
376. Which is true regarding flight operations to or from a satellite airport, without an
operating control tower, within the Class C airspace area?
Prior to takeoff, a pilot must establish communication with the ATC controlling
A.
facility.
Aircraft must be equipped with an ATC transponder and altitude reporting
B.
equipment.
Prior to landing, a pilot must establish and maintain communication with an ATC
C.
facility.
Unless otherwise authorized or directed by ATC, no person may operate an aircraft
in Class C airspace unless that aircraft is equipped with an operable transponder
and altitude reporting equipment.
Answer (A) is incorrect because pilots must establish communication with the ATC
controlling facility as soon as practicable, which may not be prior to takeoff.
Answer (C) is incorrect because communications must be established prior to
entering Class C airspace, well before a "prior to landing" point.
377. When approaching to land at an airport with an ATC facility, in Class D airspace,
the pilot must establish communications prior to
https://online.prepware.com/quiz_review_graded?quiz_id=6393576 6/3/2024, 5 53 pm
Page 121 of 218
:
A. humidity decreases.
B. altitude increases.
Answer (A) is incorrect because the humidity level does not have a bearing on
carbon monoxide or oxygen levels. Answer (C) is incorrect because oxygen
demand does not change.
The pilot in command must hold at least a private pilot certificate with an
B.
instrument rating.
A student pilot may only operate an aircraft on a solo flight in Class B airspace if
the student pilot has received both ground and flight training from an authorized
instructor and has received a logbook endorsement. No person may operate an
aircraft in a Class B airspace area unless the aircraft is equipped with the
applicable operating transponder and automatic altitude reporting equipment.
Requests for ATC authorized deviations for operation of an aircraft that is not
equipped with a transponder must be made at least one hour before the proposed
operation.
380. (Refer to Figure 52, Area 2.) When departing the RIO LINDA (L36) airport to the
northwest at an altitude of 1,000 feet AGL, you
must make contact with MC CLELLAN (MCC) control tower as soon as practical
A.
after takeoff.
are not required to contact any ATC facilities if you do not enter the Class C
B.
Airspace.
must make contact with the SACRAMENTO INTL ( SMF) control tower
C.
immediately after takeoff.
Rio Linda (L36) is within Class E airspace at the surface and within Class C
airspace starting at 1600 feet MSL. Departing to the northwest, you will enter
https://online.prepware.com/quiz_review_graded?quiz_id=6393576 6/3/2024, 5 53 pm
Page 122 of 218
:
Class C airspace at the surface and must contact Sacramento Intl (SMF) control
tower immediately after takeoff prior to entering the Class C airspace.
381. (Refer to Figure 54, point 2.) After departing from Byron Airport (C83) with a
northeast wind, you discover you are approaching Livermore Class D airspace and
flight visibility is approximately 2-1/2 miles. You must
The magenta shading indicates Class E airspace begins at 700 feet. The VFR
minimum in controlled airspace below 10,000 feet is 3 SM. Therefore, with 2-1/2
miles visibility, you must stay below 700 feet to remain in Class G airspace.
Answer (B) is incorrect because Class E space begins at 1,200 feet when
surrounded by blue shading. Answer (C) is incorrect because 119.65 is ATIS for
Livermore, not ATCT.
382. (Refer to Figure 54, point 1.) What minimum altitude is required to avoid the
Livermore Airport (LVK) Class D airspace?
The Class D airspace at Livermore has a top of 2,900 feet MSL, indicated by the
[29] within the blue segmented circle. Therefore, the minimum altitude to fly over
and avoid the Class D airspace is 2,901 feet MSL.
Answer (A) is incorrect because 2,503 feet MSL would place you within the Class
D airspace. Answer (C) is incorrect because although 3,297 feet MSL would keep
you outside the Class D airspace, it is not the minimum altitude required to avoid
it.
383. When weather information indicates that abnormally high barometric pressure
exists, or will be above _____ inches of mercury, flight operations will not be
authorized contrary to the requirements published in NOTAMs.
A. 31.00.
B. 32.00.
https://online.prepware.com/quiz_review_graded?quiz_id=6393576 6/3/2024, 5 53 pm
Page 123 of 218
:
C. 30.50.
Special flight restrictions exist when any information indicates that barometric
pressure on the route of flight currently exceeds or will exceed 31 inches of
mercury, and no person may operate an aircraft or initiate a flight contrary to the
requirements established by the Administrator and published in a NOTAM.
384. To rid itself of all the alcohol contained in one beer, the human body requires about
A. 1 hour.
B. 3 hours.
C. 4 hours.
As little as one ounce of liquor, one bottle of beer or four ounces of wine can
impair flying skills, with the alcohol consumed in these drinks being detectable in
the breath and blood for at least 3 hours.
385. What is the minimum flight visibility and proximity to cloud requirements for VFR
flight, at 6,500 feet MSL, in Class C, D, and E airspace?
In Class C, D, or E airspace at 6,500 feet MSL, the VFR flight visibility requirement
is 3 SM. The distance from cloud requirement is 500 feet below, 1,000 feet above,
and 2,000 feet horizontal.
Answer (A) is incorrect because 1 SM visibility and clear of clouds is the VFR
weather minimum when at or below 1,200 feet AGL in Class G airspace during the
day. Answer (C) is incorrect because 5 SM visibility and cloud clearance of 1,000
feet above and below is the VFR weather minimum in Class E airspace at or above
10,000 feet MSL.
A LAHSO clearance, once accepted, must be adhered to, just as any other ATC
clearance, unless an amended clearance is obtained or an emergency occurs. A
LAHSO clearance does not preclude a rejected landing.
https://online.prepware.com/quiz_review_graded?quiz_id=6393576 6/3/2024, 5 53 pm
Page 124 of 218
:
387. The 'taxiway ending' marker
Taxiway ending markers are used to indicate the taxiway does not continue.
A. All pilots must contact the controlling agency to ensure aircraft separation.
389. The passengers for a charter flight have arrived almost an hour late for a flight that
requires a reservation. Which of the following alternatives best illustrates the
ANTIAUTHORITY reaction?
The antiauthority attitude is found in people who do not like anyone telling them
what to do. In a sense, they are saying no one can tell me what to do. They may
be resentful of having someone tell them what to do or may regard rules,
regulations, and procedures as silly or unnecessary.
390. (Refer to Figure 51.) While clearing an active runway you are most likely clear of
the ILS critical area when you pass which sign?
A. Illustration D.
B. Illustration G.
C. Illustration H.
https://online.prepware.com/quiz_review_graded?quiz_id=6393576 6/3/2024, 5 53 pm
Page 125 of 218
:
While clearing an active runway, you are most likely to be clear of the ILS critical
area when you pass the sign depicted in illustration H. This is the ILS critical area
boundary sign.
Answer (A) is incorrect because this symbol prohibits aircraft entry into an area.
Answer (B) is incorrect because Illustration G indicates you are most likely clear of
the runway.
391. Pilots are required to have the anti-collision light system operating
Pilots of aircraft equipped with rotating beacons are encouraged to turn them on
when intending to fly, as an alert to other aircraft and ground personnel.
Answer (A) is incorrect because lights are not needed if the pilot is simply sitting in
the cockpit, or for example, if a mechanic is working on the aircraft and does not
intend to taxi or fly. Answer (B) is incorrect because lights are not needed if the
pilot is simply sitting in the cockpit, or for example, if a mechanic is working on the
aircraft and does not intend to taxi or fly.
C. avoid situations that will improve their abilities to handle cockpit responsibilities.
Good cockpit management begins with good life stress management. Many of the
stress coping techniques practiced for life stress management are not usually
practical in flight. Rather, you must condition yourself to relax and think rationally
when stress appears.
393. If a military training route has flights operating at or below 1,500 feet AGL, it will be
designated by
Military Training Routes (MTRs) in which flights are conducted at or below 1,500
feet AGL are designated by the letters IR or VR and a four-digit number.
https://online.prepware.com/quiz_review_graded?quiz_id=6393576 6/3/2024, 5 53 pm
Page 126 of 218
:
394. Which is true regarding pilot certification requirements for operations in Class B
airspace?
The pilot in command must hold at least a private pilot certificate with an
A.
instrument rating.
No person may operate an aircraft within a Class B airspace area unless the pilot-
in-command holds at least a Private Pilot Certificate.
The 'Pvt' after the airport names indicates Van Vleck airport is a restricted or non-
public use airport.
Answer (A) is incorrect because the R in the circle indicates the airport is a
nonpublic-use airport. Answer (B) is incorrect because the blue box near Van
Vleck airport indicates an alert area.
397. With a blood alcohol level below .04 percent, a pilot cannot fly sooner than
https://online.prepware.com/quiz_review_graded?quiz_id=6393576 6/3/2024, 5 53 pm
Page 127 of 218
:
B. 12 hours after drinking alcohol.
398. The radius of the uncharted Outer Area of Class C airspace is normally
A. 20 NM.
B. 30 NM.
C. 40 NM.
The normal radius of the outer area will be 20 NM. This is the area where
separation is provided after two-way communication is established. It is only a
requirement to contact ATC before entering the 10 NM Class C airspace depicted
on the sectional chart.
399. The pilot and passengers are anxious to get to their destination for a business
presentation. Level IV thunderstorms are reported to be in a line across their
intended route of flight. Which of the following alternatives best illustrates the
IMPULSIVITY reaction?
A. They want to hurry and get going, before things get worse.
Impulsivity is the attitude of people who frequently feel the need to do something,
anything, immediately. They do not stop to think about what they are about to do,
they do not select the best alternative, and they do the first thing that comes to
mind.
400. (Refer to Figure 52, point 4.) The obstruction within 10 NM closest to Lincoln
Regional Airport (LHM) is how high above the ground?
A. 1,245 feet.
B. 662 feet.
C. 296 feet.
The obstruction south of the airport is 296 feet above the ground, which is the
number in parenthesis.
https://online.prepware.com/quiz_review_graded?quiz_id=6393576 6/3/2024, 5 53 pm
Page 128 of 218
:
Answer (A) is incorrect because 1,245 is the height above sea level of the
obstruction 8.5 NM east of the airport. Answer (B) is incorrect because 662 feet is
the height above ground of the obstructions 8 NM SW of the airport.
A. a rapid descent.
A rapid acceleration during takeoff can create the illusion of being in a nose-up
attitude. The disoriented pilot will push the aircraft into a nose-low, or dive
attitude. This is called "somatogravic illusion."
403. (Refer to Figure 51, A.) This taxiway sign would be expected
This question will likely include an onscreen graphic of a taxiway location sign with
a direction sign or runway holding position sign. This type of sign is used at a
taxiway intersection of runways.
404. When planning a night cross-country flight, a pilot should check for
https://online.prepware.com/quiz_review_graded?quiz_id=6393576 6/3/2024, 5 53 pm
Page 129 of 218
:
C. location of rotating light beacons.
Prior to a night flight, and particularly a cross-country night flight, pilots should
check the availability and status of lighting systems at the destination airport.
on the low-pressure side of a jetstream core where the speed at the core is
C.
stronger than 110 knots.
Jet streams stronger than 110 knots (at the core) are apt to have areas of
significant turbulence near them in the sloping tropopause above the core, in the
jet stream front below the core and on the low-pressure side of the core. In these
areas there are frequently strong wind shears.
Answer (A) is incorrect because 60 to 90 knots is common for the jet stream and if
turbulence were to be found it would be to the sides and bottom of the core.
Answer (B) is incorrect because these conditions do not exclusively create wind
shear.
406. Why does the wind have a tendency to flow parallel to the isobars above the
friction level?
B. Coriolis force acts perpendicular to a line connecting the highs and lows.
Friction of the air with the Earth deflects the air perpendicular to the pressure
C.
gradient.
The pressure gradient force drives the wind and is perpendicular to isobars. When
a pressure gradient force is first established, wind begins to blow from higher to
lower pressure directly across the isobars. However, the instant air begins moving,
Coriolis force deflects it to the right. Soon the wind is deflected a full 90° and is
parallel to the isobars or contours. At this time, Coriolis force exactly balances
pressure gradient force. With the forces in balance, wind will remain parallel to
isobars or contours.
https://online.prepware.com/quiz_review_graded?quiz_id=6393576 6/3/2024, 5 53 pm
Page 130 of 218
:
C. Decrease in water vapor.
A change in ambient temperature lapse rate of an air mass will determine its
stability. Surface heating or cooling aloft can make the air more unstable.
Answer (B) is incorrect because cooling from below increases stability. Answer (C)
is incorrect because a decrease in water vapor lowers the dewpoint of the air, but
does not affect stability.
408. What type weather can one expect from moist, unstable air, and very warm
surface temperatures?
Answer (A) is incorrect because fog and stratus clouds are characteristics of stable
air. Answer (B) is incorrect because continuous precipitation is characteristic of
stable air.
409. Advection fog has drifted over a coastal airport during the day. What may tend to
dissipate or lift this fog into low stratus clouds?
A. Nighttime cooling.
B. Surface radiation.
Answer (A) is incorrect because nighttime cooling and surface radiation form
radiation fog. Answer (B) is incorrect because nighttime cooling and surface
radiation form radiation fog.
A. wind shear.
https://online.prepware.com/quiz_review_graded?quiz_id=6393576 6/3/2024, 5 53 pm
Page 131 of 218
:
C. strong convective currents.
You can be relatively certain of a shear zone in the inversion if you know the wind
at 2,000 to 4,000 feet is 25 knots or more. Allow a margin of airspeed above
normal climb or approach speed to alleviate the danger of a stall in event of
turbulence or sudden change in wind velocity.
Answer (B) is incorrect because strong surface winds do not present as great a
danger as wind shear. Answer (C) is incorrect because a temperature inversion
does not generate strong convective currents.
411. You are avoiding a thunderstorm that is in your flightpath. You are over 20 miles
from the cell however, you are under the anvil of the cell. Is this a hazard?
412. Which is true regarding actual air temperature and dewpoint temperature spread?
The temperature spread
Temperature spread is the difference between air temperature and dew point. As
the spread decreases, relative humidity increases.
413. Which is true regarding a cold front occlusion? The air ahead of the warm front
C. has the same temperature as the air behind the overtaking cold front.
In the cold front occlusion, the coldest air is under the cold front. When it
overtakes the warm front, it lifts the warm front aloft and cold air replaces cool air
at the surface.
https://online.prepware.com/quiz_review_graded?quiz_id=6393576 6/3/2024, 5 53 pm
Page 132 of 218
:
414. One of the most dangerous features of mountain waves is the turbulent areas in
and
Rotor clouds appear to remain stationary, parallel the range, and stand a few miles
leeward of the mountains. Turbulence is most frequent and most severe in and
below the standing rotors just beneath the wave crests at or below mountain-top
levels.
415. GIVEN:
Winds at 3,000 feet AGL 30 kts
Surface winds Calm
While on approach for landing under clear skies with convective turbulence a few
hours after sunrise, one should
B. keep the approach airspeed at or slightly below normal to compensate for floating.
C. not alter the approach airspeed, these conditions are nearly ideal.
When taking off or landing in calm wind under clear skies within a few hours
before or after sunrise, be prepared for a temperature inversion near the ground.
You can be relatively certain of a shear zone in the inversion if you know the wind
at 2,000 to 4,000 feet is 25 knots or more. Increase airspeed slightly above normal
climb or approach speed to alleviate the danger of a stall in event of turbulence or
sudden change in wind velocity.
Answer (B) is incorrect because the hazard is wind shear. Answer (C) is incorrect
because these conditions are not ideal - wind shear may be present.
416. During an approach, the most important and most easily recognized means of
being alerted to possible wind shear is monitoring the
Since rate of descent on the glide slope is directly related to ground speed, a high
descent rate would indicate a strong tailwind. Conversely, a low descent rate
indicates a strong headwind. The power needed to hold the glide slope also will
be different from typical, no-shear conditions. Less power than normal will be
https://online.prepware.com/quiz_review_graded?quiz_id=6393576 6/3/2024, 5 53 pm
Page 133 of 218
:
needed to maintain the glide slope when a tailwind is present and more power is
needed for strong headwind.
Answer (A) is incorrect because trim adjustments are a function of power settings,
airspeeds, and aircraft configuration. Answer (B) is incorrect because heading
changes are due to the crosswind component.
Answer (A) is incorrect because squall lines can form in any area of unstable air,
but usually are found ahead of cold fronts. Answer (B) is incorrect because squall
lines usually form quickly.
Rain falling from warm air above through colder air below may freeze during its
descent, falling as ice pellets. This can happen any time a warmer layer of air
exists above a colder layer (i.e., a warm front or a cold front).
A. Base of the clouds near the surface, heavy rain, and hail.
https://online.prepware.com/quiz_review_graded?quiz_id=6393576 6/3/2024, 5 53 pm
Page 134 of 218
:
signifies an extremely turbulent zone.
Answer (B) is incorrect because if you have flown from an area of unfavorable
weather conditions, you are flying out of the low, which means you would have a
right crosswind. Answer (C) is incorrect because the wind can provide an
indication of pressure changes and weather.
The jet stream is considerably weaker in the middle latitudes during the summer
months, and is further north than in the winter.
https://online.prepware.com/quiz_review_graded?quiz_id=6393576 6/3/2024, 5 53 pm
Page 135 of 218
:
storm.
Air moving out of a high or ridge depletes the quantity of air. Highs and ridges,
therefore, are areas of descending air.
425. The jet stream and associated clear air turbulence can sometimes be visually
identified in flight by
Long streaks of cirrus clouds can sometimes help the pilot to visually identify the
jet stream and associated clear air turbulence (CAT).
Answer (A) is incorrect because dust or haze indicates there is not enough wind or
air movement to dissipate the particles. Answer (C) is incorrect because CAT is
caused by mixing different air temperatures at different pressure levels.
426. Which in-flight hazard is most commonly associated with warm fronts?
https://online.prepware.com/quiz_review_graded?quiz_id=6393576 6/3/2024, 5 53 pm
Page 136 of 218
:
A. Advection fog.
B. Radiation fog.
C. Precipitation-induced fog.
When relatively warm rain or drizzle falls through cool air, evaporation from the
precipitation saturates the cool air and forms fog. Precipitation-induced fog can
become quite dense and continue for an extended period of time. This fog may
cover large areas, completely suspending air operations. It is most commonly
associated with warm fronts, but can occur with slow moving cold fronts and with
stationary fronts.
Answer (A) is incorrect because advection fog forms from the movement of warm,
humid air over a cold water surface. Answer (B) is incorrect because radiation fog
forms from terrestrial cooling of the Earth's surface on clear, calm nights.
427. Which is true regarding the use of airborne weather-avoidance radar for the
recognition of certain weather conditions?
The avoidance of hail is assured when flying between and just clear of the most
B.
intense echoes.
The clear area between intense echoes indicates that visual sighting of storms can
C.
be maintained when flying between the echoes.
Weather radar detects only precipitation drops. It does not detect minute cloud
droplets. Therefore, the radar scope provides no assurance of avoiding instrument
weather in clouds and fog.
Answer (B) is incorrect because hail can be thrown several miles from the intense
echoes. Answer (C) is incorrect because clouds without precipitation may exist
between the echoes.
A. cumulus clouds.
B. cumulonimbus clouds.
C. stratocumulus clouds.
You should anticipate possible hail with any thunderstorm, especially beneath the
anvil of a large cumulonimbus.
https://online.prepware.com/quiz_review_graded?quiz_id=6393576 6/3/2024, 5 53 pm
Page 137 of 218
:
A. Stratiform clouds.
Answer (B) is incorrect because cumulus clouds are characteristic of unstable air.
Answer (C) is incorrect because a rapid temperature decrease with altitude
indicates a high lapse rate and is characteristic of unstable air.
Answer (A) is incorrect because if the air is stable, steady precipitation and
stratiform clouds will form. Answer (C) is incorrect because a lifting mechanism
must exist to form cumuliform clouds and showery rain.
Answer (A) is incorrect because condensation removes moisture from the air.
Answer (B) is incorrect because condensation removes moisture from the air.
B. there is a low-level temperature inversion with strong winds above the inversion.
https://online.prepware.com/quiz_review_graded?quiz_id=6393576 6/3/2024, 5 53 pm
Page 138 of 218
:
surface winds are above 15 knots and there is no change in wind direction and
C.
windspeed with height.
When taking off or landing in calm wind under clear skies within a few hours
before or after sunrise, be prepared for a temperature inversion near the ground.
You can be relatively certain of a shear zone in the inversion if you know the wind
at 2,000 to 4,000 feet is 25 knots or more. Allow a margin of airspeed above
normal climb or approach speed to alleviate the danger of a stall in event of
turbulence or sudden change in wind velocity.
Answer (A) is incorrect because light surface winds alone would not cause wind
shear. Answer (C) is incorrect because wind shear refers to an abrupt change in
wind speed and/or direction.
433. In what ways do advection fog, radiation fog, and steam fog differ in their
formation or location?
Radiation fog is restricted to land areas; advection fog is most common along
A.
coastal areas; steam fog forms over a water surface.
Advection fog deepens as windspeed increases up to 20 knots; steam fog
B. requires calm or very light wind; radiation fog forms when the ground or water
cools the air by radiation.
Steam fog forms from moist air moving over a colder surface; advection fog
C. requires cold air over a warmer surface; radiation fog is produced by radiational
cooling of the ground.
Radiation fog is restricted to land areas because water surfaces cool little from
nighttime radiation. Advection fog is most common along coastal areas but often
develops deep in continental areas. Steam fog, also known as sea smoke, forms
in the winter when cold, dry air passes from land areas over comparatively warm
ocean waters.
434. Frost covering the upper surface of an airplane wing usually will cause
C. drag factors so large that sufficient speed cannot be obtained for takeoff.
The frost on the wing causes airflow disturbances. This will cause airflow
separation (stall) at a lower angle of attack, resulting in a tendency to stall during
takeoff.
Answer (A) is incorrect because frost on the wing surface will usually cause the
airplane to stall at a lower angle of attack. Answer (C) is incorrect because the
https://online.prepware.com/quiz_review_graded?quiz_id=6393576 6/3/2024, 5 53 pm
Page 139 of 218
:
drag will usually not be enough to prevent the aircraft from obtaining takeoff
speed.
435. Which are characteristics of a cold air mass moving over a warm surface?
Cool air moving over a warm surface is heated from below, generating instability
and increasing the possibility of showers. Unstable air is characterized by
cumuliform clouds, turbulence, and good visibility.
436. During the winter months in the middle latitudes, the jet stream shifts toward the
In middle latitudes, the wind speed of the jet stream averages considerably higher
in the winter months as it shifts farther south.
A. Atmospheric pressure.
A change in ambient temperature lapse rate of an air mass will determine its
stability. Surface heating or cooling aloft can make air more unstable. On the other
hand, surface cooling or warming aloft often tips the balance toward greater
stability.
Answer (A) is incorrect because atmospheric pressure affects temperature and air
movements, but does not determine the stability of the atmosphere. Answer (C) is
incorrect because the dry adiabatic lapse rate is a constant rate.
https://online.prepware.com/quiz_review_graded?quiz_id=6393576 6/3/2024, 5 53 pm
Page 140 of 218
:
C. They offer the most intense weather hazards to aircraft.
Answer (A) is incorrect because squall lines usually form rapidly. Answer (B) is
incorrect because squall lines can form in any area of unstable air, but usually are
found ahead of cold fronts.
It can appear suddenly during day or night, and it is more persistent than radiation
C.
fog.
Advection fog is more persistent than radiation fog and can move in rapidly
regardless of the time of day or night.
Answer (A) is incorrect because advection fog can move in rapidly regardless of
the time of day or night and is persistent. Answer (B) is incorrect because this
describes radiation fog.
440. Ice pellets encountered during flight normally are evidence that
Rain falling from warm air above through colder air below may freeze during its
descent, falling as ice pellets. This can happen any time a warmer layer of air
exists above a colder layer (i.e., a warm front or a cold front).
Answer (A) is incorrect because after the warm front has passed there will no
longer be a layer of warm air above a layer of cold air, which is required for the
formation of ice pellets. Answer (C) is incorrect because ice pellets do not
necessarily come from thunderstorms, but from rain freezing at a higher altitude.
441. Convective currents are most active on warm summer afternoons when winds are
A. light.
B. moderate.
https://online.prepware.com/quiz_review_graded?quiz_id=6393576 6/3/2024, 5 53 pm
Page 141 of 218
:
C. strong.
Convective currents are most active on warm summer afternoons when winds are
light. Heated air at the surface creates a shallow, unstable layer and the warm air
is forced upward. Convection increases in strength and to greater heights as
surface heating increases.
Answer (B) is incorrect because moderate and strong winds disrupt the vertical
movement of convective currents. Answer (C) is incorrect because moderate and
strong winds disrupt the vertical movement of convective currents.
442. The conditions necessary for the formation of stratiform clouds are a lifting action
and
Stable, moist air and adiabatic cooling is necessary to form stratiform clouds.
A. source of lift.
When stable air is forced upward, the air tends to retain horizontal flow. Any
cloudiness is flat and stratified. When unstable air is forced upward, the
disturbance grows, and any resulting cloudiness shows extensive vertical
development.
Answer (A) is incorrect because the stability of the air determines the type of
clouds that form. Answer (C) is incorrect because the temperature of the air
determines the altitude of clouds that form.
444. If clouds form as a result of very stable, moist air being forced to ascend a
mountain slope, the clouds will be
https://online.prepware.com/quiz_review_graded?quiz_id=6393576 6/3/2024, 5 53 pm
Page 142 of 218
:
Stable air resists upward movement; therefore, stratified clouds are produced.
Answer (A) is incorrect because cirrus clouds are high and composed of ice
crystals. Answer (B) is incorrect because unstable air causes vertical development.
A. Cirrus clouds.
B. Nimbostratus clouds.
Billowy fair weather cumulus clouds, usually seen on sunny afternoons, are
signposts in the sky indicating convective turbulence. Vertical heights range from
the shallow fair weather cumulus to the giant thunderstorm cumulonimbus.
Answer (A) is incorrect because cirrus clouds are high clouds made of ice crystals,
and are not generated by any convective activity. Answer (B) is incorrect because
nimbostratus clouds are flat rain clouds, formed in stable air and do not produce
convective activity or turbulence.
446. Which type of jetstream can be expected to cause the greater turbulence?
Curving jet streams, especially those which curve around a deep pressure trough,
are more apt to have turbulent edges than straight jet streams.
Answer (A) is incorrect because a curving jet stream is stronger than a straight jet
stream. Answer (C) is incorrect because the jet stream is weaker in the summer.
A. Nighttime cooling.
Answer (A) is incorrect because nighttime cooling and dryness of the underlying
land mass lead to radiation fog. Answer (B) is incorrect because nighttime cooling
https://online.prepware.com/quiz_review_graded?quiz_id=6393576 6/3/2024, 5 53 pm
Page 143 of 218
:
and dryness of the underlying land mass lead to radiation fog.
448. There is a high pressure system that is located south of your planned route in the
Northern Hemisphere on a west to east cross-country flight. To take advantage of
favorable winds, you would plan your route
High pressure flows outward in a clockwise direction so pilot will gain tailwinds by
flying on the north side of the high pressure system.
449. When conditionally unstable air with high-moisture content and very warm surface
temperature is forecast, one can expect what type of weather?
C. Pressure differences.
451. The greatest threats to an aircraft operating in the vicinity of thunderstorms are:
https://online.prepware.com/quiz_review_graded?quiz_id=6393576 6/3/2024, 5 53 pm
Page 144 of 218
:
Hazardous turbulence is present in all thunderstorms; in a severe thunderstorm, it
can damage an airframe. Hail competes with turbulence as the greatest
thunderstorm hazard to aircraft.
452. You have delayed your flight to allow a fast moving cold front to clear your
destination airport before your arrival. What type of flying conditions would you
expect after the front has passed?
Thunderstorms and heavy rain are common although not always associated with
cold fronts.
A. a heat exchange.
C. a pressure differential.
The amount of solar energy received by any region varies with time of day, with
seasons, and with latitude. These differences in solar energy create temperature
variation. Temperatures also vary with differences in topographical surface and
with altitude. This temperature variation, or heat exchange, creates forces that
drive the atmosphere in its endless motion.
Answer (B) is incorrect because the movement of air and pressure differentials are
caused by heat exchanges. Answer (C) is incorrect because the movement of air
and pressure differentials are caused by heat exchanges.
Answer (A) is incorrect because poor visibility and smooth air are characteristic of
stable air. Answer (C) is incorrect because stratiform clouds and continuous
https://online.prepware.com/quiz_review_graded?quiz_id=6393576 6/3/2024, 5 53 pm
Page 145 of 218
:
precipitation are characteristic of stable air.
B. an air mass moving inland from the coastline during the winter.
C. a warm, moist air mass settling over a cool surface under no-wind conditions.
Advection fog forms when warm, moist air moves over colder ground or water.
The fog forms offshore and is then carried inland by the wind. It is most common
along coastal areas but often develops deep in continental areas.
Answer (A) is incorrect because this describes steam fog. Answer (C) is incorrect
because this describes radiation fog.
456. During an IFR cross-country flight you picked up rime icing which you estimate is
1/2" thick on the leading edge of the wings. You are now below the clouds at
2,000 feet AGL and are approaching your destination airport under VFR. Visibility
under the clouds is more than 10 miles, winds at the destination airport are 8
knots right down the runway, and the surface temperature is 3°C. You decide to
approach and land at your normal speed since the ice is not thick enough to have
B.
any noticeable effect.
fly your approach slower than normal to lessen the 'wind chill' effect and break up
C.
the ice.
Ice will accumulate unevenly on the airplane. It will add weight and drag, and
decrease thrust and lift. With ice accumulations, landing approaches should be
made with a minimum wing flap setting and with an added margin of airspeed.
Sudden and large configuration and airspeed changes should be avoided.
Answer (B) is incorrect because ice having a thickness similar to sandpaper on the
leading edge and upper surface of a wing can reduce wing lift by as much as 30
percent and increase drag by 40 percent. Answer (C) is incorrect because ice will
increase drag, requiring additional lift (airspeed); wind chill effect cannot be relied
upon to melt/remove the ice that has already accumulated; flying slower than
normal increases the possibility of a stall due to the decreased lift.
457. Ice pellets encountered during flight are normally evidence that
https://online.prepware.com/quiz_review_graded?quiz_id=6393576 6/3/2024, 5 53 pm
Page 146 of 218
:
C. freezing rain exists at higher altitudes.
Rain falling from warm air above through colder air below may freeze during its
decent, falling as ice pellets. This can happen any time a warmer layer of air exists
above a colder layer (i.e., a warm front or a cold front).
Often there is a strong wind just above the top of an inversion layer. Flying into or
out of this wind induces a shear situation. The most prominent meteorological
phenomena that cause significant low-level wind shear problems are
thunderstorms and certain frontal systems at or near the airport.
Answer (A) is incorrect because hazardous wind shear is more commonly found
near inversions and thunderstorms. Answer (B) is incorrect because strong wind
does not mean that there will always be wind shear; the wind must be in different
directions.
459. The low-level wind shear Alert System (LLWAS) provides wind data and software
process to detect the presence of a
change in wind direction and/or speed within a very short distance above the
B.
airport.
downward motion of the air associated with continuous winds blowing with an
C.
easterly component due to the rotation of the Earth.
Wind shear may be associated with either a wind shift or a wind speed gradient at
any level in the atmosphere. It may be associated with a low-level temperature
inversion, in a frontal zone, or clear air turbulence (CAT) at high levels associated
with a jet stream or strong circulation.
Answer (A) is incorrect because this describes a tornado. Answer (C) is incorrect
because LLWAS detects wind changes close to the airport.
A. nocturnal cooling.
B. adiabatic cooling.
https://online.prepware.com/quiz_review_graded?quiz_id=6393576 6/3/2024, 5 53 pm
Page 147 of 218
:
C. evaporation of precipitation.
When relatively warm rain or drizzle falls through cool air, evaporation from the
precipitation saturates the cool air and forms fog.
Answer (A) is incorrect because nocturnal cooling produces radiation fog. Answer
(B) is incorrect because adiabatic cooling produces upslope fog.
Clear air turbulence (CAT) is greatest near the wind speed maxima, usually on the
polar sides where there is a combination of strong wind shear, curvature in the
flow, and cold air advection associated with sharply curved contours of strong
lows, troughs and ridges aloft. A frequent location of CAT is in an upper trough on
the cold, or polar, side of the jet stream.
462. Which conditions are favorable for the formation of a surface based temperature
inversion?
C. Broad areas of cumulus clouds with smooth, level bases at the same altitude.
An inversion often develops near the ground on clear, cool nights when the wind is
light.
Answer (B) is incorrect because the air near the surface must be stable to permit
the cool ground to lower the temperature of the surrounding air. Answer (C) is
incorrect because cumulus clouds are well above the surface.
463. What feature is normally associated with the cumulus stage of a thunderstorm?
A. Roll cloud.
B. Continuous updraft.
Answer (A) is incorrect because the roll cloud and the beginning of rain at the
surface are features of the mature stage. Answer (C) is incorrect because the roll
https://online.prepware.com/quiz_review_graded?quiz_id=6393576 6/3/2024, 5 53 pm
Page 148 of 218
:
cloud and the beginning of rain at the surface are features of the mature stage.
464. Which situation would most likely result in freezing precipitation? Rain falling from
air which has a temperature of
Rain falling through colder air may become supercooled, freezing on impact as
freezing rain, or it may freeze during its descent, falling as ice pellets. Water can
freeze at 0°C or 32°F.
465. What minimum distance should exist between intense radar echoes before any
attempt is made to fly between these thunderstorms?
A. 20 miles.
B. 30 miles.
C. 40 miles.
A pilot should avoid flying between very intense echoes unless they are separated
by at least 40 miles.
466. Select the true statement pertaining to the life cycle of a thunderstorm.
The beginning of rain at the Earth's surface indicates the mature stage of the
B.
thunderstorm.
The beginning of rain at the Earth's surface indicates the dissipating stage of the
C.
thunderstorm.
The mature stage of a thunderstorm starts when precipitation begins to fall from
the cloud base. The downdrafts reach speeds that may exceed 2,500 fpm.
Meanwhile, updrafts reach a maximum with speeds possibly exceeding 6,000
fpm. Updrafts and downdrafts in close proximity create strong vertical shear and a
very turbulent environment.
Answer (A) is incorrect because updrafts do not continue through the dissipating
stage of a thunderstorm. Answer (C) is incorrect because this indicates the
beginning of the mature stage.
467. Density altitude is the vertical distance above mean sea level in the standard
https://online.prepware.com/quiz_review_graded?quiz_id=6393576 6/3/2024, 5 53 pm
Page 149 of 218
:
atmosphere at which
Density altitude is the vertical distance above sea level in the standard atmosphere
at which a given density is to be found. The density of air has significant effects on
the aircraft's performance.
At the surface when air converges into a low, it cannot go outward against the
pressure gradient, nor can it go downward into the ground. It must go upward.
Therefore, a low or trough is an area of rising air.
Answer (A) is incorrect because high-pressure air descends and low-pressure air
rises. Answer (C) is incorrect because high-pressure air descends and low-
pressure air rises.
The Coriolis force deflects air to the right in the Northern Hemisphere.
A. Mature.
B. Developing.
C. Dissipating.
Answer (A) is incorrect because the mature stage has both updrafts and
https://online.prepware.com/quiz_review_graded?quiz_id=6393576 6/3/2024, 5 53 pm
Page 150 of 218
:
downdrafts. Answer (B) is incorrect because the developing stage primarily has
updrafts.
A. 5 miles.
B. At least 25 miles.
C. At least 20 miles.
Virga are the streamers of precipitation trailing beneath clouds that evaporate
before reaching the ground.
Answer (B) is incorrect because virga is usually thin and wispy. Answer (C) is
incorrect because virga is a form of precipitation.
A. 4,400 feet.
B. 8,800 feet.
C. 17,600 feet.
https://online.prepware.com/quiz_review_graded?quiz_id=6393576 6/3/2024, 5 53 pm
Page 151 of 218
:
474. With regard to windflow patterns shown on surface analysis charts; when the
isobars are
A. close together, the pressure gradient force is slight and wind velocities are weaker.
not close together, the pressure gradient force is greater and wind velocities are
B.
stronger.
close together, the pressure gradient force is greater and wind velocities are
C.
stronger.
The closer the spacing of isobars, the stronger is the pressure gradient force. The
stronger the pressure gradient force, the stronger is the wind. Thus, closely
spaced isobars mean strong winds; widely spaced isobars mean lighter wind.
475. The conditions necessary for the formation of cumulonimbus clouds are a lifting
action and
For cumulonimbus clouds to form, the air must have sufficient water vapor, an
unstable lapse rate, and an initial upward boost (lifting) to start the storm process
in motion.
A. 20 miles.
B. 10 miles.
C. 5 miles.
If the use of airborne radar indicates extremely intense echoes, they should be
avoided by at least 20 miles.
477. The general circulation of air associated with a high-pressure area in the Northern
Hemisphere is
As the air tries to blow outward from the high pressure, it is deflected to the right
https://online.prepware.com/quiz_review_graded?quiz_id=6393576 6/3/2024, 5 53 pm
Page 152 of 218
:
by the Coriolis force. Thus, the wind around a high blows clockwise. Air moving
out of a high depletes the quantity of air. Highs and ridges are areas of descending
air.
478. What is the approximate base of the cumulus clouds if the temperature at 2,000
feet MSL is 10°C and the dewpoint is 1°C?
479. The most severe weather conditions, such as destructive winds, heavy hail, and
tornadoes, are generally associated with
B. squall lines.
Answer (A) is incorrect because warm fronts do not usually produce severe
weather. Answer (C) is incorrect because the weather produced by occluded fronts
is not as severe as a squall line.
480. What determines the structure or type of clouds which will form as a result of air
being forced to ascend?
Whether the air is stable or unstable within a layer largely determines cloud
structure. When stable air is forced upward the air tends to retain horizontal flow
and any cloudiness is flat and stratified. When unstable air is forced upward, the
https://online.prepware.com/quiz_review_graded?quiz_id=6393576 6/3/2024, 5 53 pm
Page 153 of 218
:
disturbance grows and any resulting cloudiness shows extensive vertical
development.
Answer (A) is incorrect because the stability determines the type of clouds that
form. Answer (C) is incorrect because the relative humidity determines the amount
of clouds that form.
481. The conditions most favorable to wave formation over mountainous areas are a
layer of
stable air at mountaintop altitude and a wind of at least 20 knots blowing across
A.
the ridge.
unstable air at mountaintop altitude and a wind of at least 20 knots blowing across
B.
the ridge.
moist, unstable air at mountaintop altitude and a wind of less than 5 knots blowing
C.
across the ridge.
A strong mountain wave requires:
1. Marked stability in the airstream disturbed by the mountains;
2. Wind speed at the level of the summit should exceed a minimum which varies
from 15 to 25 knots, depending on the height of the range; and
3. Wind direction within 30° to the range. Lift diminishes as winds more closely
parallel the range.
482. When flying low over hilly terrain, ridges, or mountain ranges, the greatest
potential danger from turbulent air currents will usually be encountered on the
Answer (A) is incorrect because with a tailwind you would be flying away from the
mountain with the wind. Answer (C) is incorrect because you would be flying in air
that is rising up on the windward side.
483. During departure, under conditions of suspected low-level wind shear, a sudden
decrease in headwind will cause
https://online.prepware.com/quiz_review_graded?quiz_id=6393576 6/3/2024, 5 53 pm
Page 154 of 218
:
C. no change in airspeed, but groundspeed will decrease.
The worst situation on departure occurs when the aircraft encounters a rapidly
increasing tailwind, decreasing headwind, and/or downdraft. Taking off under
these circumstances would lead to a decreased performance condition. An
increasing tailwind or decreasing headwind, when encountered, will cause a
decrease in indicated airspeed. The aircraft will initially pitch down due to the
decreased lift in proportion to the airspeed loss. After encountering the shear, if
the wind remains constant, aircraft ground speed will gradually increase and
indicated airspeed will return to its original value.
Answer (B) is incorrect because a sudden decrease in headwind will cause a loss
in airspeed. Answer (C) is incorrect because there is an initial loss of airspeed,
followed by an increase in ground speed.
484. Which weather phenomenon signals the beginning of the mature stage of a
thunderstorm?
The mature stage of a thunderstorm starts when precipitation begins to fall from
the cloud base. The downdrafts reach speeds that may exceed 2,500 fpm.
Meanwhile, updrafts reach a maximum with speeds possibly exceeding 6,000
fpm.
Answer (B) is incorrect because the anvil top appears during the mature stage, but
not necessarily at the beginning. Answer (C) is incorrect because maximum cloud
growth rate occurs in the middle to the end of the mature stage.
485. When flying into a low-pressure area in the Northern Hemisphere, the wind
direction and velocity will be from the
https://online.prepware.com/quiz_review_graded?quiz_id=6393576 6/3/2024, 5 53 pm
Page 155 of 218
:
486. Which statement is true concerning the hazards of hail?
Hail damage in horizontal flight is minimal due to the vertical movement of hail in
A.
the clouds.
Hailstones can fall some distance from the storm core. Hail has been observed in
clear air several miles from the parent thunderstorm.
Answer (A) is incorrect because hail is one of the greatest hazards to aircraft.
Answer (B) is incorrect because rain at the surface does not mean the absence of
hail aloft.
487. What prevents air from flowing directly from high-pressure areas to low-pressure
areas?
A. Coriolis force.
B. Surface friction.
The pressure gradient force drives the wind and is perpendicular to isobars. When
a pressure gradient force is first established, wind begins to blow from higher to
lower pressure directly across the isobars. However, the instant air begins moving,
Coriolis force deflects it to the right. Soon the wind is deflected a full 90° and is
parallel to the isobars or contours. At this time, Coriolis force exactly balances
pressure gradient force. With the forces in balance, wind will remain parallel to
isobars or contours.
Answer (B) is incorrect because surface friction moves air from highs to lows by
decreasing wind speed, which decreases the effect of the Coriolis force. Answer
(C) is incorrect because the pressure gradient force causes the initial movement
from high-pressure areas to low-pressure areas.
It can be present at any level and can exist in both a horizontal and vertical
C.
direction.
Wind shear may be associated with either a wind shift or a wind speed gradient at
any level in the atmosphere. It may be associated with a low-level temperature
inversion, in a frontal zone, or clear air turbulence (CAT) at high levels associated
https://online.prepware.com/quiz_review_graded?quiz_id=6393576 6/3/2024, 5 53 pm
Page 156 of 218
:
with a jet stream or strong circulation.
Answer (A) is incorrect because wind shear occurs both vertically and horizontally,
and at all altitudes. Answer (B) is incorrect because wind shear occurs both
vertically and horizontally, and at all altitudes.
A change in ambient temperature lapse rate of an air mass will determine its
stability. Surface cooling or warming aloft often tips the balance toward greater
stability.
Answer (A) is incorrect because warming from below decreases stability. Answer
(C) is incorrect because a decrease in water vapor lowers the dewpoint of the air,
but does not affect stability.
491. When an air mass is stable, which of these conditions are most likely to exist?
Smoke, dust, haze, etc., concentrated at the lower levels with resulting poor
C.
visibility.
https://online.prepware.com/quiz_review_graded?quiz_id=6393576 6/3/2024, 5 53 pm
Page 157 of 218
:
turbulence are characteristic of an unstable air mass. Answer (B) is incorrect
because towering cumulus, cumulonimbus clouds, and turbulence are
characteristic of an unstable air mass.
A. Cumuliform clouds.
B. Excellent visibility.
C. Restricted visibility.
Answer (A) is incorrect because cumuliform clouds and excellent visibility are
characteristic of an unstable air mass. Answer (B) is incorrect because cumuliform
clouds and excellent visibility are characteristic of an unstable air mass.
494. The wind system associated with a low-pressure area in the Northern Hemisphere
is
The storms that develop between high-pressure systems are characterized by low
pressure. As winds try to blow inward toward the center of low pressure, they are
also deflected to the right. Thus, the wind around a low moves in a
counterclockwise direction. The low pressure and its wind system is a cyclone.
Answer (A) is incorrect because they describe a high-pressure system. Answer (C)
is incorrect because they describe a high-pressure system.
https://online.prepware.com/quiz_review_graded?quiz_id=6393576 6/3/2024, 5 53 pm
Page 158 of 218
:
495. Which feature is associated with the tropopause?
As the thin boundary layer between the troposphere and the stratosphere, the
tropopause signals an abrupt change in temperature lapse rate.
Answer (A) is incorrect because the tropopause is farther away from the Earth's
surface at the equator than the poles. Answer (C) is incorrect because clouds may
form above the tropopause.
A. Cumuliform clouds.
B. Showery precipitation.
C. Continuous precipitation.
Answer (A) is incorrect because cumuliform clouds and showery precipitation are
characteristic of an unstable air mass. Answer (B) is incorrect because cumuliform
clouds and showery precipitation are characteristic of an unstable air mass.
497. Which weather chart depicts conditions forecast to exist at a specific time in the
future?
Prognostic charts portray weather forecasts for sometime in the future for the
conterminous U.S. and adjacent areas.
Answer (A) is incorrect because there is no chart named the Freezing Level Chart.
Answer (B) is incorrect because the Weather Depiction Charts show conditions
valid only for the time given for the chart.
https://online.prepware.com/quiz_review_graded?quiz_id=6393576 6/3/2024, 5 53 pm
Page 159 of 218
:
498. What type of Inflight Weather Advisories provides an en route pilot with
information regarding the possibility of moderate icing, moderate turbulence,
winds of 30 knots or more at the surface and extensive mountain obscurement?
AIRMETs and CWAs may be of significance to any pilot or aircraft operator and are
issued for all domestic airspace. They are of particular concern to operators and
pilots of aircraft sensitive to the phenomena described and to pilots without
instrument ratings. They are issued for the following weather phenomena which
are potentially hazardous to aircraft: moderate icing, moderate turbulence,
sustained winds of 30 knots or more at the surface, widespread area of ceilings
less than 1,000 feet and/or visibility less than three miles, and extensive mountain
obscurement.
499. The station originating the following METAR observation has a field elevation of
3,500 feet MSL. If the sky cover is one continuous layer, what is the thickness of
the cloud layer? (Top of overcast reported at 7,500 feet MSL).
METAR KHOB 151250Z 17006KT 4SM OVC005 13/11 A2998
A. 2,500 feet.
B. 3,500 feet.
C. 4,000 feet.
KHOB reports a ceiling of 500 feet (OVC005). This means the bottom of the
overcast layer is 4,000 feet (3,500 feet MSL + 500 feet AGL). The top of the
overcast is reported at 7,500 feet MSL. Therefore, the overcast layer is 3,500 feet
thick (7,500 - 4,000).
500. Given the following, what is the maximum altitude to maintain VFR below the
ceiling in Class E airspace below 10,000 feet MSL.
METAR: KHOB 151250Z, 17006KT 4SM BKN025 OVC040 13/11 A2998
KHOB reports a broken ceiling at 2,500 feet (BKN025) and ceiling of 4,000 feet
(OVC040). VFR in Class E airspace below 10,000 feet requires the pilot to maintain
https://online.prepware.com/quiz_review_graded?quiz_id=6393576 6/3/2024, 5 53 pm
Page 160 of 218
:
1,000 feet above and 500 feet below distance from clouds. In this instance, the
maximum altitude to maintain VFR is 3,500 feet AGL.
501. The minimum vertical wind shear value critical for probable moderate or greater
turbulence is
The vertical shear critical for probable turbulence is 6 knots per 1,000 feet.
502. From which of the following can the observed temperature, wind, and
temperature/dewpoint spread be determined at a specified altitude?
A. Stability Charts.
503. The U.S. High-Level Significant Weather Prognostic Chart forecasts significant
weather for what airspace?
504. Which chart provides a ready means of locating observed frontal positions and
pressure centers?
https://online.prepware.com/quiz_review_graded?quiz_id=6393576 6/3/2024, 5 53 pm
Page 161 of 218
:
B. Constant Pressure Analysis Chart.
Surface analysis charts are used to denote pressure systems and fronts.
505. What weather phenomenon is implied within an area enclosed by small scalloped
lines on a U.S. High-Level Significant Weather Prognostic Chart?
Wind direction is with reference to true north and wind speed is given in knots on
winds aloft forecast reports.
507. The most current en route and destination weather information for an instrument
flight should be obtained from the
A. Flight service.
B. ATIS broadcast.
Flight Service provides more aviation weather briefing service than any other
government service outlet. FSS provides weather briefings, scheduled and
unscheduled weather broadcasts, and furnishes weather support to flight in its
area.
https://online.prepware.com/quiz_review_graded?quiz_id=6393576 6/3/2024, 5 53 pm
Page 162 of 218
:
Answer (B) is incorrect because ATIS provides information for operations at a
specific airport and does not provide information for enroute operations. Answer
(C) is incorrect because NOTAM publications contain information on airport
operations and would not have information for en route and destination weather.
508. When turbulence causes changes in altitude and/or attitude, but aircraft control
remains positive, that should be reported as
A. light.
B. severe.
C. moderate.
Answer (A) is incorrect because light turbulence momentarily causes slight, erratic
changes in altitude and/or attitude. Answer (B) is incorrect because severe
turbulence causes large, abrupt changes in altitude and/or attitude and the aircraft
may be momentarily out of control.
509. What flight planning information can a pilot derive from Constant Pressure
Analysis Charts?
Answer (B) is incorrect because clear air turbulence can be found on prognostic
charts and in area forecasts. Answer (C) is incorrect because frontal systems and
obstructions to vision aloft can be found on surface analysis and weather
depiction charts.
510. On a Surface Analysis Chart, the solid lines that depict sea level pressure patterns
are called
A. isobars.
B. isogons.
https://online.prepware.com/quiz_review_graded?quiz_id=6393576 6/3/2024, 5 53 pm
Page 163 of 218
:
C. millibars.
Isobars are solid lines depicting the pressure pattern. They are usually spaced at
four-millibar intervals on a surface analysis chart.
Answer (B) is incorrect because isogons are lines of magnetic variation, found on
navigational charts. Answer (C) is incorrect because millibars are units of pressure.
511. The difference found by subtracting the temperature of a parcel of air theoretically
lifted from the surface to 500 millibars and the existing temperature at 500
millibars is called the
A. lifted index.
B. negative index.
C. positive index.
The lifted index is computed as if a parcel of air near the surface were lifted to 500
mb. As the air is 'lifted,' it cools by expansion. The temperature the parcel would
have at 500 mb is then subtracted from the existing 500 mb temperature. The
difference is the lifted index; it may be positive, zero, or negative.
Answer (B) is incorrect because a positive index means the lifted parcel of air is
colder than the existing air at 500 mb, and the air is stable. Answer (C) is incorrect
because a negative index means the lifted parcel of air is warmer than the existing
air at 500 mb, and the air is unstable.
512. What is the meaning of the terms PROB40 2102 +TSRA as used in a Terminal
Aerodrome Forecasts (TAF)?
Probability of heavy thunderstorms with rain showers below 4000 feet at time
A.
2102.
Between 2100Z and 0200Z there is a forty percent (40%) probability of
B.
thunderstorms with heavy rain.
Beginning at 2102Z forty percent (40%) probability of heavy thunderstorms and
C.
rain showers.
The TAF reports there is a 40% probability (PROB40) between 2100Z and 0200Z
(2102) of thunderstorms and heavy rain (+TSRA).
513. What is meant by the Special METAR weather observation for KBOI?
SPECI KBOI 091854Z 32005KT 1 1/2SM RA BR OVC007 17/16 A2990 RMK
RAB12
A. Rain and fog obscuring two-tenths of the sky; rain began at 1912Z.
https://online.prepware.com/quiz_review_graded?quiz_id=6393576 6/3/2024, 5 53 pm
Page 164 of 218
:
B. Rain and mist obstructing visibility; rain began at 1812Z.
514. What is the upper limit of the Low Level Significant Weather Prognostic Chart?
A. 30,000 feet.
B. 24,000 feet.
C. 18,000 feet.
A. hurricane eye.
Answer (A) is incorrect because the hurricane eye has very low winds. Answer (C)
is incorrect because a clear area within a hatched area indicates wind of 110 to
150 knots.
516. In the following METAR/TAF for HOU, what is the ceiling and visibility forecast on
the 7th day of the month at 0600Z?
KHOU 061734Z 0618/0718 (tel:0618/0718) 16014G22KT P6SM VCSH BKN018
BKN035
FM070100 17010KT P6SM BKN015 OVC025
FM070500 17008KT 4SM BR SCT008 OVC012
https://online.prepware.com/quiz_review_graded?quiz_id=6393576 6/3/2024, 5 53 pm
Page 165 of 218
:
FM071000 18005KT 3SM BR OVC007
FM071500 23008KT 5SM BR VCSH SCT008 OVC015
The third line of the TAF reads from the 7th at 0500 wind 170 at 8 knots, visibility 4
statute miles in mist, scattered 800 AGL, overcast 1,200 AGL. Because the next
weather forecast is not given until the 7th at 1000, you would use the information
from the forecast for the 7th at 0500 to answer the question, "FM070500 17008KT
4SM BR SCT008 OVC012." Remember that a scattered layer does not signify a
ceiling.
517. During preflight preparation, weather report forecasts can best be obtained by
means of contacting the
Reports and forecasts are available through the request/reply service at all FSSs,
WSOs, and WSFOs.
Answer (B) is incorrect because ARTCC deals with air traffic control. Answer (C) is
incorrect because PATWAS is limited in the number of route forecasts and
synopses it can provide.
Answer (A) is incorrect because the surface analysis chart reports weather as of
the time of observation. Answer (C) is incorrect because surface analysis charts do
not indicate cloud heights.
https://online.prepware.com/quiz_review_graded?quiz_id=6393576 6/3/2024, 5 53 pm
Page 166 of 218
:
519. The remarks section of the Aviation Routine Weather Report (METAR) contains the
following coded information. What does it mean?
RMK FZDZB42 WSHFT 30 FROPA
Answer (A) is incorrect because farther spaced isobars indicate a weaker pressure
gradient. Answer (C) is incorrect because isotherms indicate changing
temperatures.
Answer (A) is incorrect because SIGMETs are unscheduled forecasts. Answer (B) is
incorrect because SIGMETs are issued only for severe thunderstorms and hail 3/4
https://online.prepware.com/quiz_review_graded?quiz_id=6393576 6/3/2024, 5 53 pm
Page 167 of 218
:
inch or larger and they can be valid for up to 2 hours.
522. SIGMETs are issued as a warning of weather conditions which are hazardous
A. to all aircraft.
C. Satellite Maps.
The GFA is intended to provide the necessary aviation weather information to give
users a complete picture of the weather that may impact flight in the continental
U.S. (CONUS).
524. Turbulence that is encountered above 15,000 feet AGL not associated with
cumuliform cloudiness, including thunderstorms, should be reported as
A. severe turbulence.
C. convective turbulence.
High-level turbulence (normally above 15,000 feet AGL) not associated with
cumuliform cloudiness, including thunderstorms, should be reported as CAT (clear
air turbulence) preceded by the appropriate intensity, or light or moderate chop.
https://online.prepware.com/quiz_review_graded?quiz_id=6393576 6/3/2024, 5 53 pm
Page 168 of 218
:
525. What significant cloud coverage is reported by this pilot report?
KMOB UA/OV 15NW MOB 1340Z/SK OVC-TOP025 / OVC045-TOP090
A. Three (3) separate overcast layers exist with bases at 250, 7,500 and 9,000 feet.
The top of the lower overcast is 2,500 feet; base and top of second overcast layer
B.
is 4,500 and 9,000 feet, respectively.
The base of the second overcast layer is 2,500 feet; top of second overcast layer
C.
is 7,500 feet; base of third layer is 9,000 feet.
UA - pilot report
/OV 15NW MOB - pilot was 15 miles northwest of Mobile, at 1340 UTC time
/SK OVC-TOP025 - sky coverage, overcast layer top at 2,500 feet
/OVC045-TOP090 - a second layer is at 4,500 feet with the top at 9,000 feet MSL.
526. Terminal Aerodrome Forecasts (TAF) are issued how many times a day and cover
what period of time?
A. Four times daily and are usually valid for a 24 hour period.
Six times daily and are usually valid for a 24 hour period including a 4-hour
B.
categorical outlook.
C. Four times daily and are valid for 12 hours including a 6-hour categorical outlook.
527. Dashed lines on a Surface Analysis Chart, if depicted, indicate that the pressure
gradient is
A. weak.
B. strong.
C. unstable.
When a pressure gradient is weak, dashed isobars are sometimes inserted at two-
millibar intervals to more clearly define the pressure pattern.
Answer (B) is incorrect because strong pressure gradients are indicated by closely
spaced solid isobars at 4-mb intervals. Answer (C) is incorrect because stability is
concerned with temperature lapse rates.
528. Weather Advisory Broadcasts, including Severe Weather Forecast Alerts (AWW),
SIGMETs, and Convective SIGMETs are provided by
https://online.prepware.com/quiz_review_graded?quiz_id=6393576 6/3/2024, 5 53 pm
Page 169 of 218
:
ARTCCs on all frequencies, except emergency, when any part of the area
A.
described is within 150 miles of the airspace under their jurisdiction.
Flight Service on 122.2 MHz and adjacent VORs, when any part of the area
B.
described is within 200 miles of the airspace under their jurisdiction.
529. Which statement pertaining to the following Terminal Aerodrome Forecast (TAF) is
true?
TAF
KMEM 091135Z 0915 15005KT 5SM HZ BKN060
FM1600 VRB04KT P6SM SKC
Wind in the valid period implies surface winds are forecast to be greater than 5
A.
KTS.
B. Wind direction is from 160° at 4 KTS and reported visibility is 6 statute miles.
C. SKC in the valid period indicates no significant weather and sky clear.
530. What is the bottom of the lowest overcast layer in the following pilot report?
KMOB UA /OV APE230010/TM 1515/FL085/TP BE20/ SK BKN065/WX FV03SM
HZ FU/TA 20/TB LGT
The sky cover portion of the report states "SK BKN 065" which means sky cover is
https://online.prepware.com/quiz_review_graded?quiz_id=6393576 6/3/2024, 5 53 pm
Page 170 of 218
:
broken at 6,500 feet.
Answer (A) is incorrect because the ceiling is defined in the "SK" section of the
report. Answer (B) is incorrect because "FL085" is detailing the flight level the
report was given at, not the sky coverage.
531. A pilot reporting turbulence that momentarily causes slight, erratic changes in
altitude and/or attitude should report it as
A. light chop.
B. light turbulence.
C. moderate turbulence.
Answer (A) is incorrect because light chop is slight, rapid, somewhat rhythmic
bumpiness. Answer (C) is incorrect because moderate turbulence causes changes
in altitude and/or attitude, and variations in indicated airspeed.
A. pilot reports.
B. Area Forecasts.
C. prognostic charts.
Pilot weather reports are the best means to determine observed weather
conditions between weather-reporting stations. It is also the only method of
directly observing cloud tops, icing and turbulence.
Answer (B) is incorrect because Area Forecasts and prognostic charts are
forecasts and not observed weather. Answer (C) is incorrect because Area
Forecasts and prognostic charts are forecasts and not observed weather.
533. What single reference contains information regarding a volcanic eruption, that is
occurring or expected to occur?
https://online.prepware.com/quiz_review_graded?quiz_id=6393576 6/3/2024, 5 53 pm
Page 171 of 218
:
eruption, that is occurring or expected to occur.
534. What wind conditions would you anticipate when squalls are reported at your
destination?
Peak gusts of at least 35 knots combined with a change in wind direction of 30° or
B.
more.
Sudden increases in windspeed of at least 16 knots to a sustained speed of 22
C.
knots or more for at least 1 minute.
Answer (A) is incorrect because this describes gusts. Answer (B) is incorrect
because this describes wind shear.
535. What does the contraction VRB in the Terminal Aerodrome Forecast (TAF) mean?
A variable wind direction is noted by 'VRB' where the three-digit direction usually
appears.
536. (Refer to Figure 31.) Rwy 30 is being used for landing. Which surface wind would
exceed the airplane's crosswind capability of 0.2 V(SO), if V(SO) is 60 knots?
A. 260° at 20 knots.
B. 275° at 25 knots.
C. 315° at 35 knots.
https://online.prepware.com/quiz_review_graded?quiz_id=6393576 6/3/2024, 5 53 pm
Page 172 of 218
:
537. (Refer to Figure 10.) Using a maximum rate of climb, how much fuel would be
used from engine start to 6,000 feet pressure altitude?
Aircraft weight 3,200 lb
Airport pressure altitude 2,000 ft
Temperature 27°C
A. 10 pounds.
B. 14 pounds.
C. 24 pounds.
1. Locate the section for 3,200 pounds weight. Read across the 2,000-foot PA line
to the entry under fuel used, 4 pounds.
2. Read across the 6,000-foot PA line to the entry under fuel used, 14 pounds.
3. Calculate the fuel required to climb:
14 - 4 = 10 lbs
4. Apply Note #2 before adding the fuel in Note 1. (A temperature of 27°C is
+16°C, with respect to the standard atmosphere at 2,000 feet.)
10 x 1.16 = 11.6 lbs
5. Apply Note #1:
11.6 lbs + 12.0 lbs start and taxi = 23.6 lbs total.
538. GIVEN:
Total weight 4,137 lb
CG location station 67.8
Fuel consumption 13.7 GPH
Fuel CG station 68.0
After 1 hour 30 minutes of flight time, the CG would be located at station
A. 67.79.
B. 68.79.
C. 70.78.
1. Find the weight change: 13.7 GPH for 1.5 hours = 20.55 gal
20.55 gal x 6 lbs/gal = 123.3 lbs
2. New total weight is 4,137 - 123.3 = 4,013.7
3. The distance between the CG and the fuel arm is 68.0 - 67.8 = .2
4. Place the values in the formula and cross multiply:
123.3 ÷ 4013.7 = CG change ÷ .2 = 24.66 = 4013.7(CG change)
5. Divide to determine CG change:
24.66/4013.7 = CG change = .00614
123.3 ÷ 4013.7 = CG change ÷ .2 = .00614 inches
6. Calculate the new CG:
https://online.prepware.com/quiz_review_graded?quiz_id=6393576 6/3/2024, 5 53 pm
Page 173 of 218
:
Original CG 67.80000 - CG change .00614 = New CG 67.79386.
C. No, the weight is acceptable, but the CG is aft of the aft limit.
1. Construct a table with the item, weight, and moment. The moment of each item
is found by using the loading graph in FAA Figure 38. Locate the weight of each
item on the left side of the chart, and proceed to the right to intersect the correct
item line. At point of intersection, proceed downward to find the moment for that
item at the specified weight.
Item Weight Moment
Aircraft empty weight 1,271 102.04
Pilot and copilot 400 36.00
Aft passenger 140 18.00
Cargo 100 11.50
Fuel (37.0 gal x 6 lbs/gal) + 222 + 20.00
Totals 2,133 187.54
2. Compare the total weight and moment to the limits in the center of gravity
envelope in FAA Figure 38. Move up the loaded aircraft weight scale to 2,133, then
across to 187.54 on the Moment/1,000 scale. The weight and CG are within limits.
A. 800 feet.
B. 650 feet.
https://online.prepware.com/quiz_review_graded?quiz_id=6393576 6/3/2024, 5 53 pm
Page 174 of 218
:
C. 1050 feet.
541. What effect does an uphill runway slope have on takeoff performance?
The effect of runway slope on takeoff distance is due to the component of weight
along the inclined path of the aircraft. An upslope would contribute a retarding
force component while a downslope would contribute an accelerating force
component. In the case of an upslope, the retarding force component adds to
drag and rolling friction to reduce the net accelerating force.
A. 3 hours 8 minutes.
B. 3 hours 22 minutes.
C. 3 hours 43 minutes.
1. Enter chart at the point where the 55% maximum continuous power line
intersects the curved cruise (lean) line.
2. From that point, proceed to the left and read fuel flow: 11.4 GPH
3. Compute time of flight:
47 ÷ 11.4 = 4.12 hours
4. Compute available flight time with a 45-minute (0.75 hour) reserve:
4.12 - 0.75 = 3.37 = 3 hours, 22 minutes.
A. radio communications are key to alerting other aircraft in the pattern that a go-
https://online.prepware.com/quiz_review_graded?quiz_id=6393576 6/3/2024, 5 53 pm
Page 175 of 218
:
around maneuver is being conducted.
the airplane is trimmed for a power-off condition, and application of takeoff power
B.
will cause the nose to rise rapidly
flaps should be raised as quickly as possible to reduce drag and increase airspeed
C.
for a successful go around.
When takeoff power is applied, it is usually necessary to hold considerable
pressure on the controls to maintain straight flight and a safe climb attitude. Since
the airplane is trimmed for the approach (a low power and low airspeed condition),
application of maximum allowable power requires considerable control pressure to
maintain a climb pitch attitude. The addition of power tends to raise the airplane's
nose suddenly and it veers to the left.
Answer (A) is incorrect because power is the pilot's first concern in a go-around
situation; aviate before you communicate. Answer (C) is incorrect because flaps
should only be retracted once power has been applied and the proper climb
attitude established
544. (Refer to Figure 11.) If the cruise altitude is 7,500 feet, using 64 percent power at
2,500 RPM, what would be the range with 48 gallons of usable fuel?
A. 635 miles.
B. 645 miles.
C. 810 miles.
545. GIVEN:
Weight A: 175 pounds at 135 inches aft of datum
Weight B: 135 pounds at 115 inches aft of datum
Weight C: 75 pounds at 85 inches aft of datum
The CG for the combined weights would be located how far aft of datum?
A. 91.76 inches.
B. 111.67 inches.
C. 118.24 inches.
https://online.prepware.com/quiz_review_graded?quiz_id=6393576 6/3/2024, 5 53 pm
Page 176 of 218
:
175 x 135 = 23,625 for A
135 x 115 = 15,525 for B
+ 75 x 85 = + 6,375 for C
385 lbs 45,525 lbs-in
2. Moments ÷ Total weight = CG location
45,525 ÷ 385 = 118.25 inches ^
Direction of motion and the longitudinal axis must be parallel to the runway or
skidding will occur, with possible damage to the aircraft.
Answer (A) is incorrect because the direction of motion of the airplane must be
parallel to the runway. Answer (C) is incorrect because the upwind wing should be
lowered to eliminate drift.
547. (Refer to Figure 31.) What is the headwind component for a Rwy 13 takeoff if the
surface wind is 190° at 15 knots?
A. 7 knots.
B. 13 knots.
C. 15 knots.
https://online.prepware.com/quiz_review_graded?quiz_id=6393576 6/3/2024, 5 53 pm
Page 177 of 218
:
A. 425 feet.
B. 636 feet.
C. 836 feet.
1. Enter the chart at the 50°F OAT point and proceed upward to intercept the sea
level pressure altitude line. Proceed right to the intersection with the vertical
reference line.
2. From the reference line, proceed upward to the right following the trend line,
until reaching a vertical line representing the weight of 3,000 pounds.
3. Draw a line to the right from the intercept with the weight line, to the intercept
with the right vertical reference line.
4. Construct a vertical line to represent the headwind, 10 knots. Draw a line from
the intercept with the reference line to the intersect with the headwind line.
Proceed to the right and read the total distance over a 50-foot obstacle: 1,200
feet.
5. Apply the note to calculate the ground roll (53% of the landing distance).
1,200 x 0.53 = 636 feet ground roll.
A. 1,100 feet.
B. 1,300 feet.
C. 1,500 feet.
https://online.prepware.com/quiz_review_graded?quiz_id=6393576 6/3/2024, 5 53 pm
Page 178 of 218
:
Airport pressure altitude 2,000 ft
Temperature at 2,000 feet 32°C
Using a maximum rate of climb under the given conditions, how much time would
be required to climb to a pressure altitude of 8,000 feet?
A. 7 minutes.
B. 8.4 minutes.
C. 11.2 minutes.
1. Determine the time required to climb from sea level to the PA of the airport
(2,000 feet) by interpolating between the sea level and the 4,000-foot values (for
an aircraft weight of 4,000 lbs):
(0 + 4) ÷ 2 = 2 minutes
2. Determine the time required to climb from sea level to 8,000-foot pressure
altitude: 9 minutes
3. Determine the time required to climb from 2,000 feet to 8,000-foot pressure
altitude (uncorrected, for nonstandard temperature):
9 - 2 = 7 minutes
4. Correct the time required for nonstandard temperature by applying Note #2,
using 21% increase (+32°C is +21°C relative to the standard atmosphere at 2,000
feet):
1.21 x 7 = 8.4 min.
551. An aircraft is loaded with a ramp weight of 3,650 pounds and having a CG of 94.0,
approximately how much baggage would have to be moved from the rear
baggage area at station 180 to the forward baggage area at station 40 in order to
move the CG to 92.0?
A. 52.14 pounds.
B. 62.24 pounds.
C. 78.14 pounds.
A. -15°C.
B. -20°C.
C. -25°C.
https://online.prepware.com/quiz_review_graded?quiz_id=6393576 6/3/2024, 5 53 pm
Page 179 of 218
:
Standard temperature at sea level is 15°C. The average lapse rate is 2°C/1,000
feet.
20 x -2 = -40°C temperature decrease to 20,000 feet
15°C - 40°C = -25°C standard temperature at 20,000 feet
A. 6 hours 27 minutes.
B. 6 hours 39 minutes.
C. 6 hours 56 minutes.
1. Enter the Cruise Performance Chart at the 2,200 RPM section (the left-hand
column). In that section, move right and find the 22" MP row. Follow that row to
the right and read 70 pph under the 3°C columns.
2. Calculate the available flight endurance using:
Time = Fuel ÷ Fuel Burn Rate = 465 ÷ 70 = 6.643 hours
3. Convert .643 hours into minutes: 0.643 x 60 = 38.6 minutes
4. Therefore, the flight time is 6 hours, 39 minutes.
Each cargo compartment must be designed for its placarded maximum weight of
contents in relation to the critical load factors. Normal category aircraft having a
gross weight of less than 4,000 pounds are designed for a maneuvering load
factor of 3.8 times the aircraft weight or 3.8 Gs. Thus a load factor of 3.5 Gs would
not be excessive. Load factor is the ratio of a given load to its weight. Thus:
90.0 lbs weight x 3.5 Gs = 315.0 lbs loading.
555. (Refer to Figure 9.) Using a normal climb, how much fuel would be used from
engine start to 12,000 feet pressure altitude?
https://online.prepware.com/quiz_review_graded?quiz_id=6393576 6/3/2024, 5 53 pm
Page 180 of 218
:
Aircraft weight 3,800 lb
Airport pressure altitude 4,000 ft
Temperature 26°C
A. 46 pounds.
B. 51 pounds.
C. 58 pounds.
1. Locate the section for 3,800 pounds weight. Read across the 4,000-foot PA line
to the entry under fuel used, 12 pounds.
2. Read across the 12,000-foot PA line to the entry under fuel used, 51 pounds.
3. Calculate the fuel required to climb:
51 - 12 = 39 lbs
4. Apply Note #2 before adding the fuel in Note 1. (A temperature of 26°C is
+19°C, relative to the standard atmosphere at 4,000 feet.)
39 x 1.19 = 46.4 lbs
5. Apply Note #1:
46.4 lbs + 12.0 lbs start and taxi = 58.4 lbs total.
556. With regard to the technique required for a crosswind correction on takeoff, a pilot
should use
aileron pressure into the wind and initiate the lift-off at a normal airspeed in both
A.
tailwheel and nosewheel-type airplanes.
right rudder pressure, aileron pressure into the wind, and higher than normal lift-off
B.
airspeed in both tricycle- and conventional-gear airplanes.
rudder as required to maintain directional control, aileron pressure into the wind,
C. and higher than normal lift-off airspeed in both conventional- and nosewheel-type
airplanes.
Rudder is required at all times to maintain directional control. Aileron pressure into
the wind will keep the wing down and prevent side skipping. A slightly higher than
normal takeoff speed is desirable to insure a very definite lift-off with no settling.
Answer (A) is incorrect because the lift-off speed should be increased slightly and
rudder pressure should be applied to maintain directional control. Answer (B) is
incorrect because rudder pressure should be applied to maintain directional
control, this can be both right or left deflection.
557. What effect, if any, would a change in ambient temperature or air density have on
gas turbine engine performance?
https://online.prepware.com/quiz_review_graded?quiz_id=6393576 6/3/2024, 5 53 pm
Page 181 of 218
:
C. As temperature increases, thrust decreases.
An increase in altitude causes the engine air flow to decrease in a manner nearly
identical to the altitude density ratio. This causes a significant decrease in thrust at
altitude versus thrust at sea level as altitude increases. An increase in inlet air
temperature will provide a lower combustion gas energy and cause a lower jet
velocity.
558. (Refer to Figure 10.) Using a maximum rate of climb, how much fuel would be
used from engine start to 10,000 feet pressure altitude?
Aircraft weight 3,800 lb
Airport pressure altitude 4,000 ft
Temperature 30°C
A. 28 pounds.
B. 35 pounds.
C. 40 pounds.
1. Locate the section for 3,800 pounds weight. Read across the 4,000-foot PA line
to the entry under fuel used, 12 pounds.
2. Read across the 10,000-foot PA line to the entry under fuel used, 35 pounds.
3. Calculate the fuel required to climb:
35 - 12 = 23 lbs
4. Apply Note #2 before adding the fuel in Note 1. (A temperature of 30°C is
+23°C, with respect to the standard atmosphere at 4,000 feet.)
23 x 1.23 = 28.3 lbs
5. Apply Note #1:
28.3 lbs +12.0 lbs start and taxi = 40.3 lbs total.
559. If all index units are positive when computing weight and balance, the location of
the datum would be at the
In general, the arm (index unit) of a location in the aircraft is in inches aft of the
datum. A positive arm indicates the position of the object is aft of the datum. If all
index units are positive, the datum must be at least at the nose of the aircraft.
Answer (A) is incorrect because the engine would have a negative arm if the datum
was at the centerline of the main wheels. Answer (C) is incorrect because the
propeller would have a negative arm if the datum were at the centerline of the
https://online.prepware.com/quiz_review_graded?quiz_id=6393576 6/3/2024, 5 53 pm
Page 182 of 218
:
nose or tailwheel.
560. GIVEN:
Total weight 3,037 lb
CG location station 68.8
Fuel consumption 12.7 GPH
Fuel CG station 68.0
After 1 hour 45 minutes of flight time, the CG would be located at station
A. 68.77.
B. 68.83.
C. 69.77.
1. Find the weight change: 12.7 GPH for 1.75 hours = 22.23 gal
22.23 gal x 6 lbs/gal = 133.35 lbs
2. New total weight is 3,037 - 133.35 = 2,903.65
3. The distance between the CG and the fuel arm is 68.8 - 68.0 = .8
4. Place the values in the formula and cross multiply:
133.35 ÷ 2903.65 = CG change ÷ .8 = 106.68 = 2903.65 (CG change)
5. Divide to determine the CG change:
106/68/2903.65 = CG change =.03674 inches
6. Calculate the new CG:
Original CG 68.80000 + CG change .03674 = New CG 68.83674.
A. 50 pounds.
B. 100 pounds.
C. 300 pounds.
1. Start at the right side of the chart and find the 1,000-foot takeoff distance mark.
From there, draw a horizontal line to the 20-knot headwind mark, and following the
sloped lines, draw a line to the reference line.
2. Draw a horizontal line all the way across the weight section to the other
reference line.
3. Starting from the left side of the chart, find the 75°F mark and draw a line up to
https://online.prepware.com/quiz_review_graded?quiz_id=6393576 6/3/2024, 5 53 pm
Page 183 of 218
:
the 6,000-foot pressure altitude line, and follow that sloped line across to the
reference line. Follow the sloped reference lines to the right, up to where your line
intercepts the horizontal line drawn previously across that section.
4. Draw a vertical line down from where they meet and read the maximum weight
(2,600 pounds) for the aircraft to be able to clear the 50-foot obstacle.
5. Subtract that weight from 2,900 pounds to get 300 pounds.
562. What are the standard temperature and pressure values for sea level?
A. 4.8 minutes.
B. 5 minutes.
C. 5.5 minutes.
1. Determine the time which would have been required, using a 3,400 pounds
weight, to climb from sea level to the airport pressure altitude of 4,000 feet, 5
minutes.
2. Determine the time to climb to 8,000 feet from sea level, 10 minutes.
3. The time required for climb, uncorrected for temperature, is 10 - 5 = 5 minutes.
4. Correct for nonstandard temperature (+14°C is +7°C, relative to the standard
atmosphere.).
5. Apply Note #2 using a 10% increase.
1.1 x 5 = 5.5 minutes.
564. (Refer to Figure 9.) Using a normal climb, how much fuel would be used from
engine start to 10,000 feet pressure altitude?
Aircraft weight 3,500 lb
Airport pressure altitude 4,000 ft
https://online.prepware.com/quiz_review_graded?quiz_id=6393576 6/3/2024, 5 53 pm
Page 184 of 218
:
Temperature 21°C
A. 23 pounds.
B. 31 pounds.
C. 35 pounds.
1. Locate the section for 3,500 pounds weight. Read across the 4,000-foot PA line
to the entry under fuel used, 11 pounds.
2. Read across the 10,000-foot PA line to the entry under fuel used, 31 pounds.
3. Calculate the fuel required to climb:
31 - 11 = 20 lbs
4. Apply Note #1. (A temperature of 21°C is +14°C, with respect to the standard
atmosphere at 4,000 feet.)
20 x 1.14 = 22.8 lbs
5. Apply Note #2:
22.8 lbs + 12.0 lbs start and taxi = 34.8 lbs total.
565. Which type of approach and landing is recommended during gusty wind
conditions?
566. (Refer to Figure 11.) What would be the approximate true airspeed and fuel
consumption per hour at an altitude of 7,500 feet, using 52 percent power?
https://online.prepware.com/quiz_review_graded?quiz_id=6393576 6/3/2024, 5 53 pm
Page 185 of 218
:
GIVEN:
Temperature 85°F
Pressure altitude 6,000 ft
Weight 2,800 lb
Headwind 14 kts
Determine the approximate ground roll.
A. 742 feet.
B. 1,280 feet.
C. 1,480 feet.
1. Enter the chart at the 85°F OAT point and proceed upward to intercept the
6,000-foot pressure altitude line. Proceed right to the intersection with the vertical
reference line.
2. From the reference line, proceed upward to the right following the trend line,
until reaching a vertical line representing the weight of 2,800 pounds.
3. Draw a line to the right from the intercept with the weight line, to the intercept
with the right vertical reference line.
4. Construct a vertical line to represent the headwind, 14 KIAS. Draw a line from
the intercept with the reference line to the intersect with the headwind line.
Proceed to the right and read the total distance over a 50-foot obstacle: 1,400
feet.
5. Apply the note to calculate the ground roll (53% of the landing distance).
1,400 x 0.53 = 742 feet ground roll.
568. The performance tables of an aircraft for takeoff and climb are based on
A. pressure/density altitude.
B. cabin altitude.
C. true altitude.
Pressure and density altitude are the factors used in the performance tables of the
aircraft for takeoff and climb.
Answer (B) is incorrect because the cabin altitude is the altitude that is in the
pressurized cabin. Answer (C) is incorrect because true altitude is the height above
sea level. Aeronautical charts depict obstacle elevations in true altitude.
https://online.prepware.com/quiz_review_graded?quiz_id=6393576 6/3/2024, 5 53 pm
Page 186 of 218
:
What is the maximum rate of climb under the given conditions?
A. 655 ft/min.
B. 702 ft/min.
C. 774 ft/min.
570. When turbulence is encountered during the approach to a landing, what action is
recommended and for what primary reason?
Increase the airspeed slightly above normal approach speed to attain more
A.
positive control.
Decrease the airspeed slightly below normal approach speed to avoid
B.
overstressing the airplane.
Increase the airspeed slightly above normal approach speed to penetrate the
C.
turbulence as quickly as possible.
Power-on approaches at an airspeed slightly above the normal approach speed
should be used for landing in significantly turbulent air. This provides for more
positive control of the airplane when strong horizontal wind gusts, or up- and
downdrafts, are experienced.
Answer (B) is incorrect because, since approach speed is well below V(A), no
structural damage will occur. Answer (C) is incorrect because increased approach
speed is to increase control effectiveness, not to fly through turbulence faster.
https://online.prepware.com/quiz_review_graded?quiz_id=6393576 6/3/2024, 5 53 pm
Page 187 of 218
:
A. 5 gallons, 9 minutes, 13 NM.
1. Enter the chart at 0°C and proceed to the 10,000-foot PA curve. From this point
proceed to the right. Note the intersections with the fuel, time and distance
curves. From these intersections, proceed downward and read:
Fuel = 6 gal, Time = 11 min, Dist = 16 NM
2. Enter the chart at +20°C and proceed upward to intercept the 2,000-foot PA
line, then right to the intersections with the fuel, time and distance curves. From
these intersections proceed downward and read:
Fuel = 1 gal, Time + 2 min, Dist = 3 NM
3. The differences are the fuel, time and distance to climb to cruise altitude:
= 6 - 1 = 5 gallons
Time = 11 - 2 = 9 minutes
Distance = 16 - 3 = 13 NM.
572. At higher elevation airports the pilot should know that indicated airspeed
An airplane at altitude will land at the same indicated airspeed as at sea level but,
because of the reduced air density, will have a greater true airspeed. If the true
airspeed is greater, the ground speed will be greater.
A. 305 ft/min.
B. 320 ft/min.
C. 384 ft/min.
https://online.prepware.com/quiz_review_graded?quiz_id=6393576 6/3/2024, 5 53 pm
Page 188 of 218
:
3. Interpolate or average to determine the rate of climb at 22,000 feet and -10°C;
incorporate the results for 20,000 and 24,000 feet
obtained in Steps 1 and 2.
(535 + 233) ÷ 2 = 384 fpm.
574. GIVEN:
Weight A: 140 pounds at 17 inches aft of datum
Weight B: 120 pounds at 110 inches aft of datum
Weight C: 85 pounds at 210 inches aft of datum
Based on this information, the CG would be located how far aft of datum?
A. 89.11 inches.
B. 96.89 inches.
C. 106.92 inches.
A. 1,180 feet.
B. 1,350 feet.
C. 1,850 feet.
https://online.prepware.com/quiz_review_graded?quiz_id=6393576 6/3/2024, 5 53 pm
Page 189 of 218
:
feet.
5. Calculate the ground roll using the note that the ground roll is approximately
73% of total take-off distance over a 50-foot obstacle:
1,850 x .73 = 1,350 feet ground roll.
A. 4 hours 58 minutes.
B. 5 hours 7 minutes.
C. 5 hours 12 minutes.
1. Enter the Cruise Performance Chart at the 2,500 RPM section (the left-hand
column). In that section, move right and find the 23'' MP row. Follow that row to
the right until intersecting the 3°C conditions and read 90 pph.
2. Determine the fuel flow at 2,500 RPM, 23'' MP and +23°C, 87 pph.
3. Average to determine fuel flow at +13°C, 2,500 RPM and 23'' MP:
(90 + 87) ÷ 2 = 88.5 pph
4. Calculate the available flight endurance using:
Time = Fuel ÷ Fuel Burn Rate = 460 ÷ 88.5 = 5.2 hours
5. Convert 0.2 hour into minutes:
0.2 x 60 = 12 minutes
6. Therefore, the flight time is 5 hours, 12 minutes.
577. GIVEN:
Weight A: 135 pounds at 15 inches aft of datum
Weight B: 205 pounds at 117 inches aft of datum
Weight C: 85 pounds at 195 inches aft of datum
Based on this information, the CG would be located how far aft of datum?
A. 100.2 inches.
B. 109.0 inches.
C. 121.7 inches.
https://online.prepware.com/quiz_review_graded?quiz_id=6393576 6/3/2024, 5 53 pm
Page 190 of 218
:
+ 85 x 195 = + 16,575 for C
425 lbs 42,585 lbs-in
2. Moments ÷ Total weight = CG location
42,585 ÷ 425 = 100.2 inches ^
578. If you experience an engine failure in a single-engine aircraft after takeoff, you
should
In the event of an engine failure on initial climb-out, the pilot's first responsibility is
to maintain aircraft control. At a climb pitch attitude without power, the airplane is
at or near a stalling angle of attack. At the same time, the pilot may still be holding
right rudder. The pilot must immediately lower the nose to prevent a stall while
moving the rudder to ensure coordinated flight. Attempting to turn back to the
takeoff runway should not be attempted. The pilot should establish a controlled
glide toward a plausible landing area, preferably straight ahead.
1. Construct a table with the item, weight, and moment. The moment of each item
is found by using the loading graph in FAA Figure 38. Locate the weight of each
item on the left side of the chart, and proceed to the right to intersect the correct
item line. At point of intersection, proceed downward to find the moment for that
item at the specified weight.
Item Weight Moment
Aircraft empty weight 1,271 102.04
Pilot and copilot 260 23.30
https://online.prepware.com/quiz_review_graded?quiz_id=6393576 6/3/2024, 5 53 pm
Page 191 of 218
:
Aft passenger 120 15.00
Cargo 60 6.80
Fuel (37.0 gal x 6 lbs/gal) + 222 + 20.00
Totals 1,933 167.14
2. Compare to the limits in FAA Figure 38. Using the CG envelope graph, move up
the loaded aircraft weight scale to 1,933, then across to 167.14 on the
moment/1,000 scale. The weight and CG are within limits.
A. 689 feet.
B. 716 feet.
C. 1,275 feet.
1. Enter the chart at the 70°F OAT point and proceed upward to intercept the sea
level pressure altitude line. Proceed right to the intersection with the vertical
reference line.
2. From the reference line, proceed upward to the right following the trend line,
until reaching a vertical line representing the weight of 3,400 pounds.
3. Draw a line to the right from the intercept with the weight line to the intercept
with the right vertical reference line.
4. Construct a vertical line to represent the headwind, 16 KIAS. Draw a line from
the intercept with the reference line to the intersect with the headwind line.
Proceed to the right and read the total distance over a 50-foot obstacle: 1,275
feet.
5. Apply the note to calculate the ground roll (53% of the landing distance).
1,275 x 0.53 = 676 feet ground roll.
Select the answer choice closest to this computation.
https://online.prepware.com/quiz_review_graded?quiz_id=6393576 6/3/2024, 5 53 pm
Page 192 of 218
:
A. 4 hours 50 minutes.
B. 5 hours 20 minutes.
C. 5 hours 59 minutes.
A. 30 pounds.
B. 37 pounds.
C. 46 pounds.
1. Determine the weight of the fuel required to climb from sea level to the PA of the
airport (4,000 feet) for an aircraft weight of 3,700 lbs: 12 lbs
2. Determine the weight of the fuel required to climb from sea level to 12,000-foot
PA: 37 lbs
3. Determine the weight of fuel required to climb from 4,000 to 12,000 feet,
uncorrected for nonstandard temperature:
37 - 12 = 25 lbs
4. Correct the fuel requirement for nonstandard temperature by applying Note 2,
using 20% increase (+21°C is +14°C, relative to the standard atmosphere at 4,000
feet):
1.20 x 25 = 30 lbs
5. Apply Note #1 to determine the total fuel requirement:
30 lbs + 16 lbs start and taxi = 46 lbs total.
https://online.prepware.com/quiz_review_graded?quiz_id=6393576 6/3/2024, 5 53 pm
Page 193 of 218
:
Empty weight moment (in-lb/1,000) 102.04
Pilot and copilot 360 lb
Cargo 340 lb
Fuel 37 gal
Will the CG remain within limits after 30 gallons of fuel has been used in flight?
C. Yes, but the CG will be located in the shaded area of the CG envelope.
1. Subtract your fuel burn of 30 gallons from your total fuel of 37 gallons to find
fuel remaining, 7 gallons.
2. Construct a table with the item, weight, and moment. The moment of each item
is found by using the loading graph in FAA Figure 38. Locate the weight of each
item on the left side of the chart, and proceed to the right to intersect the correct
item line. At point of intersection, proceed downward to find the moment for that
item at the specified weight.
Item Weight Moment
Aircraft empty weight 1,271 102.04
Pilot and copilot 360 32.80
Cargo 340 39.50
Fuel (7.0 gal x 6 lbs/gal + 42 + 4.00
Totals 2,013 178.34
3. Compare to the limits in FAA Figure 38. Using the CG envelope graph, move up
the loaded aircraft weight scale to 2,013, then across to 178.34 on the
moment/1,000 scale. The CG will remain within limits.
584. (Refer to Figure 8.) With 38 gallons of fuel aboard at cruise power (55 percent),
how much flight time is available with night VFR fuel reserve still remaining?
A. 2 hours 34 minutes.
B. 2 hours 49 minutes.
C. 3 hours 18 minutes.
1. Enter chart at the point where the 55% maximum continuous power line
intersects the curved cruise (lean) line.
2. From that point, proceed to the left and read fuel flow 11.4 GPH.
3. Compute time of flight:
38 gal ÷ 11.4 GPH = 3.33 hours
4. Compute available flight time with a 45-minute (0.75 hour) reserve:
3.33 - 0.75 = 2.58 = 2 hours, 34 minutes.
https://online.prepware.com/quiz_review_graded?quiz_id=6393576 6/3/2024, 5 53 pm
Page 194 of 218
:
GIVEN:
Pressure altitude 6,000 ft
Temperature -17°C
Power 2,300 RPM - 23'' MP
Usable fuel available 370 lb
What is the maximum available flight time under the conditions stated?
A. 4 hours 20 minutes.
B. 4 hours 30 minutes.
C. 4 hours 50 minutes.
1. Enter the Cruise Performance Chart at the 2,300 RPM section (the left-hand
column). In that section move right and find the 23'' MP row. Follow that row right
and read 82 pph under the -17°C columns.
2. Calculate the available flight endurance using:
Time = Fuel ÷ Fuel Burn Rate = 370 ÷ 82 = 4.51 hours
3. Convert 0.51 hour to minutes.
.51 x 60 = 30.6 minutes
4. Therefore, the flight time is 4 hours, 30 minutes.
586. GIVEN:
Weight A: 155 pounds at 45 inches aft of datum
Weight B: 165 pounds at 145 inches aft of datum
Weight C: 95 pounds at 185 inches aft of datum
Based on this information, where would the CG be located aft of datum?
A. 86.0 inches.
B. 116.8 inches.
C. 125.0 inches.
587. (Refer to Figure 3A.) What is the approximate glide distance if you are operating at
5,500 feet?
A. 10 NM.
https://online.prepware.com/quiz_review_graded?quiz_id=6393576 6/3/2024, 5 53 pm
Page 195 of 218
:
B. 8 NM.
C. 6 NM.
Follow a line to the right from the height above terrain of 5,500 feet until it
intersects the slanted line. From this intersection, draw a line downward until it
intersects the glide distance. This intersection is at 8 NM.
588. (Refer to Figure 8.) Approximately how much fuel would be consumed when
climbing at 75% power for 7 minutes?
A. 1.82 gallons.
B. 1.97 gallons.
C. 2.15 gallons.
1. Enter chart at point where the 75% maximum continuous power line intersects
the curved take off and climb line.
2. From that point proceed to the left and read fuel flow 18.4 GPH.
3. Compute fuel burn:
(18.4 x 7) ÷ 60 = 2.1 gallons.
A. 2 hours 27 minutes.
B. 3 hours 12 minutes.
C. 3 hours 42 minutes.
https://online.prepware.com/quiz_review_graded?quiz_id=6393576 6/3/2024, 5 53 pm
Page 196 of 218
:
GIVEN:
Temperature 80°F
Pressure altitude 4,000 ft
Weight 2,800 lb
Headwind 24 kts
What is the total landing distance over a 50-foot obstacle?
A. 1,125 feet.
B. 1,250 feet.
C. 1,325 feet.
1. Enter the chart at the 80°F OAT point and proceed upward to intercept the
4,000-foot pressure altitude line. Proceed right to the intersection with the vertical
reference line.
2. From the reference line, proceed upward to the right following the trend line,
until reaching a vertical line representing the weight of 2,800 pounds.
3. Draw a line to the right from the intercept with the weight line, to the intercept
with the right vertical reference line.
4. Construct a vertical line to represent the headwind, 24 knots. Draw a line from
the intercept with the reference line to the intercept with the headwind line.
Proceed to the right and read the total distance over a 50-foot obstacle: 1,125
feet.
591. (Refer to Figure 32.) Determine the approximate runway length necessary for
takeoff.
Given:
Temperature = 40 degrees F
Pressure altitude = 4,000 ft
Weight = 3,200 lbs
Headwind = 15 kts
A. 1,300 feet.
B. 850 feet.
C. 950 feet.
592. An airplane is loaded to a gross weight of 4,800 pounds, with three pieces of
https://online.prepware.com/quiz_review_graded?quiz_id=6393576 6/3/2024, 5 53 pm
Page 197 of 218
:
luggage in the rear baggage compartment. The CG is located 98 inches aft of
datum, which is 1 inch aft of limits. If luggage which weighs 90 pounds is moved
from the rear baggage compartment (145 inches aft of datum) to the front
compartment (45 inches aft of datum), what is the new CG?
A. 43 pounds.
B. 45 pounds.
C. 49 pounds.
1. Determine the weight of the fuel required to climb from sea level to the PA of the
airport (6,000 feet) by interpolating between the 4,000- and 8,000-foot values (for
an aircraft weight of 3,400 pounds):
(9 + 19) ÷ 2 = 14 lbs
2. Determine the weight of fuel required to climb from sea level to 16,000 feet PA:
39 lbs
3. Determine the weight of fuel required to climb from 6,000 feet to 16,000 feet,
uncorrected for nonstandard temperature:
39 - 14 = 25 lbs
4. Correct for nonstandard temperature, Note 2, using 7% increase (+10°C is 7°C
relative to the standard atmosphere at 6,000 feet):
1.07 x 25 = 26.7 lbs
5. Apply Note #1 to determine the total fuel requirement:
26.7 lbs + 16.0 lbs start and taxi = 42.7
= 43 lbs total.
https://online.prepware.com/quiz_review_graded?quiz_id=6393576 6/3/2024, 5 53 pm
Page 198 of 218
:
594. (Refer to Figure 3A.) What is the approximate glide distance if you are operating at
8,000 feet?
A. 12 NM.
B. 10 NM.
C. 14 NM.
Follow a line to the right from the height above terrain of 8,000 feet until it
intersects the slanted line. From this intersection, draw a line downward until it
intersects the glide distance. This intersection is at 12 NM.
A. 4 hours 17 minutes.
B. 4 hours 30 minutes.
C. 5 hours 4 minutes.
1. Enter chart at the point where the 55% maximum continuous power line
intersects the curved best power level flight line.
2. From that point, proceed to the left and read fuel flow: 13.0 GPH
3. Compute time of flight:
65 gal ÷ 13.0 = 5.0 hours
4. Compute available flight time with a 30-minute (0.5 hour) reserve:
5.0 - 0.5 = 4.5 = 4 hour, 30 minutes.
596. When computing weight and balance, the basic empty weight includes the weight
of the airframe, engine(s), and all installed optional equipment. Basic empty weight
also includes
all usable fuel, full oil, hydraulic fluid, but does not include the weight of pilot,
B.
passengers, or baggage.
all usable fuel and oil, but does not include any radio equipment or instruments
C.
that were installed by someone other than the manufacturer.
Basic empty weight includes unusable fuel, full operating fluids, and full oil.
https://online.prepware.com/quiz_review_graded?quiz_id=6393576 6/3/2024, 5 53 pm
Page 199 of 218
:
597. (Refer to Figure 31.) The surface wind is 180° at 25 knots. What is the crosswind
component for a Rwy 13 landing?
A. 19 knots.
B. 21 knots.
C. 23 knots.
A. 6 miles.
B. 8.5 miles.
C. 11 miles.
1. Enter the chart at -4°C and proceed upward until intersecting the 9,000-foot PA
line. From this point, proceed to the right until intersecting the Dist-Nautical Miles
curved line. From this point proceed downward and read the distance: 14.5 NM.
2. Enter chart at +12°C and proceed upward until intersecting the 4,000-foot PA
line. From this point, proceed to the right until intersecting the Dist-Nautical Miles
curved line. From this point, proceed downward and read the distance: 6 NM.
3. The difference is the distance required to climb:
14.5 - 6 = 8.5 NM.
Total moment is the weight of the airplane x the distance between the datum and
the CG. Therefore:
CG = Total moment ÷ Total weight.
https://online.prepware.com/quiz_review_graded?quiz_id=6393576 6/3/2024, 5 53 pm
Page 200 of 218
:
600. (Refer to Figure 8.) Determine the amount of fuel consumed during takeoff and
climb at 70 percent power for 10 minutes.
A. 2.66 gallons.
B. 2.88 gallons.
C. 3.2 gallons.
1. Interpolate a vertical line for 70% power vertically between the 65% and 75%
power lines.
2. From the point of intersection of the 70% line with the takeoff and climb curve,
draw a line to the left and read the fuel flow, 17.3 GPH.
3. Compute the fuel burned:
(17.3 x 10) ÷ 60 = 2.88 gallons.
A. -5°C.
B. -15°C.
C. +5°C.
Standard temperature at sea level is 15°C. The standard lapse rate is 2°C/1,000
feet.
10 x 2 = -20°C temperature decrease to 10,000 feet
+15° - 20° = -5°C standard temperature at 10,000 feet.
Course, time, speed, and distance computations in flight require the same basic
procedures as those used in preflight planning. However, because of cockpit
space limitations and available equipment, and because the pilot's attention must
be divided between solving the problem and operating the aircraft, the pilot must
take advantage of all possible shortcuts and rule-of-thumb computations.
Answer (A) is incorrect because any form of navigation is appropriate. Answer (B)
is incorrect because climbs may consume time and fuel.
https://online.prepware.com/quiz_review_graded?quiz_id=6393576 6/3/2024, 5 53 pm
Page 201 of 218
:
GIVEN:
Pressure altitude 18,000 ft
Temperature -21°C
Power 2,400 RPM - 28'' MP
Recommended lean mixture usable fuel 425 lb
What is the approximate flight time available under the given conditions? (Allow for
VFR day fuel reserve.)
A. 3 hours 46 minutes.
B. 4 hours 1 minute.
C. 4 hours 31 minutes.
604. (Refer to Figure 11.) What would be the endurance at an altitude of 7,500 feet,
using 52 percent power?
NOTE: (With 48 gallons fuel-no reserve.)
A. 6.1 hours.
B. 7.7 hours.
C. 8.0 hours.
605. (Refer to Figure 31.) If the tower-reported surface wind is 010° at 18 knots, what is
the crosswind component for a Rwy 08 landing?
A. 7 knots.
B. 15 knots.
C. 17 knots.
https://online.prepware.com/quiz_review_graded?quiz_id=6393576 6/3/2024, 5 53 pm
Page 202 of 218
:
080° runway direction - 010° wind direction = 070° wind angle across runway
2. Determine the crosswind component of 18 knots at 070°, 17 KIAS.
606. If fuel consumption is 14.7 gallons per hour and groundspeed is 157 knots, how
much fuel is required for an airplane to travel 612 NM?
A. 58 gallons.
B. 60 gallons.
C. 64 gallons.
607. An aircraft 60 miles from a VOR station has a CDI indication of one-fifth deflection,
this represents a course centerline deviation of approximately
A. 6 miles.
B. 2 miles.
C. 1 mile.
Aircraft displacement from course is approximately 200 feet per dot per nautical
mile. The CDI deflection indication is one-fifth deflection at 60 miles from the
station, and a one-fifth or one-dot deflection indicates a 2-mile displacement of
the aircraft from the course centerline.
608. How should the pilot make a VOR receiver check when the aircraft is located on
the designated checkpoint on the airport surface?
Set the OBS on 180° plus or minus 4°; the CDI should center with a FROM
A.
indication.
Set the OBS on the designated radial. The CDI must center within plus or minus 4°
B.
of that radial with a FROM indication.
With the aircraft headed directly toward the VOR and the OBS set to 000°, the CDI
C.
should center within plus or minus 4° of that radial with a TO indication.
https://online.prepware.com/quiz_review_graded?quiz_id=6393576 6/3/2024, 5 53 pm
Page 203 of 218
:
Caution: No correction other than the correction card figures supplied by the
manufacturer should be applied in making these VOR receiver checks.
609. When checking the course sensitivity of a VOR receiver, how many degrees should
the OBS be rotated to move the CDI from the center to the last dot on either side?
A. 5° to 10°.
B. 10° to 12°.
C. 18° to 20°.
610. (Refer to Figure 17.) Which illustration indicates that the airplane will intercept the
060 radial at a 60° angle inbound, if the present heading is maintained?
A. 6.
B. 4.
C. 5.
To intercept the 060° radial inbound (TO), the reciprocal (240°) would be selected.
Only illustrations 4, 5, and 6 have the 240° course selected. The 240° course may
be intercepted at a 60° angle with either a heading of 300° (240° + 60°) or 180°
(240° - 60°). The only illustration showing either is 6.
611. When using VOT to make a VOR receiver check, the CDI should be centered and
the OBS should indicate that the aircraft is on the
A. 090 radial.
B. 180 radial.
C. 360 radial.
To use the VOT service, tune in the VOT frequency on your VOR receiver. With the
Course Deviation Indicator (CDI) centered, the omnibearing selector (OBS) should
read 0° with the TO/FROM indication showing FROM, or the OBS should read
180° with the TO/FROM indication showing TO. Since VOR radials are always
expressed as FROM the station, the OBS should indicate the aircraft is on the
360° radial FROM.
612. To track inbound on the 215 radial of a VOR station, the recommended procedure
https://online.prepware.com/quiz_review_graded?quiz_id=6393576 6/3/2024, 5 53 pm
Page 204 of 218
:
is to set the OBS to
B. 215° and make heading corrections away from the CDI needle.
B. angles formed by isogonic lines and lines of latitude vary from point to point.
C. angles formed by lines of longitude and the course line vary from point to point.
Answer (A) is incorrect because isogonic lines are used to find magnetic course.
Answer (B) is incorrect because isogonic lines are used to find magnetic course.
https://online.prepware.com/quiz_review_graded?quiz_id=6393576 6/3/2024, 5 53 pm
Page 205 of 218
:
1. Calculate the time to climb:
Time = vertical distance ÷ vertical speed
Where vertical distance = 9,500 - 1,500 = 8,000 feet
Time = 8,000 feet ÷ 500 FPM = 16 minutes = 0.27 hour
2. Compute the fuel requirement:
14 gal/hr x 0.2666 hour = 3.73 gallons
3. Calculate the wind correction angle and ground speed using a wind triangle:
WCA = 5° left
Ground speed = 153 knots
4. Calculate true heading (TH = TC + WCA):
145° - 5° = 140° TH
5. Calculate compass heading (CH = TH + Var + Dev):
140° - 5° - 3° = 132°CH
6. Compute distance flown:
Distance = 153 knots x 0.27 hour = 41.31 NM.
615. If an aircraft is consuming 9.5 gallons of fuel per hour at a cruising altitude of
6,000 feet and the groundspeed is 135 knots, how much fuel is required to travel
420 NM?
A. 27 gallons.
B. 30 gallons.
C. 35 gallons.
616. For IFR operations off established airways, ROUTE OF FLIGHT portion of an IFR
flight plan should list VOR navigational aids which are no more than
A. 40 miles apart.
B. 70 miles apart.
C. 80 miles apart.
Operation off established airways below 18,000 feet MSL use aids not more than
80 NM apart. These aids are depicted on Enroute Low-Altitude Charts.
https://online.prepware.com/quiz_review_graded?quiz_id=6393576 6/3/2024, 5 53 pm
Page 206 of 218
:
A. 248 pounds.
B. 265 pounds.
C. 284 pounds.
618. What is the maximum bearing error (+ or -) allowed for an operational VOR
equipment check when using an FAA-approved ground test signal?
A. 4 degrees.
B. 6 degrees.
C. 8 degrees.
619. Given:
Pressure altitude 12,000 ft
True air temperature +50°F
From the conditions given, the approximate density altitude is
A. 11,900 feet.
B. 14,130 feet.
C. 18,150 feet.
Using an E6B:
1. Convert 50°F to °C using the temperature conversion table at the bottom of the
E6-B or the CX-3 Flight Computer. The result is 10°C.
2. Refer to the right-hand 'Density Altitude' window. Note that the scale above the
window is labeled air temperature (°C). The scale inside the window itself is
labeled pressure altitude (in thousands of feet). Rotate the disc and place the
pressure altitude of 12,000 feet opposite an air temperature of 10°C. The density
altitude shown in the density altitude window is 14,130 feet.
Using a CX-3:
1. From the FLT menu select Altitude.
2. Enter a PAlt of 12,000 FT and OAT of 50 degrees F to get a DAlt of 14,134 FT or
approximately 14,130 feet.
620. When navigating using only VOR/DME based RNAV, selection of a VOR NAVAID
https://online.prepware.com/quiz_review_graded?quiz_id=6393576 6/3/2024, 5 53 pm
Page 207 of 218
:
that does not have DME service will
VOR/DME-based RNAV units need both VOR and DME signals to operate in RNAV
mode. If the NAVAID selected is a VOR without DME, RNAV mode will not
function. When DME is not available the RNAV unit will function as a VOR receiver
with DME capability in VOR (or non-RNAV) mode.
621. (Refer to Figure 17.) Which illustration indicates that the airplane should be turned
150° left to intercept the 360 radial at a 60° angle inbound?
A. 1.
B. 2.
C. 3.
Only two headings intercept the 360° radial inbound at a 60° angle: 120° and 240°.
An aircraft heading 270° could turn left 150° to intercept at a 60° angle on a
heading of 120°. An aircraft heading 030° could turn left 150° to intercept at a 60°
angle on a heading of 240°. Illustration 1 shows the aircraft flying one of the two
possible headings.
622. If an airplane is consuming 14.8 gallons of fuel per hour at a cruising altitude of
7,500 feet and the groundspeed is 167 knots, how much fuel is required to travel
560 NM?
A. 50 gallons.
B. 53 gallons.
C. 57 gallons.
623. When the CDI needle is centered during an airborne VOR check, the omnibearing
selector and the TO/FROM indicator should read
https://online.prepware.com/quiz_review_graded?quiz_id=6393576 6/3/2024, 5 53 pm
Page 208 of 218
:
C. 0° TO, only if you are due south of the VOR.
If neither a test signal nor a designated checkpoint on the surface is available, use
an airborne checkpoint designated by the Administrator or, outside the United
States, by appropriate authority. The maximum permissible bearing error is ±6°.
624. GIVEN:
Distance off course 9 mi
Distance flown 95 mi
Distance to fly 125 mi
To converge at the destination, the total correction angle would be
A. 4°.
B. 6°.
C. 10°.
625. GIVEN:
True course 105°
True heading 085°
True airspeed 95 kts
Groundspeed 87 kts
Determine the wind direction and speed.
https://online.prepware.com/quiz_review_graded?quiz_id=6393576 6/3/2024, 5 53 pm
Page 209 of 218
:
3. Determine the WCA by comparing course to heading.
105° - 085° = 20° left
4. Plot WCA at TAS arc with a pencil dot, 95 knots.
5. Rotate compass azimuth so pencil dot is on the centerline.
6. Read wind direction under true index and wind speed as number of units
between grommet and pencil dot, 020° and 32 knots.
626. GIVEN:
Pressure altitude 6,000 ft
True air temperature +30°F
From the conditions given, the approximate density altitude is
A. 9,000 feet.
B. 5,500 feet.
C. 5,000 feet.
Using an E6B:
1. Convert 30°F to °C using the temperature conversion table at the bottom of the
E6-B or the CX-3 Flight Computer. The result is -1°C.
2. Refer to the right-hand 'Density Altitude' window. Note that the scale above the
window is labeled air temperature (°C). The scale inside the window itself is
labeled pressure altitude (in thousands of feet). Rotate the disc and place the
pressure altitude of 6,000 feet opposite an air temperature of -1°C. The density
altitude shown in the window is 5,500 feet.
Using a CX-3:
1. From the FLT menu select Altitude.
2. Enter a PAlt of 6,000 FT and OAT of 30 degrees F to get a DAlt of 5,494 FT or
approximately 5,500 feet.
627. GIVEN:
Pressure altitude 5,000 ft
True air temperature +30°C
From the conditions given, the approximate density altitude is
A. 7,200 feet.
B. 7,800 feet.
C. 9,000 feet.
1. Refer to the right-hand 'Density Altitude' window. Note that the scale above the
window is labeled air temperature (°C). The scale inside the window itself is
labeled pressure altitude (in thousands of feet). Rotate the disc and place the
pressure altitude of 5,000 feet opposite an air temperature of 30°C.
2. The density altitude shown in the density altitude window is 7,800 feet.
https://online.prepware.com/quiz_review_graded?quiz_id=6393576 6/3/2024, 5 53 pm
Page 210 of 218
:
628. As air temperature increases, density altitude will
A. decrease.
B. increase.
Density altitude is the altitude in standard air where the density is the same as the
existing density. It is affected by the pressure, temperature, and moisture content
of the air. Both a decrease in pressure and an increase in temperature decrease
the density of the air and increase the density altitude.
629. GIVEN:
Pressure altitude 7,000 ft
True air temperature +15°C
From the conditions given, the approximate density altitude is
A. 5,000 feet.
B. 8,500 feet.
C. 9,500 feet.
1. Refer to the right-hand 'Density Altitude' window. Note that the scale above the
window is labeled air temperature (°C). The scale inside the window itself is
labeled pressure altitude (in thousands of feet). Rotate the disc and place the
pressure altitude of 7,000 feet opposite an air temperature of 15°C.
2. The density altitude shown in the window is 8,500 feet.
630. (Refer to Figure 17.) Which illustration indicates that the airplane will intercept the
060 radial at a 75° angle outbound, if the present heading is maintained?
A. 4.
B. 5.
C. 6.
Only two possible headings will intercept the 060° radial at a 75° angle outbound:
135° and 345°. The only one of those shown is illustration 5. The illustration shows
that if the aircraft turned to 060°, it would be going FROM the station and would
have to fly left to get on the 060 radial.
https://online.prepware.com/quiz_review_graded?quiz_id=6393576 6/3/2024, 5 53 pm
Page 211 of 218
:
Airport elevation 1,700 ft
Descends to 1,000 ft AGL
Rate of descent 600 ft/min
Average true airspeed 135 kts
True course 263°
Average wind velocity 330° at 30 kts
Variation 7° E
Deviation +3°
Average fuel consumption 11.5 gal/hr
Determine the approximate time, compass heading, distance, and fuel consumed
during the descent.
https://online.prepware.com/quiz_review_graded?quiz_id=6393576 6/3/2024, 5 53 pm
Page 212 of 218
:
A. 14 minutes, 234°, 26 NM, 3.9 gallons.
633. GIVEN:
True course 345°
True heading 355°
True airspeed 85 kts
Groundspeed 95 kts
Determine the wind direction and speed.
634. Which data must be recorded in the aircraft logbook or other record by a pilot
https://online.prepware.com/quiz_review_graded?quiz_id=6393576 6/3/2024, 5 53 pm
Page 213 of 218
:
making a VOR operational check for IFR operations?
VOR name or identification, place of operational check, amount of bearing error,
A.
and date of check.
C. VOR name or identification, amount of bearing error, date of check, and signature.
Each person making a VOR operational check shall enter the date, place, bearing
error, and sign the aircraft log or other record.
Setting the OBS to a bearing that is 90° from the bearing on which the aircraft is
B.
located.
Failing to change the OBS from the selected inbound course to the outbound
C.
course after passing the station.
With the reciprocal of the inbound course set on the OBS and the indicator
showing FROM, the aircraft will be turned away from the needle for direct return to
course centerline. This is called reverse sensing.
636. When must an operational check on the aircraft VOR equipment be accomplished
to operate under IFR? Within the preceding
C. 30 days.
No person may operate a civil aircraft under IFR using the VOR system of radio
navigation unless the VOR equipment of that aircraft has been operationally
checked within the preceding 30 days, and was found to be within permissible
limits.
637. If fuel consumption is 80 pounds per hour and groundspeed is 180 knots, how
much fuel is required for an airplane to travel 460 NM?
A. 205 pounds.
B. 212 pounds.
C. 460 pounds.
https://online.prepware.com/quiz_review_graded?quiz_id=6393576 6/3/2024, 5 53 pm
Page 214 of 218
:
460 NM ÷ 180 knots = 2.556 hours
2. Determine the fuel burn:
80 lbs/hr x 2.56 hours = 204.8 pounds.
638. If an airplane is consuming 12.5 gallons of fuel per hour at a cruising altitude of
8,500 feet and the groundspeed is 145 knots, how much fuel is required to travel
435 NM?
A. 27 gallons.
B. 34 gallons.
C. 38 gallons.
639. To track outbound on the 180 radial of a VOR station, the recommended
procedure is to set the OBS to
B. 180° and make heading corrections away from the CDI needle.
https://online.prepware.com/quiz_review_graded?quiz_id=6393576 6/3/2024, 5 53 pm
Page 215 of 218
:
during the descent.
641. GIVEN:
Wind 175° at 20 kts
Distance 135 NM
True course 075°
True airspeed 80 kts
Fuel consumption 105 lb/hr
Determine the time en route and fuel consumption.
https://online.prepware.com/quiz_review_graded?quiz_id=6393576 6/3/2024, 5 53 pm
Page 216 of 218
:
642. An airplane descends to an airport under the following conditions:
Cruising altitude 6,500 ft
Airport elevation 700 ft
Descends to 800 ft AGL
Rate of descent 500 ft/min
Average true airspeed 110 kts
True course 335°
Average wind velocity 060° at 15 kts
Variation 3° W
Deviation +2°
Average fuel consumption 8.5 gal/hr
Determine the approximate time, compass heading, distance, and fuel consumed
during the descent.
643. (Refer to Figure 17.) Which statement is true regarding illustration 2, if the present
heading is maintained? The airplane will
Illustration 2 shows the aircraft heading 225°. The bearing pointer indicates a
https://online.prepware.com/quiz_review_graded?quiz_id=6393576 6/3/2024, 5 53 pm
Page 217 of 218
:
heading of 235° will take the aircraft to the station; therefore, you are on the 055°
radial. If the present heading is maintained, the station will remain to the right of
the aircraft and you will cross the 180° radial at approximately a 45° angle
outbound (225° - 180° = 45°).
644. What procedure could a pilot use to navigate under VFR from one point to another
when ground references are not visible?
A. Dead reckoning.
B. Pilotage.
645. (Refer to Figure 17.) Which is true regarding illustration 4, if the present heading is
maintained? The airplane will
Illustration 4 shows the aircraft heading 255° with a magnetic bearing of 275° to
the station. The aircraft will pass south of the station and cross the 180° radial at a
75° angle (255° - 180° = 75°).
© 2023 Aviation Supplies & Academics, Inc. All rights reserved. | 1-800-ASA-2FLY (tel:1-800-ASA-2FLY) | Website: 10_2023 DB
www.asa2fly.com (https://www.asa2fly.com) | Email: [email protected] (mailto:[email protected]) (v1.36
(/whats_new))
https://online.prepware.com/quiz_review_graded?quiz_id=6393576 6/3/2024, 5 53 pm
Page 218 of 218
: